Ultimate NEET-PG Exam Study Material

Proven Effective Content with 96% Strike Rate

NEET PG Previous Year Question Papers of Last 5 Years

Aug 4, 2023

Navigate Quickly

NEET PG 2023 Previous Year Questions

Microbiology

Anatomy

Topic: Abdomen

Biochemistry

Topic: Carbohydrate Metabolism

Parasitology

Radiology

Microbiology

Ophthalmology

Topic: Neuro Ophthalmology

Topic: Cornea

Surgery

Topic: Urology

Topic: Gastrointestinal Surgery

Dermatology

Anaesthesia

ENT

Pharmacology

Orthopaedics

Psychiatry

Physiology

Medicine

Topic: Endocrinology

PSM

Topic: Epidemiology

Gynaecology & Obstetrics

Forensic Medicine

Pediatrics

NEET PG 2022 Previous Year Questions

Microbiology

Microbiology

Anatomy

Biochemistry

Biochemistry

Ophthalmology

Surgery

Dermatology

Pharmacology

Orthopaedics

Psychiatry

Physiology

Medicine

Pediatrics

PSM

Gynaecology & Obstetrics

Forensic Medicine

NEET PG 2021 Previous Year Questions

Microbiology

Radiology

Anatomy

Biochemistry

Ophthalmology

Surgery

Dermatology

Anaesthesia

ENT

Pharmacology

Orthopaedics

Psychiatry

Physiology

Medicine

Pediatrics

PSM

Gynaecology & Obstetrics

Forensic Medicine

NEET PG 2020 Previous Year Questions

Microbiology

Radiology

Anatomy

Biochemistry

Ophthalmology

Surgery

Dermatology

Anaesthesia

ENT

Pharmacology

Orthopaedics

Psychiatry

Physiology

Medicine

Pediatrics

PSM

Gynaecology & Obstetrics

Forensic Medicine

NEET PG 2019 Previous Year Questions

Microbiology

Radiology

Anatomy

Biochemistry

Ophthalmology

Surgery

Dermatology

Anaesthesia

ENT

Pharmacology

Orthopaedics

Psychiatry

Physiology

Medicine

Pediatrics

PSM

Gynaecology & Obstetrics

Forensic Medicine

neet pg previous years question papers

Preparing for the NEET PG exam can be incredibly challenging. Aspirants require the best learning resources and strategies to ace the exam. One extremely effective tool to boost preparations is solving the NEET PG previous year question papers. 

Previous year questions provide valuable insights into the exam pattern and also help gauge your preparation level. 

In this blog, we bring you recall based NEET PG previous year question papers from the last 5 years. Leverage these to boost your confidence, refine your preparation, and succeed in the upcoming NEET PG 2024 exam. Let's dive in!"


PrepLadder 6.0

NEET PG 2023 Previous Year Questions

NEET PG 2023 was conducted on March 5, 2023, with a vast number of aspirants participating in the examination, aiming to secure a seat in postgraduate medical courses across India.

Microbiology

Topic: Virology

Q.1. A patient hailing from Delhi presents with fever, arthralgia, and extensive petechial rash for 3 days. Lab investigations revealed a haemoglobin of 9 g/dL, a white blood cell count of 9000 cells/mm3, a platelet count of 20000 cells/mm3, and a prolonged bleeding time. The clotting time was normal. What is the most likely diagnosis?
1. Dengue
2. Malaria
3. Scrub typhus
4. Typhoid

Correct Answer: Dengue

Anatomy

Topic: Head and Neck
Q.2. A student had his jaw locked while yawning. Which of the following muscles is attached to the articular disc of the temporomandibular joint?
1. Lateral pterygoid
2. Temporalis
3. Medial pterygoid
4. Masseter

Correct Answer: Lateral pterygoid

Topic: Abdomen

Q.3. A patient comes with abdominal pain, jaundice, and portal hypertension. Anastomosis between which of the following veins is seen?
1. Left colic vein and middle colic veins
2. Esophageal veins and left gastric veins
3. Superior rectal and phrenic veins
4. Sigmoid and superior rectal veins

Correct Answer: Esophageal veins and left gastric veins

Biochemistry

Topic: Vitamins

Q.4. A patient presented with diarrhea, dermatitis, and dementia. Which of the following vitamin deficiencies would you suspect in this patient?
1. Vitamin B3 deficiency
2. Vitamin B1 deficiency
3. Vitamin B6 deficiency
4. Vitamin B12 deficiency

Correct Answer: Vitamin B3 deficiency

Topic: Carbohydrate Metabolism

Q.5. A young boy presents to the OPD with hypoglycemia and muscle cramps, on exertion or while playing. Then he becomes normal after resting for a while. These episodes are recurrent after a period of activity. He has decreased serum lactate and glucose levels. Which of the following diseases is he most likely to be suffering from?
1. McArdle disease
2. Hers disease
3. Cori's disease
4. Andersen disease

Correct Answer: McArdle disease

Parasitology

Topic: Parasitology

Q.6. Which of the following statements is true about Trichomonas vaginalis?
1. It cannot be cultured
2. Twitching motility is seen on wet saline mount
3. Cysts are seen on wet saline mount of vaginal secretions
4. It is not a sexually transmitted infection

Correct Answer: Twitching motility is seen on wet saline mount

Radiology

Topic: Head & Neck Imaging

Q.7. A 2-month-old infant is brought to the OPD with a parietal swelling present since birth. The X-ray image is given below. What could be the probable diagnosis?
1. Subgaleal hematoma
2. Cephalhematoma
3. Caput succedaneum
4. Enecephalocele

Correct Answer: Cephalhematoma

Topic: Genitourinary Radiology

Q.8. A patient presented with abdominal pain and sterile pyuria, and the x-ray showed the following features. What is the likely diagnosis?
1. Putty kidney
2. Nephrocalcinosis
3. Staghorn calculus
4. Psoas calcification

Correct Answer: Putty kidney

Microbiology

Topic: Virology

Q.9. A patient hailing from Delhi presents with fever, arthralgia, and extensive petechial rash for 3 days. Lab investigations revealed a hemoglobin of 9 g/dL, a white blood cell count of 9000 cells/mm3, a platelet count of 20000 cells/mm3, and a prolonged bleeding time. The clotting time was normal. What is the most likely diagnosis?
1. Dengue
2. Malaria
3. Scrub typhus
4. Typhoid

Correct Answer: Dengue

Ophthalmology

Topic: Neuro Ophthalmology

Q.10. A female presents with loss of vision in the right halves of both eyes. Where is the lesion located in the optic pathway?
1. Left optic tract
2. Optic radiation
3. Optic chiasma
4. Right optic tract

Correct Answer: Left optic tract

Topic: Cornea

Q.11. The eye examination of a 20-year-old male shows a golden ring on the iris. What is the next investigation to diagnose the condition?
1. Alpha-1-antitrypsin
2. Alpha-fetoprotein
3. Serum Iron
4. Serum ceruloplasmin

Correct Answer: Serum ceruloplasmin

Surgery

Topic: Urology

Q.12. A 55-year-old male presented with verrucous carcinoma around the glans of the penis. Examination reveals that the inguinal lymph nodes are not enlarged. What is the appropriate management for this patient?
1. Total penectomy
2. CO2 laser excision
3. Topical 5-fluorouracil
4. Partial penectomy

Correct Answer: Partial penectomy

Topic: Gastrointestinal Surgery

Q.13. Which of the following is the most common complication following ligation of the first vessel during abdominoperineal resection for rectal carcinoma?
1. Parasympathetic-bladder dysfunction and retrograde ejaculation.
2. Sympathetic-bladder dysfunction and impotence.
3. Sympathetic-retrograde ejaculation and bladder dysfunction.
4. Sympathetic-impotence and loss of cutaneous sensation in the perineal region

Correct Answer: Sympathetic-retrograde ejaculation and bladder dysfunction.

Dermatology

Topic: Introduction to Dermatology

Q.14. A female patient presented with acne that is not resolving on oral isotretinoin and antibiotics therapy. Which of the following is the next best investigation?
1. Look for dietary triggers.
2. Evaluate for hyperandrogenism
3. Check for antibiotic resistance
4. Look for drug triggers

Correct Answer: Evaluate for hyperandrogenism

Topic: Sexually Transmitted Infections

Q.15. A young woman complains of a painless ulcer in the genital area. It is associated with non-tender inguinal lymphadenopathy. What is the most likely diagnosis?
1. Chancroid
2. Syphilis
3. Herpes genitalis
4. Granuloma inguinale

Correct Answer: Syphilis

Anaesthesia

Topic: Pre-Anaesthetic Evaluation

Q.16. Which of the following is the drug of choice for preoperative antibiotic prophylaxis in a patient undergoing cardiac surgery?
1. Penicillin G
2. Erythromycin
3. Azithromycin
4. Cefazolin

Correct Answer: Cefazolin

ENT

Topic: Ear

Q.17. A female patient with hearing loss is examined and is found to be Rinne negative at 256 Hz and 512 Hz, while Rinne positive at 1024 Hz. What is the expected air conduction and bone conduction gap?
1. 30-45 dB
2. 15-30 dB
3. 45-60 dB
4. >60 dB

Correct Answer: 30-45 dB

Topic: Pharynx

Q.18. A 10-year-old child presents with throat pain, fever, and ear pain. He is diagnosed with recurrent tonsillitis. Which nerve is responsible for the ear pain in this patient?
1. Tympanic branch of the glossopharyngeal nerve
2. Greater auricular nerve
3. Auriculotemporal nerve
4. Auricular branch of the vagus nerve

Correct Answer: Tympanic branch of the glossopharyngeal nerve

Pharmacology

Topic: Antimicrobials : Antibacterial Drugs

Q.19. A bronchial asthma patient on inhalational steroids presented with white patchy lesions on the tongue and buccal mucosa. What is the drug that can be used to treat this condition?
1. Clotrimazole
2. Griseofulvin
3. Terbinafine
4. Flucytosine

Correct Answer: Clotrimazole

Topic: General Pharmacology

Q.20. A pregnant woman with a history of bronchial asthma is in the third stage of labour. Which drugs should be avoided in managing postpartum haemorrhage in this mother?
1. Carboprost
2. Oxytocin
3. Dinoprostone
4. Methyl ergometrine

Correct Answer: Carboprost

Orthopaedics

Topic: Peripheral Nerve Injuries

Q.21. A patient at the orthopaedics OPD complains of troubled sleep at night due to numbness and tingling sensation involving his lateral 3 digits. His symptoms are relieved as he lays his arms hanging from the bed. Which of the following options correctly describes his condition and the test used to assess it?
1. Guyon's canal syndrome, Froment's test
2. Carpal tunnel syndrome, Froment's test
3. Guyon's canal syndrome, Durkan's test
4. Carpal tunnel syndrome, Durkan's test

Correct Answer: Carpal tunnel syndrome, Durkan's test

Topic: Metabolic Disorders

Q.22. A child is brought to the orthopaedics OPD with a deformity in the lower limb and hyperpigmented skin lesions. The x-ray of her thigh is shown below. What is the most likely diagnosis?
1. Non-ossifying fibroma
2. Fibrous dysplasia
3. Paget's disease
4. Osteogenesis imperfecta

Correct Answer: Fibrous dysplasia

Psychiatry

Topic: Schizophrenia Spectrum and Other Psychotic Disorders

Q.23. A schizophrenic patient was prescribed drug A after he did not respond to haloperidol and thioridazine. He now presents with excessive salivation, an increase in blood glucose, and hyperlipidemia. What is drug A?
1. Ziprasidone
2. Risperidone
3. Clozapine
4. Aripiprazole

Correct Answer: Clozapine

Topic: Mood Disorders

Q.24. A woman, who is 4 days postpartum, presented with tearfulness, mood swings, and occasional insomnia. What is the likely diagnosis?
1. Postpartum depression
2. Postpartum blues
3. Postpartum psychosis
4. Postpartum anxiety

Correct Answer: Postpartum blues

Physiology

Topic: General Physiology

Q.25. A man was brought to the emergency after suddenly becoming unconscious while working in the field. On examination, his temperature was 105 degrees Fahrenheit, and his skin turgor was decreased. Which of the following would not be seen in the patient?
1. Tachypnea
2. Hypotension
3. Sweating
4. Red hot skin

Correct Answer: Sweating

Topic: Respiratory System

Q.25. A preterm baby who was delivered at 28 weeks developed respiratory distress syndrome. Which of the following is true about surface tension and compliance in this baby?
1. Surface tension - decreased; Compliance - increased
2. Surface tension - increased; Compliance - decreased
3. Both surface tension and compliance decreased
4. Both surface tension and compliance increased

Correct Answer: Surface tension - increased; Compliance - decreased

Medicine

Topic: Nephrology / Kidney Disease

Q.26. A patient with hyperkalemia and elevated urea levels underwent dialysis. Towards the end of the session, she became drowsy and had a sudden seizure episode. On examination, the patient was hypotensive. What is the treatment for this condition?
1. Bumetanide
2. Ethacrynic acid
3. Nesiritide
4. IV Mannitol

Correct Answer: IV Mannitol

Topic: Endocrinology

Q.27. A female patient presents to the emergency department with severe restlessness, palpitations, and tremors. She is a known case of bronchial asthma. On examination, the neck looks swollen. Blood pressure is elevated, and tachycardia is noted. ECG shows atrial fibrillation. Which of the following drugs is used for immediate management in this patient?
1. Diltiazem
2. Propranolol
3. Esmolol
4. Propylthiouracil

Correct Answer: Diltiazem

PSM

Topic: Nutrition and Health

Q.28. In a village, it is observed that several farmers have crossed gait and use a stick for support to stand up and walk. Due to poor yield from farms, they consume meals containing rice and pulses only. Supplementing their diet with which of the following vitamins could have prevented this?
1. Vitamin A
2. Vitamin D
3. Vitamin C
4. Vitamin B

Correct Answer: Vitamin C

Topic: Epidemiology

Q.29. Research is being conducted to find the association between aniline dye exposure and bladder cancer in workers who have worked in the industry for >20 years. Two groups were formed: one directly involved with dye handling and the other group consisting of office clerks not directly exposed to the dye. Years of occupation were noted from records. What type of study is being performed?
1. Retrospective cohort study
2. Prospective cohort study
3. Case-control study
4. Intervention and response

Correct Answer: Retrospective cohort study

Gynaecology & Obstetrics

Topic: Obstetrics

Q.30. A woman presents to you at 36 weeks of gestation with complaints of breathlessness and excessive abdominal distension. Fetal movements are normal. On examination, fetal parts are not easily felt and the fetal heartbeat is heard but it is muffled. Her symphysis fundal height is 41 cm. Her abdomen is tense but not tender. What is the most likely diagnosis?
1. Abruptio placenta
2. Hydrocephalus of fetus
3. Polyhydramnios
4. Fetal-maternal ascites

Correct Answer: Polyhydramnios

Topic: Obstetrics

Q.31. A primigravida presents to you with anemia early in her pregnancy. She is 7 weeks pregnant as seen on ultrasound. Her hemoglobin level is 9 g/dL. When should the iron supplements be started for her?
1. 10 to 12 weeks
2. 8 to 10 weeks
3. After 14 weeks
4. After 20 weeks

Correct Answer: 8 to 10 weeks

Forensic Medicine

Topic: Forensic Toxicology

Q.32. A child before playing consumed fruit from the garden. After some time he developed a high fever, confusion, photophobia, and unable to urinate. What are the likely causative agent and the appropriate antidote used in this case?
1. Datura, Pralidoxime
2. Datura, Physostigmine
3. Yellow oleander, Pralidoxime
4. Yellow oleander, Physostigmine

Correct Answer: Datura, Physostigmine

Topic: Forensic Thanatology

Q.33. The method of autopsy carried out en masse to remove from tongue to prostate is.
1. Virchow technique
2. Rokitansky technique
3. Ghon technique
4. Letulle technique

Correct Answer: Letulle technique

Pediatrics

Topic: Fluid and Electrolyte Disturbances

Q.34. A 10-year-old child weighing 30 kg presents with a history of loose stools for 2 days. On examination, there is severe dehydration. Laboratory investigations are as follows. What is the initial management as per ISPAD guidelines?

RBS550 mg/dL
pH7.01
Na+158mEq/L
Urine glucose3+
  1. Manage ABC, NS 20 mL/kg and start insulin after 1 hour
  2. Manage ABC, NS 20 mL/kg along with insulin 0.1 IU/kg/hr
  3. Manage ABC, NS 10 mL/kg along with insulin 0.1 IU/kg/hr
  4. Manage ABC, NS 10 mL/kg and start insulin after 1 hour

Correct Answer: Manage ABC, NS 20 mL/kg and start insulin after 1 hour

Topic: Growth

Q.35. Which of the following is the best sign to indicate adequate growth in an infant with a birth weight of 2.8 kg?

  1. Increase in length of 25 centimetres in the first year
  2. Weight gain of 300 grams per month till 1 year
  3. Anterior fontanelle closure by 6 months of age
  4. Weight under the 75th percentile and height under the 25th percentile

Correct Answer: Increase in length of 25 centimetres in the first year.

Download complete

NEET PG 2022 Previous Year Questions

NEET PG 2022 was conducted on May 21, 2022, as a national-level entrance examination for postgraduate medical courses in India. It saw a substantial number of candidates vying for admission to various medical institutes across the country. Check out the NEET PG 2022 previous year questions below - 

Microbiology

Topic: Parasitology

Q.1. Microfilariae with sheathed tail and two nuclei at the tail is suggestive of?
1. Wuchereria bancrofti
2. Brugia malayi
3. Loa loa
4. Onchocerca volvulus

Correct Answer: Brugia malayi

Microbiology

Topic: Systemic Bacteriology

Q.2. A militant presents with rashes all over his body sparing the palms and soles. On examination, he was febrile and lice were noted. Which of the following is responsible for his condition?
1. Rickettsia typhi
2. Rickettsia prowazekii
3. Rickettsia akari
4. Rickettsia conorii

Correct answer: Rickettsia prowazekii

Anatomy

Topic: Lower Limb

Q.3. A patient underwent surgery for the varicose vein. He now complains of sensory loss over the medial aspect of the leg and foot. Which of the following nerves is most likely to be injured?
1. Sural nerve
2. Superficial peroneal nerve
3. Deep peroneal nerve
4. Saphenous nerve

Correct Answer: Saphenous nerve

Biochemistry

Topic: Carbohydrate Metabolism

Q.4. A patient from a Mediterranean country visits Africa, where he develops malaria. He is treated with primaquine and later develops hemolytic anemia. Deficiency of an enzyme involved in which of the following pathways could be the cause?
1. Glycolysis
2. Gluconeogenesis
3. Hexose monophosphate (HMP) pathway
4. Luebering-Rapoport pathway

Correct Answer: Hexose monophosphate (HMP) pathway

Biochemistry

Topic: Genetics

Q.5. Which of the following disorders follows autosomal recessive inheritance pattern?
1. Huntington’s disease
2. Treacher Collins syndrome
3. Cystic fibrosis
4. Achondroplasia

Correct Answer: Cystic fibrosis

Ophthalmology

Topic: Uveitis

Q.6. A patient presents with a history of penetrating injury to the eye. A diagnosis of sympathetic ophthalmitis was confirmed. Which of the following will be seen?
1. Acute anterior uveitis
2. Pars planitis
3. Panuveitis
4. Chronic anterior uveitis

Correct Answer: Panuveitis

Topic: Glaucoma

Q.7. A diabetic patient presents to you with visual acuity of 6/9 in one eye. Further investigations revealed preretinal hemorrhages with neovascularization at the optic disc. What is the next step in management?
1. Focal laser photocoagulation
2. Pan-retinal photocoagulation
3. Grid laser photocoagulation
4. Scleral buckling

Correct Answer: Pan-retinal photocoagulation

Surgery

Topic: Endocrine Surgery

Q.8. A 45-year-old female patient underwent a thyroidectomy. Three days after the surgery, she developed perioral numbness. Which of the following investigations need to be done for her?
1. Free T3, T4
2. T3, T4, thyroid–stimulating hormone
3. Radioiodine scan
4. Calcium, phosphate, and parathormone levels

Correct Answer: Calcium, phosphate, and parathormone levels

Topic: Cardiothoracic Vascular Surgery

Q.9. Which of the following is most likely to be seen due to the rupture of a saccular aneurysm?
1. Subdural haemorrhage
2. Subarachnoid haemorrhage
3. Intracerebral haemorrhage
4. Hydrocephalus

Correct Answer: Subarachnoid haemorrhage

Dermatology

Topic: Sexually Transmitted Infections

Q.10. A patient presents to you with multiple anogenital warts. The biopsy of these lesions showed squamous atypia. Which of the following human papillomavirus types are considered high-risk?
1. HPV 2
2. HPV 18
3. HPV 6
4. HPV 11

Correct Answer: HPV 18

Topic: Cutaneous Infections

Q.11. A farmer presents you with a cauliflower-shaped mass on foot, which developed after a minor injury. Microscopy shows copper penny bodies. What is the most likely diagnosis?
1. Chromoblastomycosis
2. Blastomycosis
3. Sporotrichosis
4. Phaeohyphomycosis

Correct Answer: Chromoblastomycosis

Pharmacology

Topic: Respiratory Pharmacology

Q.12. A male patient with chronic obstructive pulmonary disease (COPD) was prescribed theophylline. He noticed that his urine output had increased the following day. This action of the drug is mediated through which of the following receptors?
1. Interleukin – 10
2. Histone deacetylase
3. Adenosine A1
4. Beta 2 adrenergic receptors

Correct Answer: Adenosine A1

Topic: Endocrine Pharmacology

Q.13. Which of the following drugs is not likely to cause Pulmonary fibrosis?
1. Metformin
2. Methotrexate
3. Bleomycin
4. Nitrofurantoin

Correct Answer: Metformin

Orthopaedics

Topic: Metabolic Disorders

Q.14. An intrauterine scan at the 13th week of pregnancy showed a fetus with multiple long bone fractures. What is commonly associated with this finding?
1. Achondroplasia
2. Osteogenesis imperfecta
3. Cretinism
4. Marfan syndrome

Correct Answer: Osteogenesis imperfecta

Topic: Spine + Pelvis + Lower Limb Traumatology

Q.15. A male patient presented with a bone fracture following a road traffic accident. After 2 days he developed dyspnea, petechiae involving the whole body, and a fall in oxygen saturation. What is the likely diagnosis?
1. Fat embolism
2. Air embolism
3. Venous thromboembolism
4. Pulmonary hypertension

Correct Answer: Fat embolism

Psychiatry

Topic: Eating Disorders

Q.16. A 16-year-old girl has intense cravings for food. She eats large amounts of food, which is followed by self-induced vomiting. What is the probable diagnosis?

  1. Anorexia nervosa
  2. Bulimia nervosa
  3. Atypical depression
  4. Binge eating disorder

Correct Answer: Bulimia nervosa

Topic: Mood Disorders

Q.17. A woman, who is 4 days postpartum, presented with tearfulness, mood swings, and occasional insomnia. What is the likely diagnosis?

  1. Postpartum depression
  2. Postpartum blues
  3. Postpartum psychosis
  4. Postpartum anxiety

Correct Answer: Postpartum blues

Physiology

Topic: Nerve Muscle Physiology

Q.18. A person after sleeping overnight with the arm under his head now experiences paresis but no numbness in the morning. Which of the following is the best explanation for it?

  1. C fibres are more sensitive to pressure than A fibres
  2. A fibres are more sensitive to hypoxia than B fibres
  3. A fibres are more susceptible to pressure changes than C fibres
  4. A fibres are more susceptible to hypoxia than C fibres

Correct Answer: A fibres are more susceptible to pressure changes than C fibres

Topic: The Nervous System

Q.19. A 65-year-old suffered from a stroke 2 days ago. He now presents with involuntary, violent, and flinging movements of the limbs on one side. What is the likely site of lesion in this patient?

  1. Subthalamic nuclei
  2. Globus pallidus
  3. Putamen
  4. Caudate nucleus

Correct Answer: Subthalamic nuclei

Medicine 

Topic: Neurology

Q.20. A female patient presents to you with a unilateral headache. It is associated with nausea, photophobia, and phonophobia. What is the drug of choice for acute management?

  1. Flunarizine
  2. Sumatriptan
  3. Propranolol
  4. Topiramate

Correct Answer: Sumatriptan

Topic: Rheumatology / Connective Tissue Disorder

Q.21. A patient presents to you with fever, night sweats, ptosis, and bilateral facial nerve palsy. Investigations showed leukocytosis and bilateral hilar lymphadenopathy. Which of the following is the most likely diagnosis?

  1. Sarcoidosis
  2. Tuberculosis
  3. Lymphoma
  4. Hypersensitive pneumonitis

Correct Answer: Sarcoidosis

Pediatrics

Topic: Neonatology

Q.22. A 1 day- old neonate has not passed urine since birth. What is the next step in management?

  1. Continue breast feeding not observed
  2. Admit to NICU’
  3. Start artificial feeding
  4. Start intravenous fluids

Correct Answer: Continue breast feeding not observed

Topic: Fluid and Electrolyte Disturbances

Q.23. A 7 – year old boy presented with abdominal pain, vomiting, oliguria, and periorbital puffiness following chemotherapy. Investigations reveal hyperuricemia, raised creatinine levels, and hyperkalemia. What is the next best step in the management of this condition ?

  1. Hydration
  2. Probenecid
  3. Allopurinol
  4. Rasburicase

Correct Answer: Hydration

PSM

Topic: Concept of Health and Disease

Q.24. The average life expectancy for a woman in Japan is 87 years. Due to recent advances in testing for cervical cancer, there is an increase in life expectancy by 15 years. The healthcare utility value is 0.8. Which of the following can be calculated from the parameters given?

  1. HALE
  2. DALY
  3. DFLE
  4. QALY

Correct Answer: QALY

Topic: Communicable and Non-communicable Diseases

Q.25. Although many animals are implicated in the spread of rabies, dogs are the most common ones. Also, it usually affects children in developing countries. Knowing this, what is the most cost-effective and logical way to reduce the incidence of rabies?

  1. Testing all dogs for rabies
  2. Reduce stray dog population and vaccinate all dogs
  3. Increase the laboratory facilities
  4. Increase capacity of healthcare workers for surveillance

Correct Answer:  Reduce stray dog population and vaccinate all dogs

Gynaecology & Obstetrics

Topic: Obstetrics

Q.26. A woman at 26 weeks of gestation presents for routine evaluation. On examination, fundal height corresponds to 24 weeks. Ultrasonography revealed decreased amniotic fluid. Which of the following conditions would have led to this presentation?

  1. Renal agenesis
  2. Tracheoesophageal fistula
  3. Cardiac abnormalities
  4. Ureteral stricture

Correct answer: Renal agenesis

Topic: Obstetrics

Q.27. A type 1 diabetic mother is on magnesium sulfate infusion post – cesarean section for preeclampsia. She develops delirium and is drowsy. She has a respiratory rate of 10/min, random blood glucose level of 240 mg / dL, oliguria, and bilaterally absent knee reflex. What is the cause of her condition?

  1. Magnesium sulfate toxicity
  2. Diabetic ketoacidosis
  3. Eclampsia
  4. Diabetes insipidus

Correct answer: Magnesium sulfate toxicity

Forensic Medicine

Topic: Court of Law 

Sub-Topic: 

Q.28. A 45-year-old female patient is told about the benefits and complications of a hysterectomy, and she agrees to the procedure. What kind of consent is this?

  1. Informed consent
  2. Implied consent
  3. Opt-out
  4. Passive consent

Correct answer: Informed consent

Topic: Asphyxial Deaths 

Q.29. A dead body is brought for evaluation. On post-mortem examination, a ligature completely encircled the neck, horizontal, and below the thyroid level was seen. There was no dribbling of saliva. What is the cause of death?

  1. Throttling
  2. Ligature strangulation
  3. Gagging
  4. Hanging

Correct answer: Ligature strangulation

Download complete

NEET PG 2021 Previous Year Questions

NEET PG 2021 was a significant national-level entrance examination conducted on September 11, 2021, for aspiring medical professionals seeking admission to postgraduate medical courses in India. Check out the NEET PG 2021 previous year questions below - 

Microbiology

Topic: Parasitology

Q.1. A patient from Uttar Pradesh presented with fever, pallor, and hepatosplenomegaly. Peripheral smear examination showed pancytopenia. Buffy coat examination showed macrophages laden with organisms with a kinetoplast. What is the vector for the likely disease?

  1. Sandfly
  2. Tse-Tse fly
  3. Triatomine bug
  4. Female anopheles mosquito

Correct Answer: Sandfly

Topic: Virology

Q.2. An unimmunized 2-year old child presented with coryza, conjunctivitis, and bluish-white spots in his buccal mucosa near the lower molar teeth. A day later, he developed a maculopapular rash on the face and neck. What is the nature of the causative virus?

ss- Single stranded

ds – Double – stranded

  1. Enveloped ss RNA
  2. Naked ss RNA
  3. Naked ds RNA
  4. Enveloped ds RNA

Correct Answer: Enveloped ss RNA

Radiology

Topic: Basics of Radiology 

Q.3. A patient following a skid resulting in a motor traffic accident was brought to the emergency room 2 hours later. On examination, he was stable with GCS 15/15. The pupil was reactive to light. Tenderness and bruising over the left lower chest wall with petechiae was seen. Severe tenderness was elicited in the left hypochondriac region and BP- 90/50 mm of Hg. What is the best investigation used in the ER?
GCS: Glasgow coma scale
FAST: Focused Assessment with Sonography for Trauma

  1. FAST
  2. X-ray
  3. Diagnostic peritoneal lavage
  4. CT scan

Correct Answer: FAST

Topic: Radiotherapy

Q.4. A child with acute lymphoblastic leukaemia undergoes prophylactic irradiation prior autologous hematopoietic stem cell transplantation. Which of the following will be the least affected?

  1. Spermatogonia
  2. Intestinal epithelial cells
  3. Neurons
  4. Bone marrow/ erythroid precursor cells

Correct Answer: Neurons 

Anatomy

Topic: Neuro Anatomy

Q.5. A patient presented with vision loss. On radiological investigation, an aneurysm causing damage to the optic chiasma was noted. Which of the following arteries is most likely to be the artery that is causing the damage?

  1. Anterior communicating artery
  2. Anterior choroidal artery
  3. Middle cerebral artery
  4. Anterior cerebral artery

Correct Answer: Anterior communicating artery

Topic: Upper Limb

Q.6. A 7-year-old boy was brought to the hospital with multiple fractures of the humerus secondary to a fall from height. On examination, there is difficulty in flexion of the elbow and supination of the forearm and associated loss of sensation over the lateral aspect of the forearm. Which is the nerve most likely to be injured?

  1. Median nerve
  2. Radial nerve
  3. Musculocutaneous nerve
  4. Ulnar Nerve

Correct Answer: Musculocutaneous nerve

Biochemistry

Topic: Lipid Chemistry

Q.7. A 5 year old boy presented with easy fatigability, irritability and inability to concentrate. Labs revealed the following findings a and b (light and oil immersion respectively) on the bone marrow aspiration. Which of the following is the most likely enzyme deficient in this condition?

  1. Hexosaminidase
  2. Glucocerebrosidase
  3. Sphingomyelinase
  4. N-acetylglucosaminidase

Correct Answer: Glucocerebrosidase

Topic: Oxidative Phosphorylation 

Q.8. You are conducting an experiment on mitochondrial respiration. You add malate/pyruvate and respiration is normal. You add succinate and respiration is normal. When you add another substance in the presence of pyruvate/succinate and malate, respiration is blocked. Which of the following substances is most likely added?

  1. Rotenone
  2. Antimycin A
  3. Oligomycin
  4. 2,4-dinitrophenol

Correct Answer: Oligomycin

Ophthalmology

Topic: Neuro Ophthalmology

Q.9. A 33-year-old woman presents with complaints of progressive loss of vision in the right halves of both eyes. Where is the lesion located in the optic pathway?

  1. Left optic tract
  2. Right visual cortex
  3. Optic chiasma
  4. Right optic nerve

Correct Answer: Left optic tract

Topic: Optics

Q.10. A 15-year-old girl is not compliant with spectacles for her myopic astigmatism. What would be the appropriate management in her case?

  1. LASIK
  2. Spherical equivalent spectacles
  3. Femto LASIK
  4. Implantable collamer lens

Correct Answer: Spherical equivalent spectacles

Surgery

Topic: Urology

Q.11. A 75-year-old man with prostate carcinoma presents to you with a PSA of 9 ng/mL with a small tumor focus. His Gleason score is 6. What will be your most likely management?
PSA- Prostate-specific antigen

  1. Radical prostatectomy
  2. External beam radiation
  3. Brachytherapy
  4. Active surveillance

Correct Answer: Active surveillance

Topic: Others

Q.12. A patient after a road traffic accident presented with pain in the abdomen. The resident examined the child and found that vitals were stable and tenderness was present in the left lumbar region. Which is the best investigation of choice?

  1. Contrast enhanced CT scan
  2. Retrograde urethrogram
  3. Wait and watch
  4. Emergency laparotomy

Correct Answer: Contrast enhanced CT scan

Dermatology

Topic: Immunobullous Disorders

Q.13. A 30-year-old presented with flaccid bullae on her skin which are easy to rupture. The biopsy revealed a suprabasal split. What is the most likely diagnosis?

  1. Pemphigus vulgaris 
  2. Pemphigus foliaceus
  3. Pemphigus vegetans
  4. Erythema multiforme

Correct Answer: Pemphigus vulgaris 

Topic: Bacterial Infections

Q.14. A patient taking multi-drug therapy (MDT) presents with worsening of existing lesions and nerve involvement. What will be your next best step of action?

  1. Stop MDT, start systemic corticosteroids
  2. Continue MDT, start systemic steroids
  3. Stop MDT thalidomide
  4. Continue MDT, start thalidomide

Correct Answer: Continue MDT, start systemic steroids

Anaesthesia

Topic: Monitoring in Anesthesia

Q.15. A 30-Year-old male patient was intubated for surgery by the final-year resident. Which of the following is the best method to confirm the position of the endotracheal tube?

  1. X-ray chest
  2. Auscultation
  3. End-tidal CO2 concentration
  4. Chest rise

Correct Answer: End-tidal CO2 concentration

Topic: Monitoring in Anesthesia

Q.16. A patient undergoing surgery is maintained on anesthesia with halothane. During the procedure, the patient suddenly developed hyperthermia and muscle rigidity. Which of the following drugs is most likely implicated in this condition?

  1. D-curare
  2. Suxamethonium
  3. Cis-atracurium
  4. Rocuronium

Correct Answer: Suxamethonium

ENT

Topic: Larynx

Q.17. Following total thyroidectomy, a patient started having difficulty in breathing, and repeated attempts to extubate were unsuccessful. The most probable cause is _________

  1. Superior laryngeal nerve injury
  2. Unilateral recurrent laryngeal nerve injury
  3. Bilateral recurrent laryngeal nerve injury
  4. Hematoma

Correct Answer: Bilateral recurrent laryngeal nerve injury

Topic: Nose and Paranasal Sinuses

Q.18. A female patient presents with nasal obstruction, nasal discharge, and loss of smell. On examination, foul-smelling discharge and yellowish – green crusts are present in the nasal cavity. She is found to have merciful anosmia. Which of the following findings can also be seen during the examination of her nose?

  1. Roomy nasal cavity
  2. Nasal polyps
  3. Inferior turbinate hypertrophy
  4. Foreign body

Correct Answer: Roomy nasal cavity

Pharmacology

Topic: CVS Pharmacology

Q.19. A patient with renal insufficiency presented with very low urine output, pedal edema, headache and BP 160/90. Which anti-hypertensive can be administered?

  1. Aliskiren
  2. Chlorthalidone
  3. Amlodipine
  4. Prazosin

Correct Answer: Amlodipine

Topic: Anticancer Drugs 

Q.20. Which of the following interleukin (IL) is inhibited by tocilizumab?

  1. IL-6
  2. IL-2
  3. IL-4
  4. IL-5

Correct Answer: IL-6

Orthopaedics

Topic: Spine + Pelvis + Lower Limb Traumatology

Q.21. A 20-year-old male patient presented with a history of lower backache and early morning stiffness for two years. He also gave a history of bilateral heel pain for 6 months. Which of the following is the most likely diagnosis?

  1. Ankylosing spondylitis
  2. Tuberculosis of the spine
  3. Disc prolapse
  4. Mechanical pain

Correct Answer: Ankylosing spondylitis

Topic: Spine + Pelvis + Lower Limb Traumatology

Q.22. A patient was brought to the hospital with complaints of pain around the left hip joint following a road traffic accident. On examination, the affected limb was flexed, adducted, and medially rotated with obvious shortening. What is the most likely diagnosis?

  1. Anterior hip dislocation
  2. Posterior hip dislocation
  3. Transcervical fracture
  4. Intertrochanteric fracture

Correct Answer: Posterior hip dislocation

Psychiatry

Topic: Psychology 

Q.23. During psychotherapy, the therapist had mixed conscious and unconscious feelings toward his patient. This is known as ___________.

  1. Countertransference
  2. Transference
  3. Dissociation
  4. Preoccupation

Correct Answer: Countertransference

Topic: Substance-Related and Addictive Disorders

Q.24. A patient stopped alcohol consumption for 3 days and presented with irritability, disorientation, paranoid delusions, agitation, visual hallucinations, and altered sensorium. What is the likely diagnosis in this case?

  1. Delirium tremens
  2. Wernicke’s encephalopathy
  3. Korsakoff psychosis
  4. Alcohol-induced psychosis

Correct Answer: Delirium tremens

Physiology

Topic: General Physiology

Q.25. A 30-year-old man weighing 70 kg had a sodium level of 120 mEq/L. Calculate the sodium deficit.

  1. 280 mEq
  2. 480 mEq
  3. 840 mEq
  4. 1400 mEq

Correct Answer: 840 mEq

Topic: General Physiology

Q.26. A body fluid sample is being studied with Na: 10 mEq/L K: 140 mEq/L, Cl: 4 mEq/L. Identify the compartment from which the piece has been obtained.

  1. Interstitial
  2. Intracellular fluid
  3. Extracellular fluid
  4. Plasma

Correct Answer: Intracellular fluid

Medicine 

Topic: Neurology

Q.27. A 37-year-old woman presents with headaches for 6 months. She has been taking analgesics regularly. The headache recently increased in severity for 3 days but was reduced in stopping the analgesic. What is the likely diagnosis?

  1. Medication overuse headache
  2. Tension headache
  3. Chronic migraine
  4. Cluster headache

Correct Answer: Medication overuse headache

Topic: Cardiology

Q.28. A 20-year-old woman presents with breathlessness and chest pain. She is a known case of mitral stenosis. Her pulse is irregularly irregular. No thrombus is seen on echocardiography. What is the best agent to prevent future thrombotic events?

  1. Dabigatran
  2. Aspirin - 150 mg
  3. Oral warfarin
  4. Aspirin + Clopidogrel

Correct Answer: Oral warfarin

Pediatrics

Topic: Paediatric Cardiology

Q.29. In foetal circulation, which vessel carries deoxygenated blood back to the placenta?

  1. Umbilical Vein
  2. Umbilical Artery
  3. Descending aorta
  4. Pulmonary Artery

Correct Answer: Umbilical artery

Topic: Paediatric Neurology

Q.30. A 10-year-old boy presented with seizures. His past history is significant for an episode of fever with rash at 1 year of age which resolved spontaneously. What is the most helpful investigation to diagnose his condition?

  1. lgG measles in CSF
  2. MRI mesial temporal lobe sclerosis
  3. lgM measles in CSF
  4. C1Q4 antibodies in the CSF

Correct Answer: lgG measles in CSF

PSM 

Topic: Environment and Health

Q.31. Which of the following thermometers is used to measure the low air velocity rather than the cooling power of the air?

  1. Kata thermometer
  2. Globe thermometer
  3. Wet thermometer
  4. Dial thermometer

Correct Answer: Kata thermometer

Topic: Environment and Health

Q.32. There is an outbreak of buboes in the community. What is the vector responsible for this condition?

  1. Xenopsylla cheopis
  2. Phlebotomus argentipes
  3. Ixodes tick
  4. Female Anopheles mosquito

Correct Answer: Xenopsylla cheopis

Gynaecology & Obstetrics

Topic: Gynecology

Q.33. A 27 year old female patient was found to have uterus didelphys. Which of the following is not likely to be a complication to this uterine anomaly?

  1. Abortion
  2. Endometriosis
  3. Preterm labour
  4. Transverse lie 

Correct Answer: Transverse lie 

Topic: Obstetrics

Q.34. A pregnant woman at 36 weeks of gestation with a prosthetic valve replacement for mitral stenosis is on warfarin therapy. Her INR is 3. What is the appropriate next step in management ?

LMWH- Low Molecular Weight Heparin

  1. Stop warfarin and start LMWH
  2. Stop warfarin and start heparin
  3. Continue warfarin
  4. Stop warfarin, start LMWH and aspirin

Correct Answer: Stop warfarin and start LMWH

Forensic Medicine

Topic: Forensic Toxicology

Q.35. A guy was playing in a garden. Suddenly, he collapsed and was rushed to the hospital. His friend who played with him in the garden informed the doctors that he ate one of the fruits in the garden post which he seemed to have developed these symptoms. He also had irritability, restlessness, dry hot skin, and was unable to pass urine and stools. Identify the poison and its appropriate antidote.

  1. Datura, Pralidoxime
  2. Datura, Physostigmine
  3. Yellow oleander, Digoxin
  4. Yellow oleander, Physostigmine

Correct Answer: Datura, Physostigmine

Topic: Forensic Thanatology 

Q.36. A man was working in a field and he suddenly collapsed on a hot summer afternoon. On examination, there were no physical signs of dehydration. His serum electrolyte level was normal. The doctor found that the body temperature was 106 degrees Fahrenheit. Which of the following symptoms is least likely to be seen in this patient?

  1. Hot skin
  2. Hypotension
  3. Sweating 
  4. Disorientation

Correct Answer: Sweating

Download complete

NEET PG 2020 Previous Year Questions

NEET PG 2020 was held on January 5, 2020, for medical graduates seeking admission to postgraduate medical courses in India. Check out the NEET PG 2020 previous year questions below- 

Microbiology

Topic: Mycology

Q.1. A 35-year-old HIV-positive patient presents with mucosal lesions in the mouth as shown in the image below. On microscopy , bugging yeasts and pseudohyphae are seen. What is the likely diagnosis?

  1. Oral candidiasis
  2. Hairy leukoplakia 
  3. Lichen planus
  4. Diphtheria

Correct Answer: Oral candidiasis

Topic: Virology

Q.2. Zika virus is transmitted by:

  1. Aedes aegypti
  2. Culex
  3. Anopheles
  4. Phlebotomus papatasii

Correct Answer: Aedes aegypti

Radiology

Topic: Head & Neck Imaging 

Q.3. A cystic lesion in the suprasellar region with calcification is seen on the MRI. What is the most likely diagnosis?

  1. Craniopharyngioma
  2. Pituitary adenoma
  3. Meningioma
  4. Oligodendroglioma

Correct Answer: Craniopharyngioma

Topic: OBGyn Imaging

Q.4. Personal monitoring of radiation is done by:

  1. TLD badge
  2. Collimators
  3. Linear accelerator
  4. Grid

Correct Answer: TLD badge

Anatomy

Topic: Back Region

Q.5. The muscle marked by the arrow in the image below is innervated by the :

  1. Dorsal scapular nerve
  2. Suprascapular nerve
  3. From the dorsal rami of C1
  4. Subscapular nerve

Correct Answer: Dorsal scapular nerve

Topic: Upper Limb

Q.6. Which of the following forms the lateral boundary of the anatomical snuff box?

  1. Extensor pollicis brevis and abductor pollicis longus
  2. Extensor pollicis longus and abductor pollicis brevis
  3. Extensor pollicis longus and extensor pollicis brevis
  4. Abductor pollicis longus and abductor pollicis brevis

Correct Answer: Extensor pollicis brevis and abductor pollicis longus

Biochemistry

Topic: Introduction

Q.7. What is the function of the proteasome?

  1. Protein folding
  2. Pot-translation modification
  3. Protein degradation
  4. Protein sorting

Correct Answer: Protein degradation

Topic: Amino Acids and Proteins Chemistry

Q.8. Replacing alanine by which amino acid, will increase the UV absorbance of protein at 280 nm wavelength?

  1. Leucine
  2. Proline
  3. Arginine
  4. Tryptophan

Correct Answer: Tryptophan

Ophthalmology

Topic: Cornea

Q.9. Which of the following layers is responsible for maintaining the hydration and transparency of the cornea?

  1. Descemet’s membrane
  2. Endothelial cells
  3. Stroma
  4. Corneal epithelium

Correct Answer: Endothelial cells

Topic: Cornea

Q.10. What is the characteristic feature of a fungal ulcer?

  1. Reverse hypopyon
  2. Dendritic ulcer on a fluorescein dye
  3. Ring abscess
  4. Satellite lesion

Correct Answer: Satellite lesion

Surgery

Topic: Endocrine Surgery

Q.11. What is the most common site of gastrinoma in MEN 1 syndrome?

  1. Jejunum
  2. Ileum
  3. Duodenum
  4. Stomach

Correct Answer: Duodenum

Topic: Cardiothoracic Vascular Surgery

Q.12. A 50-year-old male who is a chronic smoker presented to the hospital with intermittent claudication pain of both thigh and buttock region on walking about 500 m. Which of the following is a likely diagnosis?

  1. Arterial disease involving the superficial femoral artery
  2. Arterial disease with aortoiliac involvement
  3. Femoral venous insufficiency
  4. Arterial disease involving the profunda femoris artery

Correct Answer: Arterial disease with aortoiliac involvement

Dermatology

Topic: Miscellaneous Disorders

Q.13. Which of the following drugs used as nail lacquer belongs to morpholines?

  1. Amorolfine
  2. Oxiconazole
  3. Ciclopirox olamine
  4. Tioconazole

Correct Answer: Amorolfine

Topic: Skin Appendages and their Disorders

Q.14. A child was born with membranes around the body and had ectropion and eclabium. He is brought to the OPD with lesions covering his face, trunk, and extremities. Which of the following is an unlikely diagnosis?

  1. Icthyosis vulgaris
  2. Lamellar ichthyosis
  3. Bathing suit ichthyosis
  4. Harlequin ichthyosis

Correct Answer: Icthyosis vulgaris

Anaesthesia

Topic: Pre-Anaesthetic Evaluation

Q.15. Which of the following investigations provides the most accurate prognostic information with respect to predicting risk of perioperative cardiac complications?

  1. Exercise ECG testing
  2. Dobutamine stress echocardiography
  3. Myocardial perfusion scintigraphy
  4. Coronary angiography

Correct Answer: Dobutamine stress echocardiography

Topic: Neuromuscular Blockade

Q.16. A patient is given a nicotinic receptor antagonist as a muscle relaxant. Which drug is given postoperatively to recover from muscle weakness?

  1. Physostigmine
  2. Neostigmine
  3. Carbachol
  4. Succinylcholine

Correct Answer: Neostigmine

ENT

Topic: Ear

Q.17. From which of the following structures does the saccule develop?

  1. Saculus anterior
  2. Saculus posterior
  3. Pars superior
  4. Pars inferior

Correct Answer: Pars inferior

Topic: Ear

Q.18. What is the surgery done to widen the cartilaginous part of the external auditory canal called?

  1. Meatoplasty
  2. Tympanoplasty
  3. Myringoplasty
  4. Otoplasty

Correct Answer: Meatoplasty

Pharmacology

Topic: Anticancer Drugs 

Q.19. Which of the following is an inhibitor of DNA synthesis?

  1. 6-mercaptopurine
  2. Mitomycin
  3. Actinomycin
  4. Asparaginase

Correct Answer: 6-mercaptopurine

Topic: Renal Pharmacology

Q.20. A patient on lithium developed hypertension. He was started on thiazides for hypertension. After a few days, he developed coarse tremors and other symptoms suggestive of lithium toxicity. Explain the likely mechanism of this interaction.

  1. Thiazide inhibits the metabolism of lithium
  2. Thiazides act as an add on drug to lithium
  3. Thiazides increase the tubular resorption of lithium
  4. Thiazides cause water loss thereby increase lithium levels

Correct Answer: Thiazides increase the tubular resorption of lithium

Orthopaedics

Topic: Arthritis

Q.21. A patient presented with pain in the hand. The joints involved were proximal interphalangeal joint, distal interphalangeal joint and first carpometacarpal joint. The wrist and metacarpophalangeal joints were spared. What is the likely diagnosis?

  1. Osteoarthritis
  2. Rheumatoid arthritis
  3. Psoriatic arthritis
  4. Pseudogout

Correct Answer: Osteoarthritis

Topic: Sports Injuries

Q.22. A maid is playing with a child by spinning him while holding his hands. A few hours later, the child starts crying, does not use his arm, and does not let anybody touch him. What is the possible diagnosis?

  1. Pulled elbow
  2. Olecranon fracture
  3. Fracture head of radius
  4. Elbow dislocation
    Correct Answer: Pulled elbow

Psychiatry

Topic: Substance-Related and Addictive Disorders

Q.23. A 25-year-old male presented to the clinic with a complaint of the feeling of insects crawling under his skin. Which of the following drug abuse can cause the symptoms of this patient?

  1. Cannabis
  2. Cocaine
  3. Amphetamine
  4. Alcohol

Correct Answer: Cocaine

Topic: Mood Disorders

Q.24. A 60-year-old male who lost his wife 3 months back complains that his intestines have become rotten. He feels responsible for his wife’s death and should be sent to prison. He complains of feeling low all the time and has lost interest in daily activities since his wife’s death. What is the most likely diagnosis?

  1. Normal grief reaction
  2. Psychotic depression
  3. Delusional disorder
  4. Schizophrenia

Correct Answer: Psychotic depression

Physiology

Topic: Respiratory System

Q.25. A person develops headache and breathlessness on a trekking expedition following a rapid ascent to over 3000 meters above the sea level. Which of the following is not used in the treatment of this condition?

  1. IV digoxin
  2. Immediate descent
  3. Administration of oxygen
  4. Tablet acetazolamide

Correct Answer: IV digoxin

Topic: Endocrine and Reproductive System

Sub-Topic: 

Q.26. Prolactin level is highest during?

  1. 24 hours after ovulation
  2. 24 hours after delivery
  3. REM sleep
  4. After running four 1 hours

Correct Answer: 24 hours after delivery

Medicine 

Topic: Nephrology / Kidney Disease

Q.27. Which of the following is associated with pauci-immune glomerulonephritis?

  1. SLE nephritis
  2. Anti-GBM glomerulonephritis
  3. IgA nephropathy
  4. Granulomatosis with polyangiitis (GPA)

Correct Answer: Granulomatosis with polyangiitis (GPA)

Topic: Emergency Medicine

Q.28. At a high altitude of 3000m, a person complains of breathlessness. All of the following can be used for management of this person except

  1. Intravenous digoxin
  2. Oxygen supplementation
  3. Immediate descent
  4. Acetazolamide

Correct Answer: Intravenous digoxin

Pediatrics

Topic: Musculoskeletal Disorders in Children

Q.29. A 4-yr old male child presents with muscle weakness. His mother says that her child has difficulty climbing stairs and getting up from the floor. On muscle biopsy, small muscle fibrils and absence of dystrophin was found. What is the diagnosis out of given options?

  1. Becker's muscle dystrophy
  2. Duchenne muscular dystrophy
  3. Myotonic dystrophy
  4. Limb-girdle muscular dystrophy

Correct Answer: Duchenne muscular dystrophy

Topic: Paediatric Neurology

Q.30. Two girls in the same class are diagnosed with meningococcal meningitis. Their 12-year old friend from the same school is afraid of contracting the disease. What advice should be given to the exposed students?

  1. Two doses of polysaccharide vaccine
  2. Antibiotic prophylaxis
  3. Two doses of conjugate vaccine
  4. Single dose of meningococcal vaccine

Correct Answer: Antibiotic prophylaxis

PSM 

Topic: Health Planning and Management

Q.31. The monetary benefit is measured in which of the following analyses?

  1. Cost-effective analysis
  2. Cost-benefit analysis
  3. Network analysis
  4. Input-Output analysis

Correct Answer: Cost-benefit analysis

Topic: Allied Health Disciplines

Q.32. An employee diagnosed with TB gets extended sickness benefit for?

  1. 1 year
  2. 2 years
  3. 3 years
  4. 4 years

Correct Answer: 2 years

Gynaecology & Obstetrics

Topic: Obstetrics

Q.33. 24 yr old women who had home delivery 2 weeks back, now present with complete perineal tear. What is the next line of management?

  1. Repair immediately
  2. Repair after 3 weeks
  3. Repair after 6 months
  4. Repair after 3 months

Correct Answer: Repair after 3 months

Topic: Gynecology

Q.34. The remnants of Wolffian ducts in females are located in

  1. Pouch of Douglas
  2. Leaves of broad ligament
  3. Uterovesical pouch
  4. Iliac fossa

Correct Answer: Leaves of broad ligament

Forensic Medicine

Topic: Forensic Identification

Q.35. A deceased male was brought to the morgue for a post-mortem examination. His relatives reported that he had a tattoo over his skin, which was not found on examination. Which of the following structures would you examine?

  1. Lymph nodes
  2. Liver
  3. Arteries
  4. Skin

Correct Answer: Lymph nodes

Topic: Forensic Toxicology

Q.36. The most common drug which causes physical dependence is?

  1. Ketamine
  2. Heroin
  3. LSD
  4. Phencyclidine

Correct Answer: Heroin

Download complete

NEET PG 2019 Previous Year Questions

NEET PG 2019 was conducted on 6th January, 2019 with huge participation. Check out the previous year questions below - 

Microbiology

Topic: Systemic Bacteriology

Q.1. BCYE medium is used to culture:

  1. Leptospira
  2. Bacillus cereus
  3. Campylobacter
  4. Legionella

Correct Answer: Legionella

Topic: Systemic Bacteriology

Q.2. A 16-year-old boy is admitted with fever, icterus, conjunctival suffusion, and hematuria for 15 days. Which serological test should be done for diagnosis?

  1. Widal test
  2. Microscopic agglutination test
  3. Weil felix reaction
  4. Paul Bunnell test

Correct Answer: Microscopic agglutination test

Radiology

Topic: Basics of Radiology 

Q.3. What is the best imaging study for the earliest diagnosis of cerebral infarct?

  1. Flair MRI
  2. Non-contrast CT
  3. CECT
  4. Diffusion weighted MRI

Correct Answer: Diffusion weighted MRI

Topic: Basics of Radiology 

Q.4. Which of the following is a water soluble contrast?

  1. Iohexol
  2. Barium 
  3. Borium 
  4. Calcium

Correct Answer: Iohexol

Anatomy

Topic: Back Region

Q.5. The movements at the _______ joint permit a person to look to the right and left.

  1. Atlanto-occipital
  2. Atlanto -axial
  3. C2-C3
  4. C3-C4

Correct Answer: Atlanto -axial

Topic: Abdomen

Q.6. What is the basis for the formation of omphaloceles?

  1. Excessive length of the intestine
  2. Physiological hernia failing to go back
  3. Herniation of the liver
  4. Herniation of umbilicus

Correct Answer: Physiological hernia failing to go back

Biochemistry

Topic: Lipid Metabolism

Q.7. An infant present with hypotonia and seizures. It was confirmed to be cerebrohepatorenal syndrome. Which of the following is accumulated in the brain in cerebrohepatorenal syndrome?

  1. Glucose
  2. Long chain fatty acid
  3. Lactic Acid
  4. Triglycerides

Correct Answer: Long Chain Fatty Acid

Topic: Genetics

Q.8. Which of the following is autosomal dominant?

  1. Cystic fibrosis
  2. Duchene muscular dystrophy
  3. Sickle cell anaemia
  4. Achondroplasia

Correct Answer: Achondroplasia

Ophthalmology

Topic: Optics

Q.9. Which of the following would be prescribed for simple myopic astigmatism?

  1. +1.00 DS
  2. -1.00 DC × 180 Degree
  3. -1.00 DS
  4. -1.00DS – 1.00 DC × 180 Degree

Correct Answer: -1.00 DC × 180 Degree

Topic: Retina

Q.10. Not true about retinitis pigmentosa is:

  1. Retinal pigmentation
  2. Pale waxy disc
  3. Narrowing of vessels
  4. ERG - Normal

Correct Answer: ERG- Normal

Surgery

Topic: Urology

Q.11. Which of the following drugs cause carcinoma bladder?

  1. Cyclophosphamide
  2. Cisplatin
  3. Taxane
  4. Tamoxifen

Correct Answer: Cyclophosphamide

Topic: Gastrointestinal Surgery

Q.12. Dohlman's procedure is for:

  1. Meckel's diverticulum
  2. Zenker’s diverticulum
  3. Bochadlek hernia
  4. Menetrier's disease

Correct Answer: Zenker’s diverticulum

Dermatology

Topic: Eczema

Q.13. Bindi Leukoderma is caused by which chemical?

  1. A Mono-benzyl ether of Hydroquinone (MBH)
  2. Crocein Scarlet MOO and Solvent Yellow 3
  3. p-phenylenediamine (PPD)
  4. Para Tertiary butylphenol (PTBP)

Correct Answer: Para Tertiary butylphenol (PTBP)

Topic: Sexually Transmitted Infections

Q.14. A young female presented with vaginal itching and green frothy genital discharge. Strawberry vagina is seen on examination. What will be the drug of choice?

  1. Doxycycline
  2. Oral fluconazole
  3. Metronidazole
  4. Azithromycin

Correct Answer: Metronidazole

Anaesthesia

Topic: Neuromuscular Blockade

Q.15. What is the mechanism of action of the curare group of muscle relaxants?

  1. Persistently depolarizing at neuromuscular junction
  2. Competitively blocking the binding of ACh to its receptors
  3. Repetitive stimulation of ACh receptors on muscle endplate
  4. Inhibiting the calcium channels on presynaptic membrane

Correct Answer: Competitively blocking the binding of ACh to its receptors

Topic: Inhalational Anaesthetic Agents

Q.16. Which of the following is not used for induction of anaesthesia in paediatric patients?

  1. Halothane
  2. Sevoflurane
  3. Desflurane
  4. Nitrous oxide

Correct Answer: Desflurane

ENT

Topic: Ear

Q.17. Which of the following is not a feature of tubercular otitis media?

  1. Ear ache
  2. Multiple perforations
  3. Pale granulation
  4. Foul smelling ear discharge

Correct Answer: Ear ache

Topic: Nose and Paranasal Sinuses

Q.18. Pott’s puffy tumor is

  1. Subperiosteal abscess of frontal bone
  2. Subperiosteal abscess of ethmoid bone
  3. Mucocele of frontal bone
  4. Mucocele of ethmoid bone

Correct Answer: Subperiosteal abscess of frontal bone

Pharmacology

Topic: Drugs Affecting Blood and Blood formation

Q.19. Which of the following is an oral factor Xa inhibitor?

  1. Bivalirudin
  2. Dabigatran
  3. Rivaroxaban
  4. Enoxaparin

Correct Answer: Rivaroxaban

Topic: Antimicrobials : Antibacterial Drugs

Q.20. What is the mechanism of action of oseltamivir and zanamivir?

  1. DNA polymerase inhibition
  2. Protein synthesis inhibition
  3. Nucleotide analogue
  4. Neuraminidase inhibition

Correct Answer: Neuraminidase inhibition

Orthopaedics

Topic: General + Upper Limb Traumatology

Q.21. Which of the following fractures is most prone to non-union?

  1. Proximal scaphoid
  2. Intertrochanteric
  3. Distal radius
  4. Talar neck

Correct Answer: Proximal scaphoid

Topic: Miscellaneous

Q.22. Degloving injury refers to_____?

  1. Skin and subcutaneous fat are stripped from the underlying fascia
  2. Skin, subcutaneous fat and fascia are stripped from tendons
  3. Skin, subcutaneous fat, fascia and tendons are stripped from bone
  4. Only skin is stripped off

Correct Answer: Skin and subcutaneous fat are stripped from the underlying fascia

Psychiatry

Topic: Substance-Related and Addictive Disorders

Q.23. Which of the following drugs is used as an antismoking agent?

  1. Busulfan
  2. Acamprosate
  3. Varenicline
  4. Gabapentin

Correct Answer: Varenicline

Topic: Schizophrenia Spectrum and Other Psychotic Disorders

Q.24. Which of the following is not a risk factor for delusional disorder?

  1. Recent immigration
  2. Social isolation 
  3. Family history
  4. Young age

Correct Answer: Young age

Physiology

Topic: Excretory System

Q.25. According to myogenic hypothesis of renal autoregulation the afferent arterioles contract in response to stretch – induced by ____________

  1. NO
  2. Noradrenaline
  3. Opening of calcium channels
  4. Adenosine

Correct Answer: Opening of calcium channels

Topic: The Nervous System

Q.26. Which of the following is true for decorticate rigidity?

  1. It is produced by the removal of cerebral cortex and basal ganglia
  2. Flexion of lower limbs and extension of upper limbs occurs
  3. Rigidity is pronounced
  4. It is characterised by flexion of upper limbs and extension of lower limbs

Correct Answer: It is characterised by flexion of upper limbs and extension of lower limbs

Medicine 

Topic: Pulmonology

Q.27. Nasal polyps are commonly associated with: -

  1. Intrinsic Asthma
  2. Brittle Asthma
  3. Extrinsic Asthma
  4. Exercise induced asthma

Correct Answer: Intrinsic Asthma

Topic: Haematology

Q.28. A 47 year old man with a diagnosis of acute myeloid leukaemia with a blood type O negative blood group presents to the transplant clinic to discuss proceeding with an allogeneic stem cell transplant. Which of the following would be an optimal donor?

  1. His identical twin brother
  2. Umbilical cord transplant
  3. His HHLA identical 50-year-old brother who is otherwise healthy and is blood type O+
  4. An HLA identical matched unrelated donor who is blood type

Correct Answer: His HHLA identical 50-year-old brother who is otherwise healthy and is blood type O+

Pediatrics

Topic: Paediatric Gastroenterology

Q.29. Unconjugated hyperbilirubinemia, which did not subside even after 3 weeks of birth, was observed in a neonate. On investigating, liver enzymes, PT/INR and albumin levels were normal. No hemolysis was seen on a peripheral blood smear. A drop in bilirubin level was observed within a week after treatment with phenobarbital. What is the most likely diagnosis?

  1. Rotor syndrome
  2. Crigler Najjar type 2
  3. Dubin Johnson syndrome
  4. Crigler Najjar type 1

Correct Answer: Crigler Najjar type 2

Topic: Pediatric Respiratory Disorders

Q.30. Which of the following is not seen in a child with cystic fibrosis?

  1. Sweat chloride test chloride conc of 70mEq/L
  2. Increase immunoreactive trypsinogen level
  3. Hyperkalemia
  4. Contraction alkalosis

Correct Answer: Hyperkalemia

PSM 

Topic: Concept of Health and Disease

Q.31. Which of the following parameters would you use to check the efficiency of the surveillance system for malaria under the National Vector Borne Disease Control Programme?

  1. Annual Parasite Index
  2. Annual Blood Examination Rate
  3. Slide positivity rate
  4. Slide falciparum rate

Correct Answer: Annual Blood Examination Rate

Topic: Screening of Diseases

Q.32. Which of the following is the best level of prevention of breast cancer?

  1. Specific protection
  2. Early diagnosis and treatment
  3. Disability limitation
  4. Rehabilitation

Correct Answer: Early diagnosis and treatment

Gynaecology & Obstetrics

Topic: Obstetrics

Sub-Topic: 

Q.33. In which trimester does acute fatty liver manifest most commonly during pregnancy?

  1. First trimester
  2. Second trimester
  3. Third trimester
  4. Both A & B

Correct Answer: Third trimester

Topic: Obstetrics

Q.34. You discover that there is evidence of cardiac malformation in a foetus during a routine USG scan at 16 weeks. What is the minimum level of fasting blood sugar at which you can suspect overt diabetes?

  1. 106 mg/dL
  2. 126 mg/dL
  3. 116 mg/dL
  4. 130 mg/dL

Correct Answer: 126 mg/dL

Forensic Medicine

Topic: IPC Sections 

Q.35. Which Section of the IPC deals with the punishment of perjury?

  1. 191
  2. 192
  3. 193
  4. 197

Correct Answer: 193

Topic: Sexual Jurisprudence

Q.36. Frotteurism is _____________

  1. Obtaining sexual pleasure by wearing clothes of opposite sex
  2. Desire to seek surgery to become member of opposite sex
  3. Sexual gratification by rubbing his private parts against another person
  4. Exposure of one’s genitals to an unsuspecting stranger

Correct Answer: Sexual gratification by rubbing his private parts against another persont

Download complete
Rapid Revision 5.0

Anatomy

Topic: Head and Neck
Q.2. A student had his jaw locked while yawning. Which of the following muscles is attached to the articular disc of the temporomandibular joint?
1. Lateral pterygoid
2. Temporalis
3. Medial pterygoid
4. Masseter

Correct Answer: Lateral pterygoid

Topic: Abdomen

Q.3. A patient comes with abdominal pain, jaundice, and portal hypertension. Anastomosis between which of the following veins is seen?
1. Left colic vein and middle colic veins
2. Esophageal veins and left gastric veins
3. Superior rectal and phrenic veins
4. Sigmoid and superior rectal veins

Correct Answer: Esophageal veins and left gastric veins

Biochemistry

Topic: Vitamins

Q.4. A patient presented with diarrhea, dermatitis, and dementia. Which of the following vitamin deficiencies would you suspect in this patient?
1. Vitamin B3 deficiency
2. Vitamin B1 deficiency
3. Vitamin B6 deficiency
4. Vitamin B12 deficiency

Correct Answer: Vitamin B3 deficiency

Topic: Carbohydrate Metabolism

Q.5. A young boy presents to the OPD with hypoglycemia and muscle cramps, on exertion or while playing. Then he becomes normal after resting for a while. These episodes are recurrent after a period of activity. He has decreased serum lactate and glucose levels. Which of the following diseases is he most likely to be suffering from?
1. McArdle disease
2. Hers disease
3. Cori's disease
4. Andersen disease

Correct Answer: McArdle disease

Parasitology

Topic: Parasitology

Q.6. Which of the following statements is true about Trichomonas vaginalis?
1. It cannot be cultured
2. Twitching motility is seen on wet saline mount
3. Cysts are seen on wet saline mount of vaginal secretions
4. It is not a sexually transmitted infection

Correct Answer: Twitching motility is seen on wet saline mount

Radiology

Topic: Head & Neck Imaging

Q.7. A 2-month-old infant is brought to the OPD with a parietal swelling present since birth. The X-ray image is given below. What could be the probable diagnosis?
1. Subgaleal hematoma
2. Cephalhematoma
3. Caput succedaneum
4. Enecephalocele

Correct Answer: Cephalhematoma

Topic: Genitourinary Radiology

Q.8. A patient presented with abdominal pain and sterile pyuria, and the x-ray showed the following features. What is the likely diagnosis?
1. Putty kidney
2. Nephrocalcinosis
3. Staghorn calculus
4. Psoas calcification

Correct Answer: Putty kidney

Microbiology

Topic: Virology

Q.9. A patient hailing from Delhi presents with fever, arthralgia, and extensive petechial rash for 3 days. Lab investigations revealed a hemoglobin of 9 g/dL, a white blood cell count of 9000 cells/mm3, a platelet count of 20000 cells/mm3, and a prolonged bleeding time. The clotting time was normal. What is the most likely diagnosis?
1. Dengue
2. Malaria
3. Scrub typhus
4. Typhoid

Correct Answer: Dengue

Ophthalmology

Topic: Neuro Ophthalmology

Q.10. A female presents with loss of vision in the right halves of both eyes. Where is the lesion located in the optic pathway?
1. Left optic tract
2. Optic radiation
3. Optic chiasma
4. Right optic tract

Correct Answer: Left optic tract

Topic: Cornea

Q.11. The eye examination of a 20-year-old male shows a golden ring on the iris. What is the next investigation to diagnose the condition?
1. Alpha-1-antitrypsin
2. Alpha-fetoprotein
3. Serum Iron
4. Serum ceruloplasmin

Correct Answer: Serum ceruloplasmin

Surgery

Topic: Urology

Q.12. A 55-year-old male presented with verrucous carcinoma around the glans of the penis. Examination reveals that the inguinal lymph nodes are not enlarged. What is the appropriate management for this patient?
1. Total penectomy
2. CO2 laser excision
3. Topical 5-fluorouracil
4. Partial penectomy

Correct Answer: Partial penectomy

Topic: Gastrointestinal Surgery

Q.13. Which of the following is the most common complication following ligation of the first vessel during abdominoperineal resection for rectal carcinoma?
1. Parasympathetic-bladder dysfunction and retrograde ejaculation.
2. Sympathetic-bladder dysfunction and impotence.
3. Sympathetic-retrograde ejaculation and bladder dysfunction.
4. Sympathetic-impotence and loss of cutaneous sensation in the perineal region

Correct Answer: Sympathetic-retrograde ejaculation and bladder dysfunction.

Dermatology

Topic: Introduction to Dermatology

Q.14. A female patient presented with acne that is not resolving on oral isotretinoin and antibiotics therapy. Which of the following is the next best investigation?
1. Look for dietary triggers.
2. Evaluate for hyperandrogenism
3. Check for antibiotic resistance
4. Look for drug triggers

Correct Answer: Evaluate for hyperandrogenism

Topic: Sexually Transmitted Infections

Q.15. A young woman complains of a painless ulcer in the genital area. It is associated with non-tender inguinal lymphadenopathy. What is the most likely diagnosis?
1. Chancroid
2. Syphilis
3. Herpes genitalis
4. Granuloma inguinale

Correct Answer: Syphilis

Anaesthesia

Topic: Pre-Anaesthetic Evaluation

Q.16. Which of the following is the drug of choice for preoperative antibiotic prophylaxis in a patient undergoing cardiac surgery?
1. Penicillin G
2. Erythromycin
3. Azithromycin
4. Cefazolin

Correct Answer: Cefazolin

ENT

Topic: Ear

Q.17. A female patient with hearing loss is examined and is found to be Rinne negative at 256 Hz and 512 Hz, while Rinne positive at 1024 Hz. What is the expected air conduction and bone conduction gap?
1. 30-45 dB
2. 15-30 dB
3. 45-60 dB
4. >60 dB

Correct Answer: 30-45 dB

Topic: Pharynx

Q.18. A 10-year-old child presents with throat pain, fever, and ear pain. He is diagnosed with recurrent tonsillitis. Which nerve is responsible for the ear pain in this patient?
1. Tympanic branch of the glossopharyngeal nerve
2. Greater auricular nerve
3. Auriculotemporal nerve
4. Auricular branch of the vagus nerve

Correct Answer: Tympanic branch of the glossopharyngeal nerve

Pharmacology

Topic: Antimicrobials : Antibacterial Drugs

Q.19. A bronchial asthma patient on inhalational steroids presented with white patchy lesions on the tongue and buccal mucosa. What is the drug that can be used to treat this condition?
1. Clotrimazole
2. Griseofulvin
3. Terbinafine
4. Flucytosine

Correct Answer: Clotrimazole

Topic: General Pharmacology

Q.20. A pregnant woman with a history of bronchial asthma is in the third stage of labour. Which drugs should be avoided in managing postpartum haemorrhage in this mother?
1. Carboprost
2. Oxytocin
3. Dinoprostone
4. Methyl ergometrine

Correct Answer: Carboprost

Orthopaedics

Topic: Peripheral Nerve Injuries

Q.21. A patient at the orthopaedics OPD complains of troubled sleep at night due to numbness and tingling sensation involving his lateral 3 digits. His symptoms are relieved as he lays his arms hanging from the bed. Which of the following options correctly describes his condition and the test used to assess it?
1. Guyon's canal syndrome, Froment's test
2. Carpal tunnel syndrome, Froment's test
3. Guyon's canal syndrome, Durkan's test
4. Carpal tunnel syndrome, Durkan's test

Correct Answer: Carpal tunnel syndrome, Durkan's test

Topic: Metabolic Disorders

Q.22. A child is brought to the orthopaedics OPD with a deformity in the lower limb and hyperpigmented skin lesions. The x-ray of her thigh is shown below. What is the most likely diagnosis?
1. Non-ossifying fibroma
2. Fibrous dysplasia
3. Paget's disease
4. Osteogenesis imperfecta

Correct Answer: Fibrous dysplasia

Psychiatry

Topic: Schizophrenia Spectrum and Other Psychotic Disorders

Q.23. A schizophrenic patient was prescribed drug A after he did not respond to haloperidol and thioridazine. He now presents with excessive salivation, an increase in blood glucose, and hyperlipidemia. What is drug A?
1. Ziprasidone
2. Risperidone
3. Clozapine
4. Aripiprazole

Correct Answer: Clozapine

Topic: Mood Disorders

Q.24. A woman, who is 4 days postpartum, presented with tearfulness, mood swings, and occasional insomnia. What is the likely diagnosis?
1. Postpartum depression
2. Postpartum blues
3. Postpartum psychosis
4. Postpartum anxiety

Correct Answer: Postpartum blues

Physiology

Topic: General Physiology

Q.25. A man was brought to the emergency after suddenly becoming unconscious while working in the field. On examination, his temperature was 105 degrees Fahrenheit, and his skin turgor was decreased. Which of the following would not be seen in the patient?
1. Tachypnea
2. Hypotension
3. Sweating
4. Red hot skin

Correct Answer: Sweating

Topic: Respiratory System

Q.25. A preterm baby who was delivered at 28 weeks developed respiratory distress syndrome. Which of the following is true about surface tension and compliance in this baby?
1. Surface tension - decreased; Compliance - increased
2. Surface tension - increased; Compliance - decreased
3. Both surface tension and compliance decreased
4. Both surface tension and compliance increased

Correct Answer: Surface tension - increased; Compliance - decreased

Medicine

Topic: Nephrology / Kidney Disease

Q.26. A patient with hyperkalemia and elevated urea levels underwent dialysis. Towards the end of the session, she became drowsy and had a sudden seizure episode. On examination, the patient was hypotensive. What is the treatment for this condition?
1. Bumetanide
2. Ethacrynic acid
3. Nesiritide
4. IV Mannitol

Correct Answer: IV Mannitol

Topic: Endocrinology

Q.27. A female patient presents to the emergency department with severe restlessness, palpitations, and tremors. She is a known case of bronchial asthma. On examination, the neck looks swollen. Blood pressure is elevated, and tachycardia is noted. ECG shows atrial fibrillation. Which of the following drugs is used for immediate management in this patient?
1. Diltiazem
2. Propranolol
3. Esmolol
4. Propylthiouracil

Correct Answer: Diltiazem

PSM

Topic: Nutrition and Health

Q.28. In a village, it is observed that several farmers have crossed gait and use a stick for support to stand up and walk. Due to poor yield from farms, they consume meals containing rice and pulses only. Supplementing their diet with which of the following vitamins could have prevented this?
1. Vitamin A
2. Vitamin D
3. Vitamin C
4. Vitamin B

Correct Answer: Vitamin C

Topic: Epidemiology

Q.29. Research is being conducted to find the association between aniline dye exposure and bladder cancer in workers who have worked in the industry for >20 years. Two groups were formed: one directly involved with dye handling and the other group consisting of office clerks not directly exposed to the dye. Years of occupation were noted from records. What type of study is being performed?
1. Retrospective cohort study
2. Prospective cohort study
3. Case-control study
4. Intervention and response

Correct Answer: Retrospective cohort study

Gynaecology & Obstetrics

Topic: Obstetrics

Q.30. A woman presents to you at 36 weeks of gestation with complaints of breathlessness and excessive abdominal distension. Fetal movements are normal. On examination, fetal parts are not easily felt and the fetal heartbeat is heard but it is muffled. Her symphysis fundal height is 41 cm. Her abdomen is tense but not tender. What is the most likely diagnosis?
1. Abruptio placenta
2. Hydrocephalus of fetus
3. Polyhydramnios
4. Fetal-maternal ascites

Correct Answer: Polyhydramnios

Topic: Obstetrics

Q.31. A primigravida presents to you with anemia early in her pregnancy. She is 7 weeks pregnant as seen on ultrasound. Her hemoglobin level is 9 g/dL. When should the iron supplements be started for her?
1. 10 to 12 weeks
2. 8 to 10 weeks
3. After 14 weeks
4. After 20 weeks

Correct Answer: 8 to 10 weeks

Forensic Medicine

Topic: Forensic Toxicology

Q.32. A child before playing consumed fruit from the garden. After some time he developed a high fever, confusion, photophobia, and unable to urinate. What are the likely causative agent and the appropriate antidote used in this case?
1. Datura, Pralidoxime
2. Datura, Physostigmine
3. Yellow oleander, Pralidoxime
4. Yellow oleander, Physostigmine

Correct Answer: Datura, Physostigmine

Topic: Forensic Thanatology

Q.33. The method of autopsy carried out en masse to remove from tongue to prostate is.
1. Virchow technique
2. Rokitansky technique
3. Ghon technique
4. Letulle technique

Correct Answer: Letulle technique

Pediatrics

Topic: Fluid and Electrolyte Disturbances

Q.34. A 10-year-old child weighing 30 kg presents with a history of loose stools for 2 days. On examination, there is severe dehydration. Laboratory investigations are as follows. What is the initial management as per ISPAD guidelines?

RBS550 mg/dL
pH7.01
Na+158mEq/L
Urine glucose3+
  1. Manage ABC, NS 20 mL/kg and start insulin after 1 hour
  2. Manage ABC, NS 20 mL/kg along with insulin 0.1 IU/kg/hr
  3. Manage ABC, NS 10 mL/kg along with insulin 0.1 IU/kg/hr
  4. Manage ABC, NS 10 mL/kg and start insulin after 1 hour

Correct Answer: Manage ABC, NS 20 mL/kg and start insulin after 1 hour

Topic: Growth

Q.35. Which of the following is the best sign to indicate adequate growth in an infant with a birth weight of 2.8 kg?

  1. Increase in length of 25 centimetres in the first year
  2. Weight gain of 300 grams per month till 1 year
  3. Anterior fontanelle closure by 6 months of age
  4. Weight under the 75th percentile and height under the 25th percentile

Correct Answer: Increase in length of 25 centimetres in the first year.

Download complete

NEET PG 2022 Previous Year Questions

NEET PG 2022 was conducted on May 21, 2022, as a national-level entrance examination for postgraduate medical courses in India. It saw a substantial number of candidates vying for admission to various medical institutes across the country. Check out the NEET PG 2022 previous year questions below - 

Microbiology

Topic: Parasitology

Q.1. Microfilariae with sheathed tail and two nuclei at the tail is suggestive of?
1. Wuchereria bancrofti
2. Brugia malayi
3. Loa loa
4. Onchocerca volvulus

Correct Answer: Brugia malayi

Microbiology

Topic: Systemic Bacteriology

Q.2. A militant presents with rashes all over his body sparing the palms and soles. On examination, he was febrile and lice were noted. Which of the following is responsible for his condition?
1. Rickettsia typhi
2. Rickettsia prowazekii
3. Rickettsia akari
4. Rickettsia conorii

Correct answer: Rickettsia prowazekii

Anatomy

Topic: Lower Limb

Q.3. A patient underwent surgery for the varicose vein. He now complains of sensory loss over the medial aspect of the leg and foot. Which of the following nerves is most likely to be injured?
1. Sural nerve
2. Superficial peroneal nerve
3. Deep peroneal nerve
4. Saphenous nerve

Correct Answer: Saphenous nerve

Biochemistry

Topic: Carbohydrate Metabolism

Q.4. A patient from a Mediterranean country visits Africa, where he develops malaria. He is treated with primaquine and later develops hemolytic anemia. Deficiency of an enzyme involved in which of the following pathways could be the cause?
1. Glycolysis
2. Gluconeogenesis
3. Hexose monophosphate (HMP) pathway
4. Luebering-Rapoport pathway

Correct Answer: Hexose monophosphate (HMP) pathway

Biochemistry

Topic: Genetics

Q.5. Which of the following disorders follows autosomal recessive inheritance pattern?
1. Huntington’s disease
2. Treacher Collins syndrome
3. Cystic fibrosis
4. Achondroplasia

Correct Answer: Cystic fibrosis

Ophthalmology

Topic: Uveitis

Q.6. A patient presents with a history of penetrating injury to the eye. A diagnosis of sympathetic ophthalmitis was confirmed. Which of the following will be seen?
1. Acute anterior uveitis
2. Pars planitis
3. Panuveitis
4. Chronic anterior uveitis

Correct Answer: Panuveitis

Topic: Glaucoma

Q.7. A diabetic patient presents to you with visual acuity of 6/9 in one eye. Further investigations revealed preretinal hemorrhages with neovascularization at the optic disc. What is the next step in management?
1. Focal laser photocoagulation
2. Pan-retinal photocoagulation
3. Grid laser photocoagulation
4. Scleral buckling

Correct Answer: Pan-retinal photocoagulation

Surgery

Topic: Endocrine Surgery

Q.8. A 45-year-old female patient underwent a thyroidectomy. Three days after the surgery, she developed perioral numbness. Which of the following investigations need to be done for her?
1. Free T3, T4
2. T3, T4, thyroid–stimulating hormone
3. Radioiodine scan
4. Calcium, phosphate, and parathormone levels

Correct Answer: Calcium, phosphate, and parathormone levels

Topic: Cardiothoracic Vascular Surgery

Q.9. Which of the following is most likely to be seen due to the rupture of a saccular aneurysm?
1. Subdural haemorrhage
2. Subarachnoid haemorrhage
3. Intracerebral haemorrhage
4. Hydrocephalus

Correct Answer: Subarachnoid haemorrhage

Dermatology

Topic: Sexually Transmitted Infections

Q.10. A patient presents to you with multiple anogenital warts. The biopsy of these lesions showed squamous atypia. Which of the following human papillomavirus types are considered high-risk?
1. HPV 2
2. HPV 18
3. HPV 6
4. HPV 11

Correct Answer: HPV 18

Topic: Cutaneous Infections

Q.11. A farmer presents you with a cauliflower-shaped mass on foot, which developed after a minor injury. Microscopy shows copper penny bodies. What is the most likely diagnosis?
1. Chromoblastomycosis
2. Blastomycosis
3. Sporotrichosis
4. Phaeohyphomycosis

Correct Answer: Chromoblastomycosis

Pharmacology

Topic: Respiratory Pharmacology

Q.12. A male patient with chronic obstructive pulmonary disease (COPD) was prescribed theophylline. He noticed that his urine output had increased the following day. This action of the drug is mediated through which of the following receptors?
1. Interleukin – 10
2. Histone deacetylase
3. Adenosine A1
4. Beta 2 adrenergic receptors

Correct Answer: Adenosine A1

Topic: Endocrine Pharmacology

Q.13. Which of the following drugs is not likely to cause Pulmonary fibrosis?
1. Metformin
2. Methotrexate
3. Bleomycin
4. Nitrofurantoin

Correct Answer: Metformin

Orthopaedics

Topic: Metabolic Disorders

Q.14. An intrauterine scan at the 13th week of pregnancy showed a fetus with multiple long bone fractures. What is commonly associated with this finding?
1. Achondroplasia
2. Osteogenesis imperfecta
3. Cretinism
4. Marfan syndrome

Correct Answer: Osteogenesis imperfecta

Topic: Spine + Pelvis + Lower Limb Traumatology

Q.15. A male patient presented with a bone fracture following a road traffic accident. After 2 days he developed dyspnea, petechiae involving the whole body, and a fall in oxygen saturation. What is the likely diagnosis?
1. Fat embolism
2. Air embolism
3. Venous thromboembolism
4. Pulmonary hypertension

Correct Answer: Fat embolism

Psychiatry

Topic: Eating Disorders

Q.16. A 16-year-old girl has intense cravings for food. She eats large amounts of food, which is followed by self-induced vomiting. What is the probable diagnosis?

  1. Anorexia nervosa
  2. Bulimia nervosa
  3. Atypical depression
  4. Binge eating disorder

Correct Answer: Bulimia nervosa

Topic: Mood Disorders

Q.17. A woman, who is 4 days postpartum, presented with tearfulness, mood swings, and occasional insomnia. What is the likely diagnosis?

  1. Postpartum depression
  2. Postpartum blues
  3. Postpartum psychosis
  4. Postpartum anxiety

Correct Answer: Postpartum blues

Physiology

Topic: Nerve Muscle Physiology

Q.18. A person after sleeping overnight with the arm under his head now experiences paresis but no numbness in the morning. Which of the following is the best explanation for it?

  1. C fibres are more sensitive to pressure than A fibres
  2. A fibres are more sensitive to hypoxia than B fibres
  3. A fibres are more susceptible to pressure changes than C fibres
  4. A fibres are more susceptible to hypoxia than C fibres

Correct Answer: A fibres are more susceptible to pressure changes than C fibres

Topic: The Nervous System

Q.19. A 65-year-old suffered from a stroke 2 days ago. He now presents with involuntary, violent, and flinging movements of the limbs on one side. What is the likely site of lesion in this patient?

  1. Subthalamic nuclei
  2. Globus pallidus
  3. Putamen
  4. Caudate nucleus

Correct Answer: Subthalamic nuclei

Medicine 

Topic: Neurology

Q.20. A female patient presents to you with a unilateral headache. It is associated with nausea, photophobia, and phonophobia. What is the drug of choice for acute management?

  1. Flunarizine
  2. Sumatriptan
  3. Propranolol
  4. Topiramate

Correct Answer: Sumatriptan

Topic: Rheumatology / Connective Tissue Disorder

Q.21. A patient presents to you with fever, night sweats, ptosis, and bilateral facial nerve palsy. Investigations showed leukocytosis and bilateral hilar lymphadenopathy. Which of the following is the most likely diagnosis?

  1. Sarcoidosis
  2. Tuberculosis
  3. Lymphoma
  4. Hypersensitive pneumonitis

Correct Answer: Sarcoidosis

Pediatrics

Topic: Neonatology

Q.22. A 1 day- old neonate has not passed urine since birth. What is the next step in management?

  1. Continue breast feeding not observed
  2. Admit to NICU’
  3. Start artificial feeding
  4. Start intravenous fluids

Correct Answer: Continue breast feeding not observed

Topic: Fluid and Electrolyte Disturbances

Q.23. A 7 – year old boy presented with abdominal pain, vomiting, oliguria, and periorbital puffiness following chemotherapy. Investigations reveal hyperuricemia, raised creatinine levels, and hyperkalemia. What is the next best step in the management of this condition ?

  1. Hydration
  2. Probenecid
  3. Allopurinol
  4. Rasburicase

Correct Answer: Hydration

PSM

Topic: Concept of Health and Disease

Q.24. The average life expectancy for a woman in Japan is 87 years. Due to recent advances in testing for cervical cancer, there is an increase in life expectancy by 15 years. The healthcare utility value is 0.8. Which of the following can be calculated from the parameters given?

  1. HALE
  2. DALY
  3. DFLE
  4. QALY

Correct Answer: QALY

Topic: Communicable and Non-communicable Diseases

Q.25. Although many animals are implicated in the spread of rabies, dogs are the most common ones. Also, it usually affects children in developing countries. Knowing this, what is the most cost-effective and logical way to reduce the incidence of rabies?

  1. Testing all dogs for rabies
  2. Reduce stray dog population and vaccinate all dogs
  3. Increase the laboratory facilities
  4. Increase capacity of healthcare workers for surveillance

Correct Answer:  Reduce stray dog population and vaccinate all dogs

Gynaecology & Obstetrics

Topic: Obstetrics

Q.26. A woman at 26 weeks of gestation presents for routine evaluation. On examination, fundal height corresponds to 24 weeks. Ultrasonography revealed decreased amniotic fluid. Which of the following conditions would have led to this presentation?

  1. Renal agenesis
  2. Tracheoesophageal fistula
  3. Cardiac abnormalities
  4. Ureteral stricture

Correct answer: Renal agenesis

Topic: Obstetrics

Q.27. A type 1 diabetic mother is on magnesium sulfate infusion post – cesarean section for preeclampsia. She develops delirium and is drowsy. She has a respiratory rate of 10/min, random blood glucose level of 240 mg / dL, oliguria, and bilaterally absent knee reflex. What is the cause of her condition?

  1. Magnesium sulfate toxicity
  2. Diabetic ketoacidosis
  3. Eclampsia
  4. Diabetes insipidus

Correct answer: Magnesium sulfate toxicity

Forensic Medicine

Topic: Court of Law 

Sub-Topic: 

Q.28. A 45-year-old female patient is told about the benefits and complications of a hysterectomy, and she agrees to the procedure. What kind of consent is this?

  1. Informed consent
  2. Implied consent
  3. Opt-out
  4. Passive consent

Correct answer: Informed consent

Topic: Asphyxial Deaths 

Q.29. A dead body is brought for evaluation. On post-mortem examination, a ligature completely encircled the neck, horizontal, and below the thyroid level was seen. There was no dribbling of saliva. What is the cause of death?

  1. Throttling
  2. Ligature strangulation
  3. Gagging
  4. Hanging

Correct answer: Ligature strangulation

Download complete

NEET PG 2021 Previous Year Questions

NEET PG 2021 was a significant national-level entrance examination conducted on September 11, 2021, for aspiring medical professionals seeking admission to postgraduate medical courses in India. Check out the NEET PG 2021 previous year questions below - 

Microbiology

Topic: Parasitology

Q.1. A patient from Uttar Pradesh presented with fever, pallor, and hepatosplenomegaly. Peripheral smear examination showed pancytopenia. Buffy coat examination showed macrophages laden with organisms with a kinetoplast. What is the vector for the likely disease?

  1. Sandfly
  2. Tse-Tse fly
  3. Triatomine bug
  4. Female anopheles mosquito

Correct Answer: Sandfly

Topic: Virology

Q.2. An unimmunized 2-year old child presented with coryza, conjunctivitis, and bluish-white spots in his buccal mucosa near the lower molar teeth. A day later, he developed a maculopapular rash on the face and neck. What is the nature of the causative virus?

ss- Single stranded

ds – Double – stranded

  1. Enveloped ss RNA
  2. Naked ss RNA
  3. Naked ds RNA
  4. Enveloped ds RNA

Correct Answer: Enveloped ss RNA

Radiology

Topic: Basics of Radiology 

Q.3. A patient following a skid resulting in a motor traffic accident was brought to the emergency room 2 hours later. On examination, he was stable with GCS 15/15. The pupil was reactive to light. Tenderness and bruising over the left lower chest wall with petechiae was seen. Severe tenderness was elicited in the left hypochondriac region and BP- 90/50 mm of Hg. What is the best investigation used in the ER?
GCS: Glasgow coma scale
FAST: Focused Assessment with Sonography for Trauma

  1. FAST
  2. X-ray
  3. Diagnostic peritoneal lavage
  4. CT scan

Correct Answer: FAST

Topic: Radiotherapy

Q.4. A child with acute lymphoblastic leukaemia undergoes prophylactic irradiation prior autologous hematopoietic stem cell transplantation. Which of the following will be the least affected?

  1. Spermatogonia
  2. Intestinal epithelial cells
  3. Neurons
  4. Bone marrow/ erythroid precursor cells

Correct Answer: Neurons 

Anatomy

Topic: Neuro Anatomy

Q.5. A patient presented with vision loss. On radiological investigation, an aneurysm causing damage to the optic chiasma was noted. Which of the following arteries is most likely to be the artery that is causing the damage?

  1. Anterior communicating artery
  2. Anterior choroidal artery
  3. Middle cerebral artery
  4. Anterior cerebral artery

Correct Answer: Anterior communicating artery

Topic: Upper Limb

Q.6. A 7-year-old boy was brought to the hospital with multiple fractures of the humerus secondary to a fall from height. On examination, there is difficulty in flexion of the elbow and supination of the forearm and associated loss of sensation over the lateral aspect of the forearm. Which is the nerve most likely to be injured?

  1. Median nerve
  2. Radial nerve
  3. Musculocutaneous nerve
  4. Ulnar Nerve

Correct Answer: Musculocutaneous nerve

Biochemistry

Topic: Lipid Chemistry

Q.7. A 5 year old boy presented with easy fatigability, irritability and inability to concentrate. Labs revealed the following findings a and b (light and oil immersion respectively) on the bone marrow aspiration. Which of the following is the most likely enzyme deficient in this condition?

  1. Hexosaminidase
  2. Glucocerebrosidase
  3. Sphingomyelinase
  4. N-acetylglucosaminidase

Correct Answer: Glucocerebrosidase

Topic: Oxidative Phosphorylation 

Q.8. You are conducting an experiment on mitochondrial respiration. You add malate/pyruvate and respiration is normal. You add succinate and respiration is normal. When you add another substance in the presence of pyruvate/succinate and malate, respiration is blocked. Which of the following substances is most likely added?

  1. Rotenone
  2. Antimycin A
  3. Oligomycin
  4. 2,4-dinitrophenol

Correct Answer: Oligomycin

Ophthalmology

Topic: Neuro Ophthalmology

Q.9. A 33-year-old woman presents with complaints of progressive loss of vision in the right halves of both eyes. Where is the lesion located in the optic pathway?

  1. Left optic tract
  2. Right visual cortex
  3. Optic chiasma
  4. Right optic nerve

Correct Answer: Left optic tract

Topic: Optics

Q.10. A 15-year-old girl is not compliant with spectacles for her myopic astigmatism. What would be the appropriate management in her case?

  1. LASIK
  2. Spherical equivalent spectacles
  3. Femto LASIK
  4. Implantable collamer lens

Correct Answer: Spherical equivalent spectacles

Surgery

Topic: Urology

Q.11. A 75-year-old man with prostate carcinoma presents to you with a PSA of 9 ng/mL with a small tumor focus. His Gleason score is 6. What will be your most likely management?
PSA- Prostate-specific antigen

  1. Radical prostatectomy
  2. External beam radiation
  3. Brachytherapy
  4. Active surveillance

Correct Answer: Active surveillance

Topic: Others

Q.12. A patient after a road traffic accident presented with pain in the abdomen. The resident examined the child and found that vitals were stable and tenderness was present in the left lumbar region. Which is the best investigation of choice?

  1. Contrast enhanced CT scan
  2. Retrograde urethrogram
  3. Wait and watch
  4. Emergency laparotomy

Correct Answer: Contrast enhanced CT scan

Dermatology

Topic: Immunobullous Disorders

Q.13. A 30-year-old presented with flaccid bullae on her skin which are easy to rupture. The biopsy revealed a suprabasal split. What is the most likely diagnosis?

  1. Pemphigus vulgaris 
  2. Pemphigus foliaceus
  3. Pemphigus vegetans
  4. Erythema multiforme

Correct Answer: Pemphigus vulgaris 

Topic: Bacterial Infections

Q.14. A patient taking multi-drug therapy (MDT) presents with worsening of existing lesions and nerve involvement. What will be your next best step of action?

  1. Stop MDT, start systemic corticosteroids
  2. Continue MDT, start systemic steroids
  3. Stop MDT thalidomide
  4. Continue MDT, start thalidomide

Correct Answer: Continue MDT, start systemic steroids

Anaesthesia

Topic: Monitoring in Anesthesia

Q.15. A 30-Year-old male patient was intubated for surgery by the final-year resident. Which of the following is the best method to confirm the position of the endotracheal tube?

  1. X-ray chest
  2. Auscultation
  3. End-tidal CO2 concentration
  4. Chest rise

Correct Answer: End-tidal CO2 concentration

Topic: Monitoring in Anesthesia

Q.16. A patient undergoing surgery is maintained on anesthesia with halothane. During the procedure, the patient suddenly developed hyperthermia and muscle rigidity. Which of the following drugs is most likely implicated in this condition?

  1. D-curare
  2. Suxamethonium
  3. Cis-atracurium
  4. Rocuronium

Correct Answer: Suxamethonium

ENT

Topic: Larynx

Q.17. Following total thyroidectomy, a patient started having difficulty in breathing, and repeated attempts to extubate were unsuccessful. The most probable cause is _________

  1. Superior laryngeal nerve injury
  2. Unilateral recurrent laryngeal nerve injury
  3. Bilateral recurrent laryngeal nerve injury
  4. Hematoma

Correct Answer: Bilateral recurrent laryngeal nerve injury

Topic: Nose and Paranasal Sinuses

Q.18. A female patient presents with nasal obstruction, nasal discharge, and loss of smell. On examination, foul-smelling discharge and yellowish – green crusts are present in the nasal cavity. She is found to have merciful anosmia. Which of the following findings can also be seen during the examination of her nose?

  1. Roomy nasal cavity
  2. Nasal polyps
  3. Inferior turbinate hypertrophy
  4. Foreign body

Correct Answer: Roomy nasal cavity

Pharmacology

Topic: CVS Pharmacology

Q.19. A patient with renal insufficiency presented with very low urine output, pedal edema, headache and BP 160/90. Which anti-hypertensive can be administered?

  1. Aliskiren
  2. Chlorthalidone
  3. Amlodipine
  4. Prazosin

Correct Answer: Amlodipine

Topic: Anticancer Drugs 

Q.20. Which of the following interleukin (IL) is inhibited by tocilizumab?

  1. IL-6
  2. IL-2
  3. IL-4
  4. IL-5

Correct Answer: IL-6

Orthopaedics

Topic: Spine + Pelvis + Lower Limb Traumatology

Q.21. A 20-year-old male patient presented with a history of lower backache and early morning stiffness for two years. He also gave a history of bilateral heel pain for 6 months. Which of the following is the most likely diagnosis?

  1. Ankylosing spondylitis
  2. Tuberculosis of the spine
  3. Disc prolapse
  4. Mechanical pain

Correct Answer: Ankylosing spondylitis

Topic: Spine + Pelvis + Lower Limb Traumatology

Q.22. A patient was brought to the hospital with complaints of pain around the left hip joint following a road traffic accident. On examination, the affected limb was flexed, adducted, and medially rotated with obvious shortening. What is the most likely diagnosis?

  1. Anterior hip dislocation
  2. Posterior hip dislocation
  3. Transcervical fracture
  4. Intertrochanteric fracture

Correct Answer: Posterior hip dislocation

Psychiatry

Topic: Psychology 

Q.23. During psychotherapy, the therapist had mixed conscious and unconscious feelings toward his patient. This is known as ___________.

  1. Countertransference
  2. Transference
  3. Dissociation
  4. Preoccupation

Correct Answer: Countertransference

Topic: Substance-Related and Addictive Disorders

Q.24. A patient stopped alcohol consumption for 3 days and presented with irritability, disorientation, paranoid delusions, agitation, visual hallucinations, and altered sensorium. What is the likely diagnosis in this case?

  1. Delirium tremens
  2. Wernicke’s encephalopathy
  3. Korsakoff psychosis
  4. Alcohol-induced psychosis

Correct Answer: Delirium tremens

Physiology

Topic: General Physiology

Q.25. A 30-year-old man weighing 70 kg had a sodium level of 120 mEq/L. Calculate the sodium deficit.

  1. 280 mEq
  2. 480 mEq
  3. 840 mEq
  4. 1400 mEq

Correct Answer: 840 mEq

Topic: General Physiology

Q.26. A body fluid sample is being studied with Na: 10 mEq/L K: 140 mEq/L, Cl: 4 mEq/L. Identify the compartment from which the piece has been obtained.

  1. Interstitial
  2. Intracellular fluid
  3. Extracellular fluid
  4. Plasma

Correct Answer: Intracellular fluid

Medicine 

Topic: Neurology

Q.27. A 37-year-old woman presents with headaches for 6 months. She has been taking analgesics regularly. The headache recently increased in severity for 3 days but was reduced in stopping the analgesic. What is the likely diagnosis?

  1. Medication overuse headache
  2. Tension headache
  3. Chronic migraine
  4. Cluster headache

Correct Answer: Medication overuse headache

Topic: Cardiology

Q.28. A 20-year-old woman presents with breathlessness and chest pain. She is a known case of mitral stenosis. Her pulse is irregularly irregular. No thrombus is seen on echocardiography. What is the best agent to prevent future thrombotic events?

  1. Dabigatran
  2. Aspirin - 150 mg
  3. Oral warfarin
  4. Aspirin + Clopidogrel

Correct Answer: Oral warfarin

Pediatrics

Topic: Paediatric Cardiology

Q.29. In foetal circulation, which vessel carries deoxygenated blood back to the placenta?

  1. Umbilical Vein
  2. Umbilical Artery
  3. Descending aorta
  4. Pulmonary Artery

Correct Answer: Umbilical artery

Topic: Paediatric Neurology

Q.30. A 10-year-old boy presented with seizures. His past history is significant for an episode of fever with rash at 1 year of age which resolved spontaneously. What is the most helpful investigation to diagnose his condition?

  1. lgG measles in CSF
  2. MRI mesial temporal lobe sclerosis
  3. lgM measles in CSF
  4. C1Q4 antibodies in the CSF

Correct Answer: lgG measles in CSF

PSM 

Topic: Environment and Health

Q.31. Which of the following thermometers is used to measure the low air velocity rather than the cooling power of the air?

  1. Kata thermometer
  2. Globe thermometer
  3. Wet thermometer
  4. Dial thermometer

Correct Answer: Kata thermometer

Topic: Environment and Health

Q.32. There is an outbreak of buboes in the community. What is the vector responsible for this condition?

  1. Xenopsylla cheopis
  2. Phlebotomus argentipes
  3. Ixodes tick
  4. Female Anopheles mosquito

Correct Answer: Xenopsylla cheopis

Gynaecology & Obstetrics

Topic: Gynecology

Q.33. A 27 year old female patient was found to have uterus didelphys. Which of the following is not likely to be a complication to this uterine anomaly?

  1. Abortion
  2. Endometriosis
  3. Preterm labour
  4. Transverse lie 

Correct Answer: Transverse lie 

Topic: Obstetrics

Q.34. A pregnant woman at 36 weeks of gestation with a prosthetic valve replacement for mitral stenosis is on warfarin therapy. Her INR is 3. What is the appropriate next step in management ?

LMWH- Low Molecular Weight Heparin

  1. Stop warfarin and start LMWH
  2. Stop warfarin and start heparin
  3. Continue warfarin
  4. Stop warfarin, start LMWH and aspirin

Correct Answer: Stop warfarin and start LMWH

Forensic Medicine

Topic: Forensic Toxicology

Q.35. A guy was playing in a garden. Suddenly, he collapsed and was rushed to the hospital. His friend who played with him in the garden informed the doctors that he ate one of the fruits in the garden post which he seemed to have developed these symptoms. He also had irritability, restlessness, dry hot skin, and was unable to pass urine and stools. Identify the poison and its appropriate antidote.

  1. Datura, Pralidoxime
  2. Datura, Physostigmine
  3. Yellow oleander, Digoxin
  4. Yellow oleander, Physostigmine

Correct Answer: Datura, Physostigmine

Topic: Forensic Thanatology 

Q.36. A man was working in a field and he suddenly collapsed on a hot summer afternoon. On examination, there were no physical signs of dehydration. His serum electrolyte level was normal. The doctor found that the body temperature was 106 degrees Fahrenheit. Which of the following symptoms is least likely to be seen in this patient?

  1. Hot skin
  2. Hypotension
  3. Sweating 
  4. Disorientation

Correct Answer: Sweating

Download complete

NEET PG 2020 Previous Year Questions

NEET PG 2020 was held on January 5, 2020, for medical graduates seeking admission to postgraduate medical courses in India. Check out the NEET PG 2020 previous year questions below- 

Microbiology

Topic: Mycology

Q.1. A 35-year-old HIV-positive patient presents with mucosal lesions in the mouth as shown in the image below. On microscopy , bugging yeasts and pseudohyphae are seen. What is the likely diagnosis?

  1. Oral candidiasis
  2. Hairy leukoplakia 
  3. Lichen planus
  4. Diphtheria

Correct Answer: Oral candidiasis

Topic: Virology

Q.2. Zika virus is transmitted by:

  1. Aedes aegypti
  2. Culex
  3. Anopheles
  4. Phlebotomus papatasii

Correct Answer: Aedes aegypti

Radiology

Topic: Head & Neck Imaging 

Q.3. A cystic lesion in the suprasellar region with calcification is seen on the MRI. What is the most likely diagnosis?

  1. Craniopharyngioma
  2. Pituitary adenoma
  3. Meningioma
  4. Oligodendroglioma

Correct Answer: Craniopharyngioma

Topic: OBGyn Imaging

Q.4. Personal monitoring of radiation is done by:

  1. TLD badge
  2. Collimators
  3. Linear accelerator
  4. Grid

Correct Answer: TLD badge

Anatomy

Topic: Back Region

Q.5. The muscle marked by the arrow in the image below is innervated by the :

  1. Dorsal scapular nerve
  2. Suprascapular nerve
  3. From the dorsal rami of C1
  4. Subscapular nerve

Correct Answer: Dorsal scapular nerve

Topic: Upper Limb

Q.6. Which of the following forms the lateral boundary of the anatomical snuff box?

  1. Extensor pollicis brevis and abductor pollicis longus
  2. Extensor pollicis longus and abductor pollicis brevis
  3. Extensor pollicis longus and extensor pollicis brevis
  4. Abductor pollicis longus and abductor pollicis brevis

Correct Answer: Extensor pollicis brevis and abductor pollicis longus

Biochemistry

Topic: Introduction

Q.7. What is the function of the proteasome?

  1. Protein folding
  2. Pot-translation modification
  3. Protein degradation
  4. Protein sorting

Correct Answer: Protein degradation

Topic: Amino Acids and Proteins Chemistry

Q.8. Replacing alanine by which amino acid, will increase the UV absorbance of protein at 280 nm wavelength?

  1. Leucine
  2. Proline
  3. Arginine
  4. Tryptophan

Correct Answer: Tryptophan

Ophthalmology

Topic: Cornea

Q.9. Which of the following layers is responsible for maintaining the hydration and transparency of the cornea?

  1. Descemet’s membrane
  2. Endothelial cells
  3. Stroma
  4. Corneal epithelium

Correct Answer: Endothelial cells

Topic: Cornea

Q.10. What is the characteristic feature of a fungal ulcer?

  1. Reverse hypopyon
  2. Dendritic ulcer on a fluorescein dye
  3. Ring abscess
  4. Satellite lesion

Correct Answer: Satellite lesion

Surgery

Topic: Endocrine Surgery

Q.11. What is the most common site of gastrinoma in MEN 1 syndrome?

  1. Jejunum
  2. Ileum
  3. Duodenum
  4. Stomach

Correct Answer: Duodenum

Topic: Cardiothoracic Vascular Surgery

Q.12. A 50-year-old male who is a chronic smoker presented to the hospital with intermittent claudication pain of both thigh and buttock region on walking about 500 m. Which of the following is a likely diagnosis?

  1. Arterial disease involving the superficial femoral artery
  2. Arterial disease with aortoiliac involvement
  3. Femoral venous insufficiency
  4. Arterial disease involving the profunda femoris artery

Correct Answer: Arterial disease with aortoiliac involvement

Dermatology

Topic: Miscellaneous Disorders

Q.13. Which of the following drugs used as nail lacquer belongs to morpholines?

  1. Amorolfine
  2. Oxiconazole
  3. Ciclopirox olamine
  4. Tioconazole

Correct Answer: Amorolfine

Topic: Skin Appendages and their Disorders

Q.14. A child was born with membranes around the body and had ectropion and eclabium. He is brought to the OPD with lesions covering his face, trunk, and extremities. Which of the following is an unlikely diagnosis?

  1. Icthyosis vulgaris
  2. Lamellar ichthyosis
  3. Bathing suit ichthyosis
  4. Harlequin ichthyosis

Correct Answer: Icthyosis vulgaris

Anaesthesia

Topic: Pre-Anaesthetic Evaluation

Q.15. Which of the following investigations provides the most accurate prognostic information with respect to predicting risk of perioperative cardiac complications?

  1. Exercise ECG testing
  2. Dobutamine stress echocardiography
  3. Myocardial perfusion scintigraphy
  4. Coronary angiography

Correct Answer: Dobutamine stress echocardiography

Topic: Neuromuscular Blockade

Q.16. A patient is given a nicotinic receptor antagonist as a muscle relaxant. Which drug is given postoperatively to recover from muscle weakness?

  1. Physostigmine
  2. Neostigmine
  3. Carbachol
  4. Succinylcholine

Correct Answer: Neostigmine

ENT

Topic: Ear

Q.17. From which of the following structures does the saccule develop?

  1. Saculus anterior
  2. Saculus posterior
  3. Pars superior
  4. Pars inferior

Correct Answer: Pars inferior

Topic: Ear

Q.18. What is the surgery done to widen the cartilaginous part of the external auditory canal called?

  1. Meatoplasty
  2. Tympanoplasty
  3. Myringoplasty
  4. Otoplasty

Correct Answer: Meatoplasty

Pharmacology

Topic: Anticancer Drugs 

Q.19. Which of the following is an inhibitor of DNA synthesis?

  1. 6-mercaptopurine
  2. Mitomycin
  3. Actinomycin
  4. Asparaginase

Correct Answer: 6-mercaptopurine

Topic: Renal Pharmacology

Q.20. A patient on lithium developed hypertension. He was started on thiazides for hypertension. After a few days, he developed coarse tremors and other symptoms suggestive of lithium toxicity. Explain the likely mechanism of this interaction.

  1. Thiazide inhibits the metabolism of lithium
  2. Thiazides act as an add on drug to lithium
  3. Thiazides increase the tubular resorption of lithium
  4. Thiazides cause water loss thereby increase lithium levels

Correct Answer: Thiazides increase the tubular resorption of lithium

Orthopaedics

Topic: Arthritis

Q.21. A patient presented with pain in the hand. The joints involved were proximal interphalangeal joint, distal interphalangeal joint and first carpometacarpal joint. The wrist and metacarpophalangeal joints were spared. What is the likely diagnosis?

  1. Osteoarthritis
  2. Rheumatoid arthritis
  3. Psoriatic arthritis
  4. Pseudogout

Correct Answer: Osteoarthritis

Topic: Sports Injuries

Q.22. A maid is playing with a child by spinning him while holding his hands. A few hours later, the child starts crying, does not use his arm, and does not let anybody touch him. What is the possible diagnosis?

  1. Pulled elbow
  2. Olecranon fracture
  3. Fracture head of radius
  4. Elbow dislocation
    Correct Answer: Pulled elbow

Psychiatry

Topic: Substance-Related and Addictive Disorders

Q.23. A 25-year-old male presented to the clinic with a complaint of the feeling of insects crawling under his skin. Which of the following drug abuse can cause the symptoms of this patient?

  1. Cannabis
  2. Cocaine
  3. Amphetamine
  4. Alcohol

Correct Answer: Cocaine

Topic: Mood Disorders

Q.24. A 60-year-old male who lost his wife 3 months back complains that his intestines have become rotten. He feels responsible for his wife’s death and should be sent to prison. He complains of feeling low all the time and has lost interest in daily activities since his wife’s death. What is the most likely diagnosis?

  1. Normal grief reaction
  2. Psychotic depression
  3. Delusional disorder
  4. Schizophrenia

Correct Answer: Psychotic depression

Physiology

Topic: Respiratory System

Q.25. A person develops headache and breathlessness on a trekking expedition following a rapid ascent to over 3000 meters above the sea level. Which of the following is not used in the treatment of this condition?

  1. IV digoxin
  2. Immediate descent
  3. Administration of oxygen
  4. Tablet acetazolamide

Correct Answer: IV digoxin

Topic: Endocrine and Reproductive System

Sub-Topic: 

Q.26. Prolactin level is highest during?

  1. 24 hours after ovulation
  2. 24 hours after delivery
  3. REM sleep
  4. After running four 1 hours

Correct Answer: 24 hours after delivery

Medicine 

Topic: Nephrology / Kidney Disease

Q.27. Which of the following is associated with pauci-immune glomerulonephritis?

  1. SLE nephritis
  2. Anti-GBM glomerulonephritis
  3. IgA nephropathy
  4. Granulomatosis with polyangiitis (GPA)

Correct Answer: Granulomatosis with polyangiitis (GPA)

Topic: Emergency Medicine

Q.28. At a high altitude of 3000m, a person complains of breathlessness. All of the following can be used for management of this person except

  1. Intravenous digoxin
  2. Oxygen supplementation
  3. Immediate descent
  4. Acetazolamide

Correct Answer: Intravenous digoxin

Pediatrics

Topic: Musculoskeletal Disorders in Children

Q.29. A 4-yr old male child presents with muscle weakness. His mother says that her child has difficulty climbing stairs and getting up from the floor. On muscle biopsy, small muscle fibrils and absence of dystrophin was found. What is the diagnosis out of given options?

  1. Becker's muscle dystrophy
  2. Duchenne muscular dystrophy
  3. Myotonic dystrophy
  4. Limb-girdle muscular dystrophy

Correct Answer: Duchenne muscular dystrophy

Topic: Paediatric Neurology

Q.30. Two girls in the same class are diagnosed with meningococcal meningitis. Their 12-year old friend from the same school is afraid of contracting the disease. What advice should be given to the exposed students?

  1. Two doses of polysaccharide vaccine
  2. Antibiotic prophylaxis
  3. Two doses of conjugate vaccine
  4. Single dose of meningococcal vaccine

Correct Answer: Antibiotic prophylaxis

PSM 

Topic: Health Planning and Management

Q.31. The monetary benefit is measured in which of the following analyses?

  1. Cost-effective analysis
  2. Cost-benefit analysis
  3. Network analysis
  4. Input-Output analysis

Correct Answer: Cost-benefit analysis

Topic: Allied Health Disciplines

Q.32. An employee diagnosed with TB gets extended sickness benefit for?

  1. 1 year
  2. 2 years
  3. 3 years
  4. 4 years

Correct Answer: 2 years

Gynaecology & Obstetrics

Topic: Obstetrics

Q.33. 24 yr old women who had home delivery 2 weeks back, now present with complete perineal tear. What is the next line of management?

  1. Repair immediately
  2. Repair after 3 weeks
  3. Repair after 6 months
  4. Repair after 3 months

Correct Answer: Repair after 3 months

Topic: Gynecology

Q.34. The remnants of Wolffian ducts in females are located in

  1. Pouch of Douglas
  2. Leaves of broad ligament
  3. Uterovesical pouch
  4. Iliac fossa

Correct Answer: Leaves of broad ligament

Forensic Medicine

Topic: Forensic Identification

Q.35. A deceased male was brought to the morgue for a post-mortem examination. His relatives reported that he had a tattoo over his skin, which was not found on examination. Which of the following structures would you examine?

  1. Lymph nodes
  2. Liver
  3. Arteries
  4. Skin

Correct Answer: Lymph nodes

Topic: Forensic Toxicology

Q.36. The most common drug which causes physical dependence is?

  1. Ketamine
  2. Heroin
  3. LSD
  4. Phencyclidine

Correct Answer: Heroin

Download complete

NEET PG 2019 Previous Year Questions

NEET PG 2019 was conducted on 6th January, 2019 with huge participation. Check out the previous year questions below - 

Microbiology

Topic: Systemic Bacteriology

Q.1. BCYE medium is used to culture:

  1. Leptospira
  2. Bacillus cereus
  3. Campylobacter
  4. Legionella

Correct Answer: Legionella

Topic: Systemic Bacteriology

Q.2. A 16-year-old boy is admitted with fever, icterus, conjunctival suffusion, and hematuria for 15 days. Which serological test should be done for diagnosis?

  1. Widal test
  2. Microscopic agglutination test
  3. Weil felix reaction
  4. Paul Bunnell test

Correct Answer: Microscopic agglutination test

Radiology

Topic: Basics of Radiology 

Q.3. What is the best imaging study for the earliest diagnosis of cerebral infarct?

  1. Flair MRI
  2. Non-contrast CT
  3. CECT
  4. Diffusion weighted MRI

Correct Answer: Diffusion weighted MRI

Topic: Basics of Radiology 

Q.4. Which of the following is a water soluble contrast?

  1. Iohexol
  2. Barium 
  3. Borium 
  4. Calcium

Correct Answer: Iohexol

Anatomy

Topic: Back Region

Q.5. The movements at the _______ joint permit a person to look to the right and left.

  1. Atlanto-occipital
  2. Atlanto -axial
  3. C2-C3
  4. C3-C4

Correct Answer: Atlanto -axial

Topic: Abdomen

Q.6. What is the basis for the formation of omphaloceles?

  1. Excessive length of the intestine
  2. Physiological hernia failing to go back
  3. Herniation of the liver
  4. Herniation of umbilicus

Correct Answer: Physiological hernia failing to go back

Biochemistry

Topic: Lipid Metabolism

Q.7. An infant present with hypotonia and seizures. It was confirmed to be cerebrohepatorenal syndrome. Which of the following is accumulated in the brain in cerebrohepatorenal syndrome?

  1. Glucose
  2. Long chain fatty acid
  3. Lactic Acid
  4. Triglycerides

Correct Answer: Long Chain Fatty Acid

Topic: Genetics

Q.8. Which of the following is autosomal dominant?

  1. Cystic fibrosis
  2. Duchene muscular dystrophy
  3. Sickle cell anaemia
  4. Achondroplasia

Correct Answer: Achondroplasia

Ophthalmology

Topic: Optics

Q.9. Which of the following would be prescribed for simple myopic astigmatism?

  1. +1.00 DS
  2. -1.00 DC × 180 Degree
  3. -1.00 DS
  4. -1.00DS – 1.00 DC × 180 Degree

Correct Answer: -1.00 DC × 180 Degree

Topic: Retina

Q.10. Not true about retinitis pigmentosa is:

  1. Retinal pigmentation
  2. Pale waxy disc
  3. Narrowing of vessels
  4. ERG - Normal

Correct Answer: ERG- Normal

Surgery

Topic: Urology

Q.11. Which of the following drugs cause carcinoma bladder?

  1. Cyclophosphamide
  2. Cisplatin
  3. Taxane
  4. Tamoxifen

Correct Answer: Cyclophosphamide

Topic: Gastrointestinal Surgery

Q.12. Dohlman's procedure is for:

  1. Meckel's diverticulum
  2. Zenker’s diverticulum
  3. Bochadlek hernia
  4. Menetrier's disease

Correct Answer: Zenker’s diverticulum

Dermatology

Topic: Eczema

Q.13. Bindi Leukoderma is caused by which chemical?

  1. A Mono-benzyl ether of Hydroquinone (MBH)
  2. Crocein Scarlet MOO and Solvent Yellow 3
  3. p-phenylenediamine (PPD)
  4. Para Tertiary butylphenol (PTBP)

Correct Answer: Para Tertiary butylphenol (PTBP)

Topic: Sexually Transmitted Infections

Q.14. A young female presented with vaginal itching and green frothy genital discharge. Strawberry vagina is seen on examination. What will be the drug of choice?

  1. Doxycycline
  2. Oral fluconazole
  3. Metronidazole
  4. Azithromycin

Correct Answer: Metronidazole

Anaesthesia

Topic: Neuromuscular Blockade

Q.15. What is the mechanism of action of the curare group of muscle relaxants?

  1. Persistently depolarizing at neuromuscular junction
  2. Competitively blocking the binding of ACh to its receptors
  3. Repetitive stimulation of ACh receptors on muscle endplate
  4. Inhibiting the calcium channels on presynaptic membrane

Correct Answer: Competitively blocking the binding of ACh to its receptors

Topic: Inhalational Anaesthetic Agents

Q.16. Which of the following is not used for induction of anaesthesia in paediatric patients?

  1. Halothane
  2. Sevoflurane
  3. Desflurane
  4. Nitrous oxide

Correct Answer: Desflurane

ENT

Topic: Ear

Q.17. Which of the following is not a feature of tubercular otitis media?

  1. Ear ache
  2. Multiple perforations
  3. Pale granulation
  4. Foul smelling ear discharge

Correct Answer: Ear ache

Topic: Nose and Paranasal Sinuses

Q.18. Pott’s puffy tumor is

  1. Subperiosteal abscess of frontal bone
  2. Subperiosteal abscess of ethmoid bone
  3. Mucocele of frontal bone
  4. Mucocele of ethmoid bone

Correct Answer: Subperiosteal abscess of frontal bone

Pharmacology

Topic: Drugs Affecting Blood and Blood formation

Q.19. Which of the following is an oral factor Xa inhibitor?

  1. Bivalirudin
  2. Dabigatran
  3. Rivaroxaban
  4. Enoxaparin

Correct Answer: Rivaroxaban

Topic: Antimicrobials : Antibacterial Drugs

Q.20. What is the mechanism of action of oseltamivir and zanamivir?

  1. DNA polymerase inhibition
  2. Protein synthesis inhibition
  3. Nucleotide analogue
  4. Neuraminidase inhibition

Correct Answer: Neuraminidase inhibition

Orthopaedics

Topic: General + Upper Limb Traumatology

Q.21. Which of the following fractures is most prone to non-union?

  1. Proximal scaphoid
  2. Intertrochanteric
  3. Distal radius
  4. Talar neck

Correct Answer: Proximal scaphoid

Topic: Miscellaneous

Q.22. Degloving injury refers to_____?

  1. Skin and subcutaneous fat are stripped from the underlying fascia
  2. Skin, subcutaneous fat and fascia are stripped from tendons
  3. Skin, subcutaneous fat, fascia and tendons are stripped from bone
  4. Only skin is stripped off

Correct Answer: Skin and subcutaneous fat are stripped from the underlying fascia

Psychiatry

Topic: Substance-Related and Addictive Disorders

Q.23. Which of the following drugs is used as an antismoking agent?

  1. Busulfan
  2. Acamprosate
  3. Varenicline
  4. Gabapentin

Correct Answer: Varenicline

Topic: Schizophrenia Spectrum and Other Psychotic Disorders

Q.24. Which of the following is not a risk factor for delusional disorder?

  1. Recent immigration
  2. Social isolation 
  3. Family history
  4. Young age

Correct Answer: Young age

Physiology

Topic: Excretory System

Q.25. According to myogenic hypothesis of renal autoregulation the afferent arterioles contract in response to stretch – induced by ____________

  1. NO
  2. Noradrenaline
  3. Opening of calcium channels
  4. Adenosine

Correct Answer: Opening of calcium channels

Topic: The Nervous System

Q.26. Which of the following is true for decorticate rigidity?

  1. It is produced by the removal of cerebral cortex and basal ganglia
  2. Flexion of lower limbs and extension of upper limbs occurs
  3. Rigidity is pronounced
  4. It is characterised by flexion of upper limbs and extension of lower limbs

Correct Answer: It is characterised by flexion of upper limbs and extension of lower limbs

Medicine 

Topic: Pulmonology

Q.27. Nasal polyps are commonly associated with: -

  1. Intrinsic Asthma
  2. Brittle Asthma
  3. Extrinsic Asthma
  4. Exercise induced asthma

Correct Answer: Intrinsic Asthma

Topic: Haematology

Q.28. A 47 year old man with a diagnosis of acute myeloid leukaemia with a blood type O negative blood group presents to the transplant clinic to discuss proceeding with an allogeneic stem cell transplant. Which of the following would be an optimal donor?

  1. His identical twin brother
  2. Umbilical cord transplant
  3. His HHLA identical 50-year-old brother who is otherwise healthy and is blood type O+
  4. An HLA identical matched unrelated donor who is blood type

Correct Answer: His HHLA identical 50-year-old brother who is otherwise healthy and is blood type O+

Pediatrics

Topic: Paediatric Gastroenterology

Q.29. Unconjugated hyperbilirubinemia, which did not subside even after 3 weeks of birth, was observed in a neonate. On investigating, liver enzymes, PT/INR and albumin levels were normal. No hemolysis was seen on a peripheral blood smear. A drop in bilirubin level was observed within a week after treatment with phenobarbital. What is the most likely diagnosis?

  1. Rotor syndrome
  2. Crigler Najjar type 2
  3. Dubin Johnson syndrome
  4. Crigler Najjar type 1

Correct Answer: Crigler Najjar type 2

Topic: Pediatric Respiratory Disorders

Q.30. Which of the following is not seen in a child with cystic fibrosis?

  1. Sweat chloride test chloride conc of 70mEq/L
  2. Increase immunoreactive trypsinogen level
  3. Hyperkalemia
  4. Contraction alkalosis

Correct Answer: Hyperkalemia

PSM 

Topic: Concept of Health and Disease

Q.31. Which of the following parameters would you use to check the efficiency of the surveillance system for malaria under the National Vector Borne Disease Control Programme?

  1. Annual Parasite Index
  2. Annual Blood Examination Rate
  3. Slide positivity rate
  4. Slide falciparum rate

Correct Answer: Annual Blood Examination Rate

Topic: Screening of Diseases

Q.32. Which of the following is the best level of prevention of breast cancer?

  1. Specific protection
  2. Early diagnosis and treatment
  3. Disability limitation
  4. Rehabilitation

Correct Answer: Early diagnosis and treatment

Gynaecology & Obstetrics

Topic: Obstetrics

Sub-Topic: 

Q.33. In which trimester does acute fatty liver manifest most commonly during pregnancy?

  1. First trimester
  2. Second trimester
  3. Third trimester
  4. Both A & B

Correct Answer: Third trimester

Topic: Obstetrics

Q.34. You discover that there is evidence of cardiac malformation in a foetus during a routine USG scan at 16 weeks. What is the minimum level of fasting blood sugar at which you can suspect overt diabetes?

  1. 106 mg/dL
  2. 126 mg/dL
  3. 116 mg/dL
  4. 130 mg/dL

Correct Answer: 126 mg/dL

Forensic Medicine

Topic: IPC Sections 

Q.35. Which Section of the IPC deals with the punishment of perjury?

  1. 191
  2. 192
  3. 193
  4. 197

Correct Answer: 193

Topic: Sexual Jurisprudence

Q.36. Frotteurism is _____________

  1. Obtaining sexual pleasure by wearing clothes of opposite sex
  2. Desire to seek surgery to become member of opposite sex
  3. Sexual gratification by rubbing his private parts against another person
  4. Exposure of one’s genitals to an unsuspecting stranger

Correct Answer: Sexual gratification by rubbing his private parts against another persont

Download complete
Rapid Revision 5.0

Radiology

Topic: Head & Neck Imaging

Q.7. A 2-month-old infant is brought to the OPD with a parietal swelling present since birth. The X-ray image is given below. What could be the probable diagnosis?
1. Subgaleal hematoma
2. Cephalhematoma
3. Caput succedaneum
4. Enecephalocele

Correct Answer: Cephalhematoma

Topic: Genitourinary Radiology

Q.8. A patient presented with abdominal pain and sterile pyuria, and the x-ray showed the following features. What is the likely diagnosis?
1. Putty kidney
2. Nephrocalcinosis
3. Staghorn calculus
4. Psoas calcification

Correct Answer: Putty kidney

Microbiology

Topic: Virology

Q.9. A patient hailing from Delhi presents with fever, arthralgia, and extensive petechial rash for 3 days. Lab investigations revealed a hemoglobin of 9 g/dL, a white blood cell count of 9000 cells/mm3, a platelet count of 20000 cells/mm3, and a prolonged bleeding time. The clotting time was normal. What is the most likely diagnosis?
1. Dengue
2. Malaria
3. Scrub typhus
4. Typhoid

Correct Answer: Dengue

Ophthalmology

Topic: Neuro Ophthalmology

Q.10. A female presents with loss of vision in the right halves of both eyes. Where is the lesion located in the optic pathway?
1. Left optic tract
2. Optic radiation
3. Optic chiasma
4. Right optic tract

Correct Answer: Left optic tract

Topic: Cornea

Q.11. The eye examination of a 20-year-old male shows a golden ring on the iris. What is the next investigation to diagnose the condition?
1. Alpha-1-antitrypsin
2. Alpha-fetoprotein
3. Serum Iron
4. Serum ceruloplasmin

Correct Answer: Serum ceruloplasmin

Surgery

Topic: Urology

Q.12. A 55-year-old male presented with verrucous carcinoma around the glans of the penis. Examination reveals that the inguinal lymph nodes are not enlarged. What is the appropriate management for this patient?
1. Total penectomy
2. CO2 laser excision
3. Topical 5-fluorouracil
4. Partial penectomy

Correct Answer: Partial penectomy

Topic: Gastrointestinal Surgery

Q.13. Which of the following is the most common complication following ligation of the first vessel during abdominoperineal resection for rectal carcinoma?
1. Parasympathetic-bladder dysfunction and retrograde ejaculation.
2. Sympathetic-bladder dysfunction and impotence.
3. Sympathetic-retrograde ejaculation and bladder dysfunction.
4. Sympathetic-impotence and loss of cutaneous sensation in the perineal region

Correct Answer: Sympathetic-retrograde ejaculation and bladder dysfunction.

Dermatology

Topic: Introduction to Dermatology

Q.14. A female patient presented with acne that is not resolving on oral isotretinoin and antibiotics therapy. Which of the following is the next best investigation?
1. Look for dietary triggers.
2. Evaluate for hyperandrogenism
3. Check for antibiotic resistance
4. Look for drug triggers

Correct Answer: Evaluate for hyperandrogenism

Topic: Sexually Transmitted Infections

Q.15. A young woman complains of a painless ulcer in the genital area. It is associated with non-tender inguinal lymphadenopathy. What is the most likely diagnosis?
1. Chancroid
2. Syphilis
3. Herpes genitalis
4. Granuloma inguinale

Correct Answer: Syphilis

Anaesthesia

Topic: Pre-Anaesthetic Evaluation

Q.16. Which of the following is the drug of choice for preoperative antibiotic prophylaxis in a patient undergoing cardiac surgery?
1. Penicillin G
2. Erythromycin
3. Azithromycin
4. Cefazolin

Correct Answer: Cefazolin

ENT

Topic: Ear

Q.17. A female patient with hearing loss is examined and is found to be Rinne negative at 256 Hz and 512 Hz, while Rinne positive at 1024 Hz. What is the expected air conduction and bone conduction gap?
1. 30-45 dB
2. 15-30 dB
3. 45-60 dB
4. >60 dB

Correct Answer: 30-45 dB

Topic: Pharynx

Q.18. A 10-year-old child presents with throat pain, fever, and ear pain. He is diagnosed with recurrent tonsillitis. Which nerve is responsible for the ear pain in this patient?
1. Tympanic branch of the glossopharyngeal nerve
2. Greater auricular nerve
3. Auriculotemporal nerve
4. Auricular branch of the vagus nerve

Correct Answer: Tympanic branch of the glossopharyngeal nerve

Pharmacology

Topic: Antimicrobials : Antibacterial Drugs

Q.19. A bronchial asthma patient on inhalational steroids presented with white patchy lesions on the tongue and buccal mucosa. What is the drug that can be used to treat this condition?
1. Clotrimazole
2. Griseofulvin
3. Terbinafine
4. Flucytosine

Correct Answer: Clotrimazole

Topic: General Pharmacology

Q.20. A pregnant woman with a history of bronchial asthma is in the third stage of labour. Which drugs should be avoided in managing postpartum haemorrhage in this mother?
1. Carboprost
2. Oxytocin
3. Dinoprostone
4. Methyl ergometrine

Correct Answer: Carboprost

Orthopaedics

Topic: Peripheral Nerve Injuries

Q.21. A patient at the orthopaedics OPD complains of troubled sleep at night due to numbness and tingling sensation involving his lateral 3 digits. His symptoms are relieved as he lays his arms hanging from the bed. Which of the following options correctly describes his condition and the test used to assess it?
1. Guyon's canal syndrome, Froment's test
2. Carpal tunnel syndrome, Froment's test
3. Guyon's canal syndrome, Durkan's test
4. Carpal tunnel syndrome, Durkan's test

Correct Answer: Carpal tunnel syndrome, Durkan's test

Topic: Metabolic Disorders

Q.22. A child is brought to the orthopaedics OPD with a deformity in the lower limb and hyperpigmented skin lesions. The x-ray of her thigh is shown below. What is the most likely diagnosis?
1. Non-ossifying fibroma
2. Fibrous dysplasia
3. Paget's disease
4. Osteogenesis imperfecta

Correct Answer: Fibrous dysplasia

Psychiatry

Topic: Schizophrenia Spectrum and Other Psychotic Disorders

Q.23. A schizophrenic patient was prescribed drug A after he did not respond to haloperidol and thioridazine. He now presents with excessive salivation, an increase in blood glucose, and hyperlipidemia. What is drug A?
1. Ziprasidone
2. Risperidone
3. Clozapine
4. Aripiprazole

Correct Answer: Clozapine

Topic: Mood Disorders

Q.24. A woman, who is 4 days postpartum, presented with tearfulness, mood swings, and occasional insomnia. What is the likely diagnosis?
1. Postpartum depression
2. Postpartum blues
3. Postpartum psychosis
4. Postpartum anxiety

Correct Answer: Postpartum blues

Physiology

Topic: General Physiology

Q.25. A man was brought to the emergency after suddenly becoming unconscious while working in the field. On examination, his temperature was 105 degrees Fahrenheit, and his skin turgor was decreased. Which of the following would not be seen in the patient?
1. Tachypnea
2. Hypotension
3. Sweating
4. Red hot skin

Correct Answer: Sweating

Topic: Respiratory System

Q.25. A preterm baby who was delivered at 28 weeks developed respiratory distress syndrome. Which of the following is true about surface tension and compliance in this baby?
1. Surface tension - decreased; Compliance - increased
2. Surface tension - increased; Compliance - decreased
3. Both surface tension and compliance decreased
4. Both surface tension and compliance increased

Correct Answer: Surface tension - increased; Compliance - decreased

Medicine

Topic: Nephrology / Kidney Disease

Q.26. A patient with hyperkalemia and elevated urea levels underwent dialysis. Towards the end of the session, she became drowsy and had a sudden seizure episode. On examination, the patient was hypotensive. What is the treatment for this condition?
1. Bumetanide
2. Ethacrynic acid
3. Nesiritide
4. IV Mannitol

Correct Answer: IV Mannitol

Topic: Endocrinology

Q.27. A female patient presents to the emergency department with severe restlessness, palpitations, and tremors. She is a known case of bronchial asthma. On examination, the neck looks swollen. Blood pressure is elevated, and tachycardia is noted. ECG shows atrial fibrillation. Which of the following drugs is used for immediate management in this patient?
1. Diltiazem
2. Propranolol
3. Esmolol
4. Propylthiouracil

Correct Answer: Diltiazem

PSM

Topic: Nutrition and Health

Q.28. In a village, it is observed that several farmers have crossed gait and use a stick for support to stand up and walk. Due to poor yield from farms, they consume meals containing rice and pulses only. Supplementing their diet with which of the following vitamins could have prevented this?
1. Vitamin A
2. Vitamin D
3. Vitamin C
4. Vitamin B

Correct Answer: Vitamin C

Topic: Epidemiology

Q.29. Research is being conducted to find the association between aniline dye exposure and bladder cancer in workers who have worked in the industry for >20 years. Two groups were formed: one directly involved with dye handling and the other group consisting of office clerks not directly exposed to the dye. Years of occupation were noted from records. What type of study is being performed?
1. Retrospective cohort study
2. Prospective cohort study
3. Case-control study
4. Intervention and response

Correct Answer: Retrospective cohort study

Gynaecology & Obstetrics

Topic: Obstetrics

Q.30. A woman presents to you at 36 weeks of gestation with complaints of breathlessness and excessive abdominal distension. Fetal movements are normal. On examination, fetal parts are not easily felt and the fetal heartbeat is heard but it is muffled. Her symphysis fundal height is 41 cm. Her abdomen is tense but not tender. What is the most likely diagnosis?
1. Abruptio placenta
2. Hydrocephalus of fetus
3. Polyhydramnios
4. Fetal-maternal ascites

Correct Answer: Polyhydramnios

Topic: Obstetrics

Q.31. A primigravida presents to you with anemia early in her pregnancy. She is 7 weeks pregnant as seen on ultrasound. Her hemoglobin level is 9 g/dL. When should the iron supplements be started for her?
1. 10 to 12 weeks
2. 8 to 10 weeks
3. After 14 weeks
4. After 20 weeks

Correct Answer: 8 to 10 weeks

Forensic Medicine

Topic: Forensic Toxicology

Q.32. A child before playing consumed fruit from the garden. After some time he developed a high fever, confusion, photophobia, and unable to urinate. What are the likely causative agent and the appropriate antidote used in this case?
1. Datura, Pralidoxime
2. Datura, Physostigmine
3. Yellow oleander, Pralidoxime
4. Yellow oleander, Physostigmine

Correct Answer: Datura, Physostigmine

Topic: Forensic Thanatology

Q.33. The method of autopsy carried out en masse to remove from tongue to prostate is.
1. Virchow technique
2. Rokitansky technique
3. Ghon technique
4. Letulle technique

Correct Answer: Letulle technique

Pediatrics

Topic: Fluid and Electrolyte Disturbances

Q.34. A 10-year-old child weighing 30 kg presents with a history of loose stools for 2 days. On examination, there is severe dehydration. Laboratory investigations are as follows. What is the initial management as per ISPAD guidelines?

RBS550 mg/dL
pH7.01
Na+158mEq/L
Urine glucose3+
  1. Manage ABC, NS 20 mL/kg and start insulin after 1 hour
  2. Manage ABC, NS 20 mL/kg along with insulin 0.1 IU/kg/hr
  3. Manage ABC, NS 10 mL/kg along with insulin 0.1 IU/kg/hr
  4. Manage ABC, NS 10 mL/kg and start insulin after 1 hour

Correct Answer: Manage ABC, NS 20 mL/kg and start insulin after 1 hour

Topic: Growth

Q.35. Which of the following is the best sign to indicate adequate growth in an infant with a birth weight of 2.8 kg?

  1. Increase in length of 25 centimetres in the first year
  2. Weight gain of 300 grams per month till 1 year
  3. Anterior fontanelle closure by 6 months of age
  4. Weight under the 75th percentile and height under the 25th percentile

Correct Answer: Increase in length of 25 centimetres in the first year.

Download complete

NEET PG 2022 Previous Year Questions

NEET PG 2022 was conducted on May 21, 2022, as a national-level entrance examination for postgraduate medical courses in India. It saw a substantial number of candidates vying for admission to various medical institutes across the country. Check out the NEET PG 2022 previous year questions below - 

Microbiology

Topic: Parasitology

Q.1. Microfilariae with sheathed tail and two nuclei at the tail is suggestive of?
1. Wuchereria bancrofti
2. Brugia malayi
3. Loa loa
4. Onchocerca volvulus

Correct Answer: Brugia malayi

Microbiology

Topic: Systemic Bacteriology

Q.2. A militant presents with rashes all over his body sparing the palms and soles. On examination, he was febrile and lice were noted. Which of the following is responsible for his condition?
1. Rickettsia typhi
2. Rickettsia prowazekii
3. Rickettsia akari
4. Rickettsia conorii

Correct answer: Rickettsia prowazekii

Anatomy

Topic: Lower Limb

Q.3. A patient underwent surgery for the varicose vein. He now complains of sensory loss over the medial aspect of the leg and foot. Which of the following nerves is most likely to be injured?
1. Sural nerve
2. Superficial peroneal nerve
3. Deep peroneal nerve
4. Saphenous nerve

Correct Answer: Saphenous nerve

Biochemistry

Topic: Carbohydrate Metabolism

Q.4. A patient from a Mediterranean country visits Africa, where he develops malaria. He is treated with primaquine and later develops hemolytic anemia. Deficiency of an enzyme involved in which of the following pathways could be the cause?
1. Glycolysis
2. Gluconeogenesis
3. Hexose monophosphate (HMP) pathway
4. Luebering-Rapoport pathway

Correct Answer: Hexose monophosphate (HMP) pathway

Biochemistry

Topic: Genetics

Q.5. Which of the following disorders follows autosomal recessive inheritance pattern?
1. Huntington’s disease
2. Treacher Collins syndrome
3. Cystic fibrosis
4. Achondroplasia

Correct Answer: Cystic fibrosis

Ophthalmology

Topic: Uveitis

Q.6. A patient presents with a history of penetrating injury to the eye. A diagnosis of sympathetic ophthalmitis was confirmed. Which of the following will be seen?
1. Acute anterior uveitis
2. Pars planitis
3. Panuveitis
4. Chronic anterior uveitis

Correct Answer: Panuveitis

Topic: Glaucoma

Q.7. A diabetic patient presents to you with visual acuity of 6/9 in one eye. Further investigations revealed preretinal hemorrhages with neovascularization at the optic disc. What is the next step in management?
1. Focal laser photocoagulation
2. Pan-retinal photocoagulation
3. Grid laser photocoagulation
4. Scleral buckling

Correct Answer: Pan-retinal photocoagulation

Surgery

Topic: Endocrine Surgery

Q.8. A 45-year-old female patient underwent a thyroidectomy. Three days after the surgery, she developed perioral numbness. Which of the following investigations need to be done for her?
1. Free T3, T4
2. T3, T4, thyroid–stimulating hormone
3. Radioiodine scan
4. Calcium, phosphate, and parathormone levels

Correct Answer: Calcium, phosphate, and parathormone levels

Topic: Cardiothoracic Vascular Surgery

Q.9. Which of the following is most likely to be seen due to the rupture of a saccular aneurysm?
1. Subdural haemorrhage
2. Subarachnoid haemorrhage
3. Intracerebral haemorrhage
4. Hydrocephalus

Correct Answer: Subarachnoid haemorrhage

Dermatology

Topic: Sexually Transmitted Infections

Q.10. A patient presents to you with multiple anogenital warts. The biopsy of these lesions showed squamous atypia. Which of the following human papillomavirus types are considered high-risk?
1. HPV 2
2. HPV 18
3. HPV 6
4. HPV 11

Correct Answer: HPV 18

Topic: Cutaneous Infections

Q.11. A farmer presents you with a cauliflower-shaped mass on foot, which developed after a minor injury. Microscopy shows copper penny bodies. What is the most likely diagnosis?
1. Chromoblastomycosis
2. Blastomycosis
3. Sporotrichosis
4. Phaeohyphomycosis

Correct Answer: Chromoblastomycosis

Pharmacology

Topic: Respiratory Pharmacology

Q.12. A male patient with chronic obstructive pulmonary disease (COPD) was prescribed theophylline. He noticed that his urine output had increased the following day. This action of the drug is mediated through which of the following receptors?
1. Interleukin – 10
2. Histone deacetylase
3. Adenosine A1
4. Beta 2 adrenergic receptors

Correct Answer: Adenosine A1

Topic: Endocrine Pharmacology

Q.13. Which of the following drugs is not likely to cause Pulmonary fibrosis?
1. Metformin
2. Methotrexate
3. Bleomycin
4. Nitrofurantoin

Correct Answer: Metformin

Orthopaedics

Topic: Metabolic Disorders

Q.14. An intrauterine scan at the 13th week of pregnancy showed a fetus with multiple long bone fractures. What is commonly associated with this finding?
1. Achondroplasia
2. Osteogenesis imperfecta
3. Cretinism
4. Marfan syndrome

Correct Answer: Osteogenesis imperfecta

Topic: Spine + Pelvis + Lower Limb Traumatology

Q.15. A male patient presented with a bone fracture following a road traffic accident. After 2 days he developed dyspnea, petechiae involving the whole body, and a fall in oxygen saturation. What is the likely diagnosis?
1. Fat embolism
2. Air embolism
3. Venous thromboembolism
4. Pulmonary hypertension

Correct Answer: Fat embolism

Psychiatry

Topic: Eating Disorders

Q.16. A 16-year-old girl has intense cravings for food. She eats large amounts of food, which is followed by self-induced vomiting. What is the probable diagnosis?

  1. Anorexia nervosa
  2. Bulimia nervosa
  3. Atypical depression
  4. Binge eating disorder

Correct Answer: Bulimia nervosa

Topic: Mood Disorders

Q.17. A woman, who is 4 days postpartum, presented with tearfulness, mood swings, and occasional insomnia. What is the likely diagnosis?

  1. Postpartum depression
  2. Postpartum blues
  3. Postpartum psychosis
  4. Postpartum anxiety

Correct Answer: Postpartum blues

Physiology

Topic: Nerve Muscle Physiology

Q.18. A person after sleeping overnight with the arm under his head now experiences paresis but no numbness in the morning. Which of the following is the best explanation for it?

  1. C fibres are more sensitive to pressure than A fibres
  2. A fibres are more sensitive to hypoxia than B fibres
  3. A fibres are more susceptible to pressure changes than C fibres
  4. A fibres are more susceptible to hypoxia than C fibres

Correct Answer: A fibres are more susceptible to pressure changes than C fibres

Topic: The Nervous System

Q.19. A 65-year-old suffered from a stroke 2 days ago. He now presents with involuntary, violent, and flinging movements of the limbs on one side. What is the likely site of lesion in this patient?

  1. Subthalamic nuclei
  2. Globus pallidus
  3. Putamen
  4. Caudate nucleus

Correct Answer: Subthalamic nuclei

Medicine 

Topic: Neurology

Q.20. A female patient presents to you with a unilateral headache. It is associated with nausea, photophobia, and phonophobia. What is the drug of choice for acute management?

  1. Flunarizine
  2. Sumatriptan
  3. Propranolol
  4. Topiramate

Correct Answer: Sumatriptan

Topic: Rheumatology / Connective Tissue Disorder

Q.21. A patient presents to you with fever, night sweats, ptosis, and bilateral facial nerve palsy. Investigations showed leukocytosis and bilateral hilar lymphadenopathy. Which of the following is the most likely diagnosis?

  1. Sarcoidosis
  2. Tuberculosis
  3. Lymphoma
  4. Hypersensitive pneumonitis

Correct Answer: Sarcoidosis

Pediatrics

Topic: Neonatology

Q.22. A 1 day- old neonate has not passed urine since birth. What is the next step in management?

  1. Continue breast feeding not observed
  2. Admit to NICU’
  3. Start artificial feeding
  4. Start intravenous fluids

Correct Answer: Continue breast feeding not observed

Topic: Fluid and Electrolyte Disturbances

Q.23. A 7 – year old boy presented with abdominal pain, vomiting, oliguria, and periorbital puffiness following chemotherapy. Investigations reveal hyperuricemia, raised creatinine levels, and hyperkalemia. What is the next best step in the management of this condition ?

  1. Hydration
  2. Probenecid
  3. Allopurinol
  4. Rasburicase

Correct Answer: Hydration

PSM

Topic: Concept of Health and Disease

Q.24. The average life expectancy for a woman in Japan is 87 years. Due to recent advances in testing for cervical cancer, there is an increase in life expectancy by 15 years. The healthcare utility value is 0.8. Which of the following can be calculated from the parameters given?

  1. HALE
  2. DALY
  3. DFLE
  4. QALY

Correct Answer: QALY

Topic: Communicable and Non-communicable Diseases

Q.25. Although many animals are implicated in the spread of rabies, dogs are the most common ones. Also, it usually affects children in developing countries. Knowing this, what is the most cost-effective and logical way to reduce the incidence of rabies?

  1. Testing all dogs for rabies
  2. Reduce stray dog population and vaccinate all dogs
  3. Increase the laboratory facilities
  4. Increase capacity of healthcare workers for surveillance

Correct Answer:  Reduce stray dog population and vaccinate all dogs

Gynaecology & Obstetrics

Topic: Obstetrics

Q.26. A woman at 26 weeks of gestation presents for routine evaluation. On examination, fundal height corresponds to 24 weeks. Ultrasonography revealed decreased amniotic fluid. Which of the following conditions would have led to this presentation?

  1. Renal agenesis
  2. Tracheoesophageal fistula
  3. Cardiac abnormalities
  4. Ureteral stricture

Correct answer: Renal agenesis

Topic: Obstetrics

Q.27. A type 1 diabetic mother is on magnesium sulfate infusion post – cesarean section for preeclampsia. She develops delirium and is drowsy. She has a respiratory rate of 10/min, random blood glucose level of 240 mg / dL, oliguria, and bilaterally absent knee reflex. What is the cause of her condition?

  1. Magnesium sulfate toxicity
  2. Diabetic ketoacidosis
  3. Eclampsia
  4. Diabetes insipidus

Correct answer: Magnesium sulfate toxicity

Forensic Medicine

Topic: Court of Law 

Sub-Topic: 

Q.28. A 45-year-old female patient is told about the benefits and complications of a hysterectomy, and she agrees to the procedure. What kind of consent is this?

  1. Informed consent
  2. Implied consent
  3. Opt-out
  4. Passive consent

Correct answer: Informed consent

Topic: Asphyxial Deaths 

Q.29. A dead body is brought for evaluation. On post-mortem examination, a ligature completely encircled the neck, horizontal, and below the thyroid level was seen. There was no dribbling of saliva. What is the cause of death?

  1. Throttling
  2. Ligature strangulation
  3. Gagging
  4. Hanging

Correct answer: Ligature strangulation

Download complete

NEET PG 2021 Previous Year Questions

NEET PG 2021 was a significant national-level entrance examination conducted on September 11, 2021, for aspiring medical professionals seeking admission to postgraduate medical courses in India. Check out the NEET PG 2021 previous year questions below - 

Microbiology

Topic: Parasitology

Q.1. A patient from Uttar Pradesh presented with fever, pallor, and hepatosplenomegaly. Peripheral smear examination showed pancytopenia. Buffy coat examination showed macrophages laden with organisms with a kinetoplast. What is the vector for the likely disease?

  1. Sandfly
  2. Tse-Tse fly
  3. Triatomine bug
  4. Female anopheles mosquito

Correct Answer: Sandfly

Topic: Virology

Q.2. An unimmunized 2-year old child presented with coryza, conjunctivitis, and bluish-white spots in his buccal mucosa near the lower molar teeth. A day later, he developed a maculopapular rash on the face and neck. What is the nature of the causative virus?

ss- Single stranded

ds – Double – stranded

  1. Enveloped ss RNA
  2. Naked ss RNA
  3. Naked ds RNA
  4. Enveloped ds RNA

Correct Answer: Enveloped ss RNA

Radiology

Topic: Basics of Radiology 

Q.3. A patient following a skid resulting in a motor traffic accident was brought to the emergency room 2 hours later. On examination, he was stable with GCS 15/15. The pupil was reactive to light. Tenderness and bruising over the left lower chest wall with petechiae was seen. Severe tenderness was elicited in the left hypochondriac region and BP- 90/50 mm of Hg. What is the best investigation used in the ER?
GCS: Glasgow coma scale
FAST: Focused Assessment with Sonography for Trauma

  1. FAST
  2. X-ray
  3. Diagnostic peritoneal lavage
  4. CT scan

Correct Answer: FAST

Topic: Radiotherapy

Q.4. A child with acute lymphoblastic leukaemia undergoes prophylactic irradiation prior autologous hematopoietic stem cell transplantation. Which of the following will be the least affected?

  1. Spermatogonia
  2. Intestinal epithelial cells
  3. Neurons
  4. Bone marrow/ erythroid precursor cells

Correct Answer: Neurons 

Anatomy

Topic: Neuro Anatomy

Q.5. A patient presented with vision loss. On radiological investigation, an aneurysm causing damage to the optic chiasma was noted. Which of the following arteries is most likely to be the artery that is causing the damage?

  1. Anterior communicating artery
  2. Anterior choroidal artery
  3. Middle cerebral artery
  4. Anterior cerebral artery

Correct Answer: Anterior communicating artery

Topic: Upper Limb

Q.6. A 7-year-old boy was brought to the hospital with multiple fractures of the humerus secondary to a fall from height. On examination, there is difficulty in flexion of the elbow and supination of the forearm and associated loss of sensation over the lateral aspect of the forearm. Which is the nerve most likely to be injured?

  1. Median nerve
  2. Radial nerve
  3. Musculocutaneous nerve
  4. Ulnar Nerve

Correct Answer: Musculocutaneous nerve

Biochemistry

Topic: Lipid Chemistry

Q.7. A 5 year old boy presented with easy fatigability, irritability and inability to concentrate. Labs revealed the following findings a and b (light and oil immersion respectively) on the bone marrow aspiration. Which of the following is the most likely enzyme deficient in this condition?

  1. Hexosaminidase
  2. Glucocerebrosidase
  3. Sphingomyelinase
  4. N-acetylglucosaminidase

Correct Answer: Glucocerebrosidase

Topic: Oxidative Phosphorylation 

Q.8. You are conducting an experiment on mitochondrial respiration. You add malate/pyruvate and respiration is normal. You add succinate and respiration is normal. When you add another substance in the presence of pyruvate/succinate and malate, respiration is blocked. Which of the following substances is most likely added?

  1. Rotenone
  2. Antimycin A
  3. Oligomycin
  4. 2,4-dinitrophenol

Correct Answer: Oligomycin

Ophthalmology

Topic: Neuro Ophthalmology

Q.9. A 33-year-old woman presents with complaints of progressive loss of vision in the right halves of both eyes. Where is the lesion located in the optic pathway?

  1. Left optic tract
  2. Right visual cortex
  3. Optic chiasma
  4. Right optic nerve

Correct Answer: Left optic tract

Topic: Optics

Q.10. A 15-year-old girl is not compliant with spectacles for her myopic astigmatism. What would be the appropriate management in her case?

  1. LASIK
  2. Spherical equivalent spectacles
  3. Femto LASIK
  4. Implantable collamer lens

Correct Answer: Spherical equivalent spectacles

Surgery

Topic: Urology

Q.11. A 75-year-old man with prostate carcinoma presents to you with a PSA of 9 ng/mL with a small tumor focus. His Gleason score is 6. What will be your most likely management?
PSA- Prostate-specific antigen

  1. Radical prostatectomy
  2. External beam radiation
  3. Brachytherapy
  4. Active surveillance

Correct Answer: Active surveillance

Topic: Others

Q.12. A patient after a road traffic accident presented with pain in the abdomen. The resident examined the child and found that vitals were stable and tenderness was present in the left lumbar region. Which is the best investigation of choice?

  1. Contrast enhanced CT scan
  2. Retrograde urethrogram
  3. Wait and watch
  4. Emergency laparotomy

Correct Answer: Contrast enhanced CT scan

Dermatology

Topic: Immunobullous Disorders

Q.13. A 30-year-old presented with flaccid bullae on her skin which are easy to rupture. The biopsy revealed a suprabasal split. What is the most likely diagnosis?

  1. Pemphigus vulgaris 
  2. Pemphigus foliaceus
  3. Pemphigus vegetans
  4. Erythema multiforme

Correct Answer: Pemphigus vulgaris 

Topic: Bacterial Infections

Q.14. A patient taking multi-drug therapy (MDT) presents with worsening of existing lesions and nerve involvement. What will be your next best step of action?

  1. Stop MDT, start systemic corticosteroids
  2. Continue MDT, start systemic steroids
  3. Stop MDT thalidomide
  4. Continue MDT, start thalidomide

Correct Answer: Continue MDT, start systemic steroids

Anaesthesia

Topic: Monitoring in Anesthesia

Q.15. A 30-Year-old male patient was intubated for surgery by the final-year resident. Which of the following is the best method to confirm the position of the endotracheal tube?

  1. X-ray chest
  2. Auscultation
  3. End-tidal CO2 concentration
  4. Chest rise

Correct Answer: End-tidal CO2 concentration

Topic: Monitoring in Anesthesia

Q.16. A patient undergoing surgery is maintained on anesthesia with halothane. During the procedure, the patient suddenly developed hyperthermia and muscle rigidity. Which of the following drugs is most likely implicated in this condition?

  1. D-curare
  2. Suxamethonium
  3. Cis-atracurium
  4. Rocuronium

Correct Answer: Suxamethonium

ENT

Topic: Larynx

Q.17. Following total thyroidectomy, a patient started having difficulty in breathing, and repeated attempts to extubate were unsuccessful. The most probable cause is _________

  1. Superior laryngeal nerve injury
  2. Unilateral recurrent laryngeal nerve injury
  3. Bilateral recurrent laryngeal nerve injury
  4. Hematoma

Correct Answer: Bilateral recurrent laryngeal nerve injury

Topic: Nose and Paranasal Sinuses

Q.18. A female patient presents with nasal obstruction, nasal discharge, and loss of smell. On examination, foul-smelling discharge and yellowish – green crusts are present in the nasal cavity. She is found to have merciful anosmia. Which of the following findings can also be seen during the examination of her nose?

  1. Roomy nasal cavity
  2. Nasal polyps
  3. Inferior turbinate hypertrophy
  4. Foreign body

Correct Answer: Roomy nasal cavity

Pharmacology

Topic: CVS Pharmacology

Q.19. A patient with renal insufficiency presented with very low urine output, pedal edema, headache and BP 160/90. Which anti-hypertensive can be administered?

  1. Aliskiren
  2. Chlorthalidone
  3. Amlodipine
  4. Prazosin

Correct Answer: Amlodipine

Topic: Anticancer Drugs 

Q.20. Which of the following interleukin (IL) is inhibited by tocilizumab?

  1. IL-6
  2. IL-2
  3. IL-4
  4. IL-5

Correct Answer: IL-6

Orthopaedics

Topic: Spine + Pelvis + Lower Limb Traumatology

Q.21. A 20-year-old male patient presented with a history of lower backache and early morning stiffness for two years. He also gave a history of bilateral heel pain for 6 months. Which of the following is the most likely diagnosis?

  1. Ankylosing spondylitis
  2. Tuberculosis of the spine
  3. Disc prolapse
  4. Mechanical pain

Correct Answer: Ankylosing spondylitis

Topic: Spine + Pelvis + Lower Limb Traumatology

Q.22. A patient was brought to the hospital with complaints of pain around the left hip joint following a road traffic accident. On examination, the affected limb was flexed, adducted, and medially rotated with obvious shortening. What is the most likely diagnosis?

  1. Anterior hip dislocation
  2. Posterior hip dislocation
  3. Transcervical fracture
  4. Intertrochanteric fracture

Correct Answer: Posterior hip dislocation

Psychiatry

Topic: Psychology 

Q.23. During psychotherapy, the therapist had mixed conscious and unconscious feelings toward his patient. This is known as ___________.

  1. Countertransference
  2. Transference
  3. Dissociation
  4. Preoccupation

Correct Answer: Countertransference

Topic: Substance-Related and Addictive Disorders

Q.24. A patient stopped alcohol consumption for 3 days and presented with irritability, disorientation, paranoid delusions, agitation, visual hallucinations, and altered sensorium. What is the likely diagnosis in this case?

  1. Delirium tremens
  2. Wernicke’s encephalopathy
  3. Korsakoff psychosis
  4. Alcohol-induced psychosis

Correct Answer: Delirium tremens

Physiology

Topic: General Physiology

Q.25. A 30-year-old man weighing 70 kg had a sodium level of 120 mEq/L. Calculate the sodium deficit.

  1. 280 mEq
  2. 480 mEq
  3. 840 mEq
  4. 1400 mEq

Correct Answer: 840 mEq

Topic: General Physiology

Q.26. A body fluid sample is being studied with Na: 10 mEq/L K: 140 mEq/L, Cl: 4 mEq/L. Identify the compartment from which the piece has been obtained.

  1. Interstitial
  2. Intracellular fluid
  3. Extracellular fluid
  4. Plasma

Correct Answer: Intracellular fluid

Medicine 

Topic: Neurology

Q.27. A 37-year-old woman presents with headaches for 6 months. She has been taking analgesics regularly. The headache recently increased in severity for 3 days but was reduced in stopping the analgesic. What is the likely diagnosis?

  1. Medication overuse headache
  2. Tension headache
  3. Chronic migraine
  4. Cluster headache

Correct Answer: Medication overuse headache

Topic: Cardiology

Q.28. A 20-year-old woman presents with breathlessness and chest pain. She is a known case of mitral stenosis. Her pulse is irregularly irregular. No thrombus is seen on echocardiography. What is the best agent to prevent future thrombotic events?

  1. Dabigatran
  2. Aspirin - 150 mg
  3. Oral warfarin
  4. Aspirin + Clopidogrel

Correct Answer: Oral warfarin

Pediatrics

Topic: Paediatric Cardiology

Q.29. In foetal circulation, which vessel carries deoxygenated blood back to the placenta?

  1. Umbilical Vein
  2. Umbilical Artery
  3. Descending aorta
  4. Pulmonary Artery

Correct Answer: Umbilical artery

Topic: Paediatric Neurology

Q.30. A 10-year-old boy presented with seizures. His past history is significant for an episode of fever with rash at 1 year of age which resolved spontaneously. What is the most helpful investigation to diagnose his condition?

  1. lgG measles in CSF
  2. MRI mesial temporal lobe sclerosis
  3. lgM measles in CSF
  4. C1Q4 antibodies in the CSF

Correct Answer: lgG measles in CSF

PSM 

Topic: Environment and Health

Q.31. Which of the following thermometers is used to measure the low air velocity rather than the cooling power of the air?

  1. Kata thermometer
  2. Globe thermometer
  3. Wet thermometer
  4. Dial thermometer

Correct Answer: Kata thermometer

Topic: Environment and Health

Q.32. There is an outbreak of buboes in the community. What is the vector responsible for this condition?

  1. Xenopsylla cheopis
  2. Phlebotomus argentipes
  3. Ixodes tick
  4. Female Anopheles mosquito

Correct Answer: Xenopsylla cheopis

Gynaecology & Obstetrics

Topic: Gynecology

Q.33. A 27 year old female patient was found to have uterus didelphys. Which of the following is not likely to be a complication to this uterine anomaly?

  1. Abortion
  2. Endometriosis
  3. Preterm labour
  4. Transverse lie 

Correct Answer: Transverse lie 

Topic: Obstetrics

Q.34. A pregnant woman at 36 weeks of gestation with a prosthetic valve replacement for mitral stenosis is on warfarin therapy. Her INR is 3. What is the appropriate next step in management ?

LMWH- Low Molecular Weight Heparin

  1. Stop warfarin and start LMWH
  2. Stop warfarin and start heparin
  3. Continue warfarin
  4. Stop warfarin, start LMWH and aspirin

Correct Answer: Stop warfarin and start LMWH

Forensic Medicine

Topic: Forensic Toxicology

Q.35. A guy was playing in a garden. Suddenly, he collapsed and was rushed to the hospital. His friend who played with him in the garden informed the doctors that he ate one of the fruits in the garden post which he seemed to have developed these symptoms. He also had irritability, restlessness, dry hot skin, and was unable to pass urine and stools. Identify the poison and its appropriate antidote.

  1. Datura, Pralidoxime
  2. Datura, Physostigmine
  3. Yellow oleander, Digoxin
  4. Yellow oleander, Physostigmine

Correct Answer: Datura, Physostigmine

Topic: Forensic Thanatology 

Q.36. A man was working in a field and he suddenly collapsed on a hot summer afternoon. On examination, there were no physical signs of dehydration. His serum electrolyte level was normal. The doctor found that the body temperature was 106 degrees Fahrenheit. Which of the following symptoms is least likely to be seen in this patient?

  1. Hot skin
  2. Hypotension
  3. Sweating 
  4. Disorientation

Correct Answer: Sweating

Download complete

NEET PG 2020 Previous Year Questions

NEET PG 2020 was held on January 5, 2020, for medical graduates seeking admission to postgraduate medical courses in India. Check out the NEET PG 2020 previous year questions below- 

Microbiology

Topic: Mycology

Q.1. A 35-year-old HIV-positive patient presents with mucosal lesions in the mouth as shown in the image below. On microscopy , bugging yeasts and pseudohyphae are seen. What is the likely diagnosis?

  1. Oral candidiasis
  2. Hairy leukoplakia 
  3. Lichen planus
  4. Diphtheria

Correct Answer: Oral candidiasis

Topic: Virology

Q.2. Zika virus is transmitted by:

  1. Aedes aegypti
  2. Culex
  3. Anopheles
  4. Phlebotomus papatasii

Correct Answer: Aedes aegypti

Radiology

Topic: Head & Neck Imaging 

Q.3. A cystic lesion in the suprasellar region with calcification is seen on the MRI. What is the most likely diagnosis?

  1. Craniopharyngioma
  2. Pituitary adenoma
  3. Meningioma
  4. Oligodendroglioma

Correct Answer: Craniopharyngioma

Topic: OBGyn Imaging

Q.4. Personal monitoring of radiation is done by:

  1. TLD badge
  2. Collimators
  3. Linear accelerator
  4. Grid

Correct Answer: TLD badge

Anatomy

Topic: Back Region

Q.5. The muscle marked by the arrow in the image below is innervated by the :

  1. Dorsal scapular nerve
  2. Suprascapular nerve
  3. From the dorsal rami of C1
  4. Subscapular nerve

Correct Answer: Dorsal scapular nerve

Topic: Upper Limb

Q.6. Which of the following forms the lateral boundary of the anatomical snuff box?

  1. Extensor pollicis brevis and abductor pollicis longus
  2. Extensor pollicis longus and abductor pollicis brevis
  3. Extensor pollicis longus and extensor pollicis brevis
  4. Abductor pollicis longus and abductor pollicis brevis

Correct Answer: Extensor pollicis brevis and abductor pollicis longus

Biochemistry

Topic: Introduction

Q.7. What is the function of the proteasome?

  1. Protein folding
  2. Pot-translation modification
  3. Protein degradation
  4. Protein sorting

Correct Answer: Protein degradation

Topic: Amino Acids and Proteins Chemistry

Q.8. Replacing alanine by which amino acid, will increase the UV absorbance of protein at 280 nm wavelength?

  1. Leucine
  2. Proline
  3. Arginine
  4. Tryptophan

Correct Answer: Tryptophan

Ophthalmology

Topic: Cornea

Q.9. Which of the following layers is responsible for maintaining the hydration and transparency of the cornea?

  1. Descemet’s membrane
  2. Endothelial cells
  3. Stroma
  4. Corneal epithelium

Correct Answer: Endothelial cells

Topic: Cornea

Q.10. What is the characteristic feature of a fungal ulcer?

  1. Reverse hypopyon
  2. Dendritic ulcer on a fluorescein dye
  3. Ring abscess
  4. Satellite lesion

Correct Answer: Satellite lesion

Surgery

Topic: Endocrine Surgery

Q.11. What is the most common site of gastrinoma in MEN 1 syndrome?

  1. Jejunum
  2. Ileum
  3. Duodenum
  4. Stomach

Correct Answer: Duodenum

Topic: Cardiothoracic Vascular Surgery

Q.12. A 50-year-old male who is a chronic smoker presented to the hospital with intermittent claudication pain of both thigh and buttock region on walking about 500 m. Which of the following is a likely diagnosis?

  1. Arterial disease involving the superficial femoral artery
  2. Arterial disease with aortoiliac involvement
  3. Femoral venous insufficiency
  4. Arterial disease involving the profunda femoris artery

Correct Answer: Arterial disease with aortoiliac involvement

Dermatology

Topic: Miscellaneous Disorders

Q.13. Which of the following drugs used as nail lacquer belongs to morpholines?

  1. Amorolfine
  2. Oxiconazole
  3. Ciclopirox olamine
  4. Tioconazole

Correct Answer: Amorolfine

Topic: Skin Appendages and their Disorders

Q.14. A child was born with membranes around the body and had ectropion and eclabium. He is brought to the OPD with lesions covering his face, trunk, and extremities. Which of the following is an unlikely diagnosis?

  1. Icthyosis vulgaris
  2. Lamellar ichthyosis
  3. Bathing suit ichthyosis
  4. Harlequin ichthyosis

Correct Answer: Icthyosis vulgaris

Anaesthesia

Topic: Pre-Anaesthetic Evaluation

Q.15. Which of the following investigations provides the most accurate prognostic information with respect to predicting risk of perioperative cardiac complications?

  1. Exercise ECG testing
  2. Dobutamine stress echocardiography
  3. Myocardial perfusion scintigraphy
  4. Coronary angiography

Correct Answer: Dobutamine stress echocardiography

Topic: Neuromuscular Blockade

Q.16. A patient is given a nicotinic receptor antagonist as a muscle relaxant. Which drug is given postoperatively to recover from muscle weakness?

  1. Physostigmine
  2. Neostigmine
  3. Carbachol
  4. Succinylcholine

Correct Answer: Neostigmine

ENT

Topic: Ear

Q.17. From which of the following structures does the saccule develop?

  1. Saculus anterior
  2. Saculus posterior
  3. Pars superior
  4. Pars inferior

Correct Answer: Pars inferior

Topic: Ear

Q.18. What is the surgery done to widen the cartilaginous part of the external auditory canal called?

  1. Meatoplasty
  2. Tympanoplasty
  3. Myringoplasty
  4. Otoplasty

Correct Answer: Meatoplasty

Pharmacology

Topic: Anticancer Drugs 

Q.19. Which of the following is an inhibitor of DNA synthesis?

  1. 6-mercaptopurine
  2. Mitomycin
  3. Actinomycin
  4. Asparaginase

Correct Answer: 6-mercaptopurine

Topic: Renal Pharmacology

Q.20. A patient on lithium developed hypertension. He was started on thiazides for hypertension. After a few days, he developed coarse tremors and other symptoms suggestive of lithium toxicity. Explain the likely mechanism of this interaction.

  1. Thiazide inhibits the metabolism of lithium
  2. Thiazides act as an add on drug to lithium
  3. Thiazides increase the tubular resorption of lithium
  4. Thiazides cause water loss thereby increase lithium levels

Correct Answer: Thiazides increase the tubular resorption of lithium

Orthopaedics

Topic: Arthritis

Q.21. A patient presented with pain in the hand. The joints involved were proximal interphalangeal joint, distal interphalangeal joint and first carpometacarpal joint. The wrist and metacarpophalangeal joints were spared. What is the likely diagnosis?

  1. Osteoarthritis
  2. Rheumatoid arthritis
  3. Psoriatic arthritis
  4. Pseudogout

Correct Answer: Osteoarthritis

Topic: Sports Injuries

Q.22. A maid is playing with a child by spinning him while holding his hands. A few hours later, the child starts crying, does not use his arm, and does not let anybody touch him. What is the possible diagnosis?

  1. Pulled elbow
  2. Olecranon fracture
  3. Fracture head of radius
  4. Elbow dislocation
    Correct Answer: Pulled elbow

Psychiatry

Topic: Substance-Related and Addictive Disorders

Q.23. A 25-year-old male presented to the clinic with a complaint of the feeling of insects crawling under his skin. Which of the following drug abuse can cause the symptoms of this patient?

  1. Cannabis
  2. Cocaine
  3. Amphetamine
  4. Alcohol

Correct Answer: Cocaine

Topic: Mood Disorders

Q.24. A 60-year-old male who lost his wife 3 months back complains that his intestines have become rotten. He feels responsible for his wife’s death and should be sent to prison. He complains of feeling low all the time and has lost interest in daily activities since his wife’s death. What is the most likely diagnosis?

  1. Normal grief reaction
  2. Psychotic depression
  3. Delusional disorder
  4. Schizophrenia

Correct Answer: Psychotic depression

Physiology

Topic: Respiratory System

Q.25. A person develops headache and breathlessness on a trekking expedition following a rapid ascent to over 3000 meters above the sea level. Which of the following is not used in the treatment of this condition?

  1. IV digoxin
  2. Immediate descent
  3. Administration of oxygen
  4. Tablet acetazolamide

Correct Answer: IV digoxin

Topic: Endocrine and Reproductive System

Sub-Topic: 

Q.26. Prolactin level is highest during?

  1. 24 hours after ovulation
  2. 24 hours after delivery
  3. REM sleep
  4. After running four 1 hours

Correct Answer: 24 hours after delivery

Medicine 

Topic: Nephrology / Kidney Disease

Q.27. Which of the following is associated with pauci-immune glomerulonephritis?

  1. SLE nephritis
  2. Anti-GBM glomerulonephritis
  3. IgA nephropathy
  4. Granulomatosis with polyangiitis (GPA)

Correct Answer: Granulomatosis with polyangiitis (GPA)

Topic: Emergency Medicine

Q.28. At a high altitude of 3000m, a person complains of breathlessness. All of the following can be used for management of this person except

  1. Intravenous digoxin
  2. Oxygen supplementation
  3. Immediate descent
  4. Acetazolamide

Correct Answer: Intravenous digoxin

Pediatrics

Topic: Musculoskeletal Disorders in Children

Q.29. A 4-yr old male child presents with muscle weakness. His mother says that her child has difficulty climbing stairs and getting up from the floor. On muscle biopsy, small muscle fibrils and absence of dystrophin was found. What is the diagnosis out of given options?

  1. Becker's muscle dystrophy
  2. Duchenne muscular dystrophy
  3. Myotonic dystrophy
  4. Limb-girdle muscular dystrophy

Correct Answer: Duchenne muscular dystrophy

Topic: Paediatric Neurology

Q.30. Two girls in the same class are diagnosed with meningococcal meningitis. Their 12-year old friend from the same school is afraid of contracting the disease. What advice should be given to the exposed students?

  1. Two doses of polysaccharide vaccine
  2. Antibiotic prophylaxis
  3. Two doses of conjugate vaccine
  4. Single dose of meningococcal vaccine

Correct Answer: Antibiotic prophylaxis

PSM 

Topic: Health Planning and Management

Q.31. The monetary benefit is measured in which of the following analyses?

  1. Cost-effective analysis
  2. Cost-benefit analysis
  3. Network analysis
  4. Input-Output analysis

Correct Answer: Cost-benefit analysis

Topic: Allied Health Disciplines

Q.32. An employee diagnosed with TB gets extended sickness benefit for?

  1. 1 year
  2. 2 years
  3. 3 years
  4. 4 years

Correct Answer: 2 years

Gynaecology & Obstetrics

Topic: Obstetrics

Q.33. 24 yr old women who had home delivery 2 weeks back, now present with complete perineal tear. What is the next line of management?

  1. Repair immediately
  2. Repair after 3 weeks
  3. Repair after 6 months
  4. Repair after 3 months

Correct Answer: Repair after 3 months

Topic: Gynecology

Q.34. The remnants of Wolffian ducts in females are located in

  1. Pouch of Douglas
  2. Leaves of broad ligament
  3. Uterovesical pouch
  4. Iliac fossa

Correct Answer: Leaves of broad ligament

Forensic Medicine

Topic: Forensic Identification

Q.35. A deceased male was brought to the morgue for a post-mortem examination. His relatives reported that he had a tattoo over his skin, which was not found on examination. Which of the following structures would you examine?

  1. Lymph nodes
  2. Liver
  3. Arteries
  4. Skin

Correct Answer: Lymph nodes

Topic: Forensic Toxicology

Q.36. The most common drug which causes physical dependence is?

  1. Ketamine
  2. Heroin
  3. LSD
  4. Phencyclidine

Correct Answer: Heroin

Download complete

NEET PG 2019 Previous Year Questions

NEET PG 2019 was conducted on 6th January, 2019 with huge participation. Check out the previous year questions below - 

Microbiology

Topic: Systemic Bacteriology

Q.1. BCYE medium is used to culture:

  1. Leptospira
  2. Bacillus cereus
  3. Campylobacter
  4. Legionella

Correct Answer: Legionella

Topic: Systemic Bacteriology

Q.2. A 16-year-old boy is admitted with fever, icterus, conjunctival suffusion, and hematuria for 15 days. Which serological test should be done for diagnosis?

  1. Widal test
  2. Microscopic agglutination test
  3. Weil felix reaction
  4. Paul Bunnell test

Correct Answer: Microscopic agglutination test

Radiology

Topic: Basics of Radiology 

Q.3. What is the best imaging study for the earliest diagnosis of cerebral infarct?

  1. Flair MRI
  2. Non-contrast CT
  3. CECT
  4. Diffusion weighted MRI

Correct Answer: Diffusion weighted MRI

Topic: Basics of Radiology 

Q.4. Which of the following is a water soluble contrast?

  1. Iohexol
  2. Barium 
  3. Borium 
  4. Calcium

Correct Answer: Iohexol

Anatomy

Topic: Back Region

Q.5. The movements at the _______ joint permit a person to look to the right and left.

  1. Atlanto-occipital
  2. Atlanto -axial
  3. C2-C3
  4. C3-C4

Correct Answer: Atlanto -axial

Topic: Abdomen

Q.6. What is the basis for the formation of omphaloceles?

  1. Excessive length of the intestine
  2. Physiological hernia failing to go back
  3. Herniation of the liver
  4. Herniation of umbilicus

Correct Answer: Physiological hernia failing to go back

Biochemistry

Topic: Lipid Metabolism

Q.7. An infant present with hypotonia and seizures. It was confirmed to be cerebrohepatorenal syndrome. Which of the following is accumulated in the brain in cerebrohepatorenal syndrome?

  1. Glucose
  2. Long chain fatty acid
  3. Lactic Acid
  4. Triglycerides

Correct Answer: Long Chain Fatty Acid

Topic: Genetics

Q.8. Which of the following is autosomal dominant?

  1. Cystic fibrosis
  2. Duchene muscular dystrophy
  3. Sickle cell anaemia
  4. Achondroplasia

Correct Answer: Achondroplasia

Ophthalmology

Topic: Optics

Q.9. Which of the following would be prescribed for simple myopic astigmatism?

  1. +1.00 DS
  2. -1.00 DC × 180 Degree
  3. -1.00 DS
  4. -1.00DS – 1.00 DC × 180 Degree

Correct Answer: -1.00 DC × 180 Degree

Topic: Retina

Q.10. Not true about retinitis pigmentosa is:

  1. Retinal pigmentation
  2. Pale waxy disc
  3. Narrowing of vessels
  4. ERG - Normal

Correct Answer: ERG- Normal

Surgery

Topic: Urology

Q.11. Which of the following drugs cause carcinoma bladder?

  1. Cyclophosphamide
  2. Cisplatin
  3. Taxane
  4. Tamoxifen

Correct Answer: Cyclophosphamide

Topic: Gastrointestinal Surgery

Q.12. Dohlman's procedure is for:

  1. Meckel's diverticulum
  2. Zenker’s diverticulum
  3. Bochadlek hernia
  4. Menetrier's disease

Correct Answer: Zenker’s diverticulum

Dermatology

Topic: Eczema

Q.13. Bindi Leukoderma is caused by which chemical?

  1. A Mono-benzyl ether of Hydroquinone (MBH)
  2. Crocein Scarlet MOO and Solvent Yellow 3
  3. p-phenylenediamine (PPD)
  4. Para Tertiary butylphenol (PTBP)

Correct Answer: Para Tertiary butylphenol (PTBP)

Topic: Sexually Transmitted Infections

Q.14. A young female presented with vaginal itching and green frothy genital discharge. Strawberry vagina is seen on examination. What will be the drug of choice?

  1. Doxycycline
  2. Oral fluconazole
  3. Metronidazole
  4. Azithromycin

Correct Answer: Metronidazole

Anaesthesia

Topic: Neuromuscular Blockade

Q.15. What is the mechanism of action of the curare group of muscle relaxants?

  1. Persistently depolarizing at neuromuscular junction
  2. Competitively blocking the binding of ACh to its receptors
  3. Repetitive stimulation of ACh receptors on muscle endplate
  4. Inhibiting the calcium channels on presynaptic membrane

Correct Answer: Competitively blocking the binding of ACh to its receptors

Topic: Inhalational Anaesthetic Agents

Q.16. Which of the following is not used for induction of anaesthesia in paediatric patients?

  1. Halothane
  2. Sevoflurane
  3. Desflurane
  4. Nitrous oxide

Correct Answer: Desflurane

ENT

Topic: Ear

Q.17. Which of the following is not a feature of tubercular otitis media?

  1. Ear ache
  2. Multiple perforations
  3. Pale granulation
  4. Foul smelling ear discharge

Correct Answer: Ear ache

Topic: Nose and Paranasal Sinuses

Q.18. Pott’s puffy tumor is

  1. Subperiosteal abscess of frontal bone
  2. Subperiosteal abscess of ethmoid bone
  3. Mucocele of frontal bone
  4. Mucocele of ethmoid bone

Correct Answer: Subperiosteal abscess of frontal bone

Pharmacology

Topic: Drugs Affecting Blood and Blood formation

Q.19. Which of the following is an oral factor Xa inhibitor?

  1. Bivalirudin
  2. Dabigatran
  3. Rivaroxaban
  4. Enoxaparin

Correct Answer: Rivaroxaban

Topic: Antimicrobials : Antibacterial Drugs

Q.20. What is the mechanism of action of oseltamivir and zanamivir?

  1. DNA polymerase inhibition
  2. Protein synthesis inhibition
  3. Nucleotide analogue
  4. Neuraminidase inhibition

Correct Answer: Neuraminidase inhibition

Orthopaedics

Topic: General + Upper Limb Traumatology

Q.21. Which of the following fractures is most prone to non-union?

  1. Proximal scaphoid
  2. Intertrochanteric
  3. Distal radius
  4. Talar neck

Correct Answer: Proximal scaphoid

Topic: Miscellaneous

Q.22. Degloving injury refers to_____?

  1. Skin and subcutaneous fat are stripped from the underlying fascia
  2. Skin, subcutaneous fat and fascia are stripped from tendons
  3. Skin, subcutaneous fat, fascia and tendons are stripped from bone
  4. Only skin is stripped off

Correct Answer: Skin and subcutaneous fat are stripped from the underlying fascia

Psychiatry

Topic: Substance-Related and Addictive Disorders

Q.23. Which of the following drugs is used as an antismoking agent?

  1. Busulfan
  2. Acamprosate
  3. Varenicline
  4. Gabapentin

Correct Answer: Varenicline

Topic: Schizophrenia Spectrum and Other Psychotic Disorders

Q.24. Which of the following is not a risk factor for delusional disorder?

  1. Recent immigration
  2. Social isolation 
  3. Family history
  4. Young age

Correct Answer: Young age

Physiology

Topic: Excretory System

Q.25. According to myogenic hypothesis of renal autoregulation the afferent arterioles contract in response to stretch – induced by ____________

  1. NO
  2. Noradrenaline
  3. Opening of calcium channels
  4. Adenosine

Correct Answer: Opening of calcium channels

Topic: The Nervous System

Q.26. Which of the following is true for decorticate rigidity?

  1. It is produced by the removal of cerebral cortex and basal ganglia
  2. Flexion of lower limbs and extension of upper limbs occurs
  3. Rigidity is pronounced
  4. It is characterised by flexion of upper limbs and extension of lower limbs

Correct Answer: It is characterised by flexion of upper limbs and extension of lower limbs

Medicine 

Topic: Pulmonology

Q.27. Nasal polyps are commonly associated with: -

  1. Intrinsic Asthma
  2. Brittle Asthma
  3. Extrinsic Asthma
  4. Exercise induced asthma

Correct Answer: Intrinsic Asthma

Topic: Haematology

Q.28. A 47 year old man with a diagnosis of acute myeloid leukaemia with a blood type O negative blood group presents to the transplant clinic to discuss proceeding with an allogeneic stem cell transplant. Which of the following would be an optimal donor?

  1. His identical twin brother
  2. Umbilical cord transplant
  3. His HHLA identical 50-year-old brother who is otherwise healthy and is blood type O+
  4. An HLA identical matched unrelated donor who is blood type

Correct Answer: His HHLA identical 50-year-old brother who is otherwise healthy and is blood type O+

Pediatrics

Topic: Paediatric Gastroenterology

Q.29. Unconjugated hyperbilirubinemia, which did not subside even after 3 weeks of birth, was observed in a neonate. On investigating, liver enzymes, PT/INR and albumin levels were normal. No hemolysis was seen on a peripheral blood smear. A drop in bilirubin level was observed within a week after treatment with phenobarbital. What is the most likely diagnosis?

  1. Rotor syndrome
  2. Crigler Najjar type 2
  3. Dubin Johnson syndrome
  4. Crigler Najjar type 1

Correct Answer: Crigler Najjar type 2

Topic: Pediatric Respiratory Disorders

Q.30. Which of the following is not seen in a child with cystic fibrosis?

  1. Sweat chloride test chloride conc of 70mEq/L
  2. Increase immunoreactive trypsinogen level
  3. Hyperkalemia
  4. Contraction alkalosis

Correct Answer: Hyperkalemia

PSM 

Topic: Concept of Health and Disease

Q.31. Which of the following parameters would you use to check the efficiency of the surveillance system for malaria under the National Vector Borne Disease Control Programme?

  1. Annual Parasite Index
  2. Annual Blood Examination Rate
  3. Slide positivity rate
  4. Slide falciparum rate

Correct Answer: Annual Blood Examination Rate

Topic: Screening of Diseases

Q.32. Which of the following is the best level of prevention of breast cancer?

  1. Specific protection
  2. Early diagnosis and treatment
  3. Disability limitation
  4. Rehabilitation

Correct Answer: Early diagnosis and treatment

Gynaecology & Obstetrics

Topic: Obstetrics

Sub-Topic: 

Q.33. In which trimester does acute fatty liver manifest most commonly during pregnancy?

  1. First trimester
  2. Second trimester
  3. Third trimester
  4. Both A & B

Correct Answer: Third trimester

Topic: Obstetrics

Q.34. You discover that there is evidence of cardiac malformation in a foetus during a routine USG scan at 16 weeks. What is the minimum level of fasting blood sugar at which you can suspect overt diabetes?

  1. 106 mg/dL
  2. 126 mg/dL
  3. 116 mg/dL
  4. 130 mg/dL

Correct Answer: 126 mg/dL

Forensic Medicine

Topic: IPC Sections 

Q.35. Which Section of the IPC deals with the punishment of perjury?

  1. 191
  2. 192
  3. 193
  4. 197

Correct Answer: 193

Topic: Sexual Jurisprudence

Q.36. Frotteurism is _____________

  1. Obtaining sexual pleasure by wearing clothes of opposite sex
  2. Desire to seek surgery to become member of opposite sex
  3. Sexual gratification by rubbing his private parts against another person
  4. Exposure of one’s genitals to an unsuspecting stranger

Correct Answer: Sexual gratification by rubbing his private parts against another persont

Download complete
Rapid Revision 5.0

Microbiology

Topic: Virology

Q.9. A patient hailing from Delhi presents with fever, arthralgia, and extensive petechial rash for 3 days. Lab investigations revealed a hemoglobin of 9 g/dL, a white blood cell count of 9000 cells/mm3, a platelet count of 20000 cells/mm3, and a prolonged bleeding time. The clotting time was normal. What is the most likely diagnosis?
1. Dengue
2. Malaria
3. Scrub typhus
4. Typhoid

Correct Answer: Dengue

Ophthalmology

Topic: Neuro Ophthalmology

Q.10. A female presents with loss of vision in the right halves of both eyes. Where is the lesion located in the optic pathway?
1. Left optic tract
2. Optic radiation
3. Optic chiasma
4. Right optic tract

Correct Answer: Left optic tract

Topic: Cornea

Q.11. The eye examination of a 20-year-old male shows a golden ring on the iris. What is the next investigation to diagnose the condition?
1. Alpha-1-antitrypsin
2. Alpha-fetoprotein
3. Serum Iron
4. Serum ceruloplasmin

Correct Answer: Serum ceruloplasmin

Surgery

Topic: Urology

Q.12. A 55-year-old male presented with verrucous carcinoma around the glans of the penis. Examination reveals that the inguinal lymph nodes are not enlarged. What is the appropriate management for this patient?
1. Total penectomy
2. CO2 laser excision
3. Topical 5-fluorouracil
4. Partial penectomy

Correct Answer: Partial penectomy

Topic: Gastrointestinal Surgery

Q.13. Which of the following is the most common complication following ligation of the first vessel during abdominoperineal resection for rectal carcinoma?
1. Parasympathetic-bladder dysfunction and retrograde ejaculation.
2. Sympathetic-bladder dysfunction and impotence.
3. Sympathetic-retrograde ejaculation and bladder dysfunction.
4. Sympathetic-impotence and loss of cutaneous sensation in the perineal region

Correct Answer: Sympathetic-retrograde ejaculation and bladder dysfunction.

Dermatology

Topic: Introduction to Dermatology

Q.14. A female patient presented with acne that is not resolving on oral isotretinoin and antibiotics therapy. Which of the following is the next best investigation?
1. Look for dietary triggers.
2. Evaluate for hyperandrogenism
3. Check for antibiotic resistance
4. Look for drug triggers

Correct Answer: Evaluate for hyperandrogenism

Topic: Sexually Transmitted Infections

Q.15. A young woman complains of a painless ulcer in the genital area. It is associated with non-tender inguinal lymphadenopathy. What is the most likely diagnosis?
1. Chancroid
2. Syphilis
3. Herpes genitalis
4. Granuloma inguinale

Correct Answer: Syphilis

Anaesthesia

Topic: Pre-Anaesthetic Evaluation

Q.16. Which of the following is the drug of choice for preoperative antibiotic prophylaxis in a patient undergoing cardiac surgery?
1. Penicillin G
2. Erythromycin
3. Azithromycin
4. Cefazolin

Correct Answer: Cefazolin

ENT

Topic: Ear

Q.17. A female patient with hearing loss is examined and is found to be Rinne negative at 256 Hz and 512 Hz, while Rinne positive at 1024 Hz. What is the expected air conduction and bone conduction gap?
1. 30-45 dB
2. 15-30 dB
3. 45-60 dB
4. >60 dB

Correct Answer: 30-45 dB

Topic: Pharynx

Q.18. A 10-year-old child presents with throat pain, fever, and ear pain. He is diagnosed with recurrent tonsillitis. Which nerve is responsible for the ear pain in this patient?
1. Tympanic branch of the glossopharyngeal nerve
2. Greater auricular nerve
3. Auriculotemporal nerve
4. Auricular branch of the vagus nerve

Correct Answer: Tympanic branch of the glossopharyngeal nerve

Pharmacology

Topic: Antimicrobials : Antibacterial Drugs

Q.19. A bronchial asthma patient on inhalational steroids presented with white patchy lesions on the tongue and buccal mucosa. What is the drug that can be used to treat this condition?
1. Clotrimazole
2. Griseofulvin
3. Terbinafine
4. Flucytosine

Correct Answer: Clotrimazole

Topic: General Pharmacology

Q.20. A pregnant woman with a history of bronchial asthma is in the third stage of labour. Which drugs should be avoided in managing postpartum haemorrhage in this mother?
1. Carboprost
2. Oxytocin
3. Dinoprostone
4. Methyl ergometrine

Correct Answer: Carboprost

Orthopaedics

Topic: Peripheral Nerve Injuries

Q.21. A patient at the orthopaedics OPD complains of troubled sleep at night due to numbness and tingling sensation involving his lateral 3 digits. His symptoms are relieved as he lays his arms hanging from the bed. Which of the following options correctly describes his condition and the test used to assess it?
1. Guyon's canal syndrome, Froment's test
2. Carpal tunnel syndrome, Froment's test
3. Guyon's canal syndrome, Durkan's test
4. Carpal tunnel syndrome, Durkan's test

Correct Answer: Carpal tunnel syndrome, Durkan's test

Topic: Metabolic Disorders

Q.22. A child is brought to the orthopaedics OPD with a deformity in the lower limb and hyperpigmented skin lesions. The x-ray of her thigh is shown below. What is the most likely diagnosis?
1. Non-ossifying fibroma
2. Fibrous dysplasia
3. Paget's disease
4. Osteogenesis imperfecta

Correct Answer: Fibrous dysplasia

Psychiatry

Topic: Schizophrenia Spectrum and Other Psychotic Disorders

Q.23. A schizophrenic patient was prescribed drug A after he did not respond to haloperidol and thioridazine. He now presents with excessive salivation, an increase in blood glucose, and hyperlipidemia. What is drug A?
1. Ziprasidone
2. Risperidone
3. Clozapine
4. Aripiprazole

Correct Answer: Clozapine

Topic: Mood Disorders

Q.24. A woman, who is 4 days postpartum, presented with tearfulness, mood swings, and occasional insomnia. What is the likely diagnosis?
1. Postpartum depression
2. Postpartum blues
3. Postpartum psychosis
4. Postpartum anxiety

Correct Answer: Postpartum blues

Physiology

Topic: General Physiology

Q.25. A man was brought to the emergency after suddenly becoming unconscious while working in the field. On examination, his temperature was 105 degrees Fahrenheit, and his skin turgor was decreased. Which of the following would not be seen in the patient?
1. Tachypnea
2. Hypotension
3. Sweating
4. Red hot skin

Correct Answer: Sweating

Topic: Respiratory System

Q.25. A preterm baby who was delivered at 28 weeks developed respiratory distress syndrome. Which of the following is true about surface tension and compliance in this baby?
1. Surface tension - decreased; Compliance - increased
2. Surface tension - increased; Compliance - decreased
3. Both surface tension and compliance decreased
4. Both surface tension and compliance increased

Correct Answer: Surface tension - increased; Compliance - decreased

Medicine

Topic: Nephrology / Kidney Disease

Q.26. A patient with hyperkalemia and elevated urea levels underwent dialysis. Towards the end of the session, she became drowsy and had a sudden seizure episode. On examination, the patient was hypotensive. What is the treatment for this condition?
1. Bumetanide
2. Ethacrynic acid
3. Nesiritide
4. IV Mannitol

Correct Answer: IV Mannitol

Topic: Endocrinology

Q.27. A female patient presents to the emergency department with severe restlessness, palpitations, and tremors. She is a known case of bronchial asthma. On examination, the neck looks swollen. Blood pressure is elevated, and tachycardia is noted. ECG shows atrial fibrillation. Which of the following drugs is used for immediate management in this patient?
1. Diltiazem
2. Propranolol
3. Esmolol
4. Propylthiouracil

Correct Answer: Diltiazem

PSM

Topic: Nutrition and Health

Q.28. In a village, it is observed that several farmers have crossed gait and use a stick for support to stand up and walk. Due to poor yield from farms, they consume meals containing rice and pulses only. Supplementing their diet with which of the following vitamins could have prevented this?
1. Vitamin A
2. Vitamin D
3. Vitamin C
4. Vitamin B

Correct Answer: Vitamin C

Topic: Epidemiology

Q.29. Research is being conducted to find the association between aniline dye exposure and bladder cancer in workers who have worked in the industry for >20 years. Two groups were formed: one directly involved with dye handling and the other group consisting of office clerks not directly exposed to the dye. Years of occupation were noted from records. What type of study is being performed?
1. Retrospective cohort study
2. Prospective cohort study
3. Case-control study
4. Intervention and response

Correct Answer: Retrospective cohort study

Gynaecology & Obstetrics

Topic: Obstetrics

Q.30. A woman presents to you at 36 weeks of gestation with complaints of breathlessness and excessive abdominal distension. Fetal movements are normal. On examination, fetal parts are not easily felt and the fetal heartbeat is heard but it is muffled. Her symphysis fundal height is 41 cm. Her abdomen is tense but not tender. What is the most likely diagnosis?
1. Abruptio placenta
2. Hydrocephalus of fetus
3. Polyhydramnios
4. Fetal-maternal ascites

Correct Answer: Polyhydramnios

Topic: Obstetrics

Q.31. A primigravida presents to you with anemia early in her pregnancy. She is 7 weeks pregnant as seen on ultrasound. Her hemoglobin level is 9 g/dL. When should the iron supplements be started for her?
1. 10 to 12 weeks
2. 8 to 10 weeks
3. After 14 weeks
4. After 20 weeks

Correct Answer: 8 to 10 weeks

Forensic Medicine

Topic: Forensic Toxicology

Q.32. A child before playing consumed fruit from the garden. After some time he developed a high fever, confusion, photophobia, and unable to urinate. What are the likely causative agent and the appropriate antidote used in this case?
1. Datura, Pralidoxime
2. Datura, Physostigmine
3. Yellow oleander, Pralidoxime
4. Yellow oleander, Physostigmine

Correct Answer: Datura, Physostigmine

Topic: Forensic Thanatology

Q.33. The method of autopsy carried out en masse to remove from tongue to prostate is.
1. Virchow technique
2. Rokitansky technique
3. Ghon technique
4. Letulle technique

Correct Answer: Letulle technique

Pediatrics

Topic: Fluid and Electrolyte Disturbances

Q.34. A 10-year-old child weighing 30 kg presents with a history of loose stools for 2 days. On examination, there is severe dehydration. Laboratory investigations are as follows. What is the initial management as per ISPAD guidelines?

RBS550 mg/dL
pH7.01
Na+158mEq/L
Urine glucose3+
  1. Manage ABC, NS 20 mL/kg and start insulin after 1 hour
  2. Manage ABC, NS 20 mL/kg along with insulin 0.1 IU/kg/hr
  3. Manage ABC, NS 10 mL/kg along with insulin 0.1 IU/kg/hr
  4. Manage ABC, NS 10 mL/kg and start insulin after 1 hour

Correct Answer: Manage ABC, NS 20 mL/kg and start insulin after 1 hour

Topic: Growth

Q.35. Which of the following is the best sign to indicate adequate growth in an infant with a birth weight of 2.8 kg?

  1. Increase in length of 25 centimetres in the first year
  2. Weight gain of 300 grams per month till 1 year
  3. Anterior fontanelle closure by 6 months of age
  4. Weight under the 75th percentile and height under the 25th percentile

Correct Answer: Increase in length of 25 centimetres in the first year.

Download complete

NEET PG 2022 Previous Year Questions

NEET PG 2022 was conducted on May 21, 2022, as a national-level entrance examination for postgraduate medical courses in India. It saw a substantial number of candidates vying for admission to various medical institutes across the country. Check out the NEET PG 2022 previous year questions below - 

Microbiology

Topic: Parasitology

Q.1. Microfilariae with sheathed tail and two nuclei at the tail is suggestive of?
1. Wuchereria bancrofti
2. Brugia malayi
3. Loa loa
4. Onchocerca volvulus

Correct Answer: Brugia malayi

Microbiology

Topic: Systemic Bacteriology

Q.2. A militant presents with rashes all over his body sparing the palms and soles. On examination, he was febrile and lice were noted. Which of the following is responsible for his condition?
1. Rickettsia typhi
2. Rickettsia prowazekii
3. Rickettsia akari
4. Rickettsia conorii

Correct answer: Rickettsia prowazekii

Anatomy

Topic: Lower Limb

Q.3. A patient underwent surgery for the varicose vein. He now complains of sensory loss over the medial aspect of the leg and foot. Which of the following nerves is most likely to be injured?
1. Sural nerve
2. Superficial peroneal nerve
3. Deep peroneal nerve
4. Saphenous nerve

Correct Answer: Saphenous nerve

Biochemistry

Topic: Carbohydrate Metabolism

Q.4. A patient from a Mediterranean country visits Africa, where he develops malaria. He is treated with primaquine and later develops hemolytic anemia. Deficiency of an enzyme involved in which of the following pathways could be the cause?
1. Glycolysis
2. Gluconeogenesis
3. Hexose monophosphate (HMP) pathway
4. Luebering-Rapoport pathway

Correct Answer: Hexose monophosphate (HMP) pathway

Biochemistry

Topic: Genetics

Q.5. Which of the following disorders follows autosomal recessive inheritance pattern?
1. Huntington’s disease
2. Treacher Collins syndrome
3. Cystic fibrosis
4. Achondroplasia

Correct Answer: Cystic fibrosis

Ophthalmology

Topic: Uveitis

Q.6. A patient presents with a history of penetrating injury to the eye. A diagnosis of sympathetic ophthalmitis was confirmed. Which of the following will be seen?
1. Acute anterior uveitis
2. Pars planitis
3. Panuveitis
4. Chronic anterior uveitis

Correct Answer: Panuveitis

Topic: Glaucoma

Q.7. A diabetic patient presents to you with visual acuity of 6/9 in one eye. Further investigations revealed preretinal hemorrhages with neovascularization at the optic disc. What is the next step in management?
1. Focal laser photocoagulation
2. Pan-retinal photocoagulation
3. Grid laser photocoagulation
4. Scleral buckling

Correct Answer: Pan-retinal photocoagulation

Surgery

Topic: Endocrine Surgery

Q.8. A 45-year-old female patient underwent a thyroidectomy. Three days after the surgery, she developed perioral numbness. Which of the following investigations need to be done for her?
1. Free T3, T4
2. T3, T4, thyroid–stimulating hormone
3. Radioiodine scan
4. Calcium, phosphate, and parathormone levels

Correct Answer: Calcium, phosphate, and parathormone levels

Topic: Cardiothoracic Vascular Surgery

Q.9. Which of the following is most likely to be seen due to the rupture of a saccular aneurysm?
1. Subdural haemorrhage
2. Subarachnoid haemorrhage
3. Intracerebral haemorrhage
4. Hydrocephalus

Correct Answer: Subarachnoid haemorrhage

Dermatology

Topic: Sexually Transmitted Infections

Q.10. A patient presents to you with multiple anogenital warts. The biopsy of these lesions showed squamous atypia. Which of the following human papillomavirus types are considered high-risk?
1. HPV 2
2. HPV 18
3. HPV 6
4. HPV 11

Correct Answer: HPV 18

Topic: Cutaneous Infections

Q.11. A farmer presents you with a cauliflower-shaped mass on foot, which developed after a minor injury. Microscopy shows copper penny bodies. What is the most likely diagnosis?
1. Chromoblastomycosis
2. Blastomycosis
3. Sporotrichosis
4. Phaeohyphomycosis

Correct Answer: Chromoblastomycosis

Pharmacology

Topic: Respiratory Pharmacology

Q.12. A male patient with chronic obstructive pulmonary disease (COPD) was prescribed theophylline. He noticed that his urine output had increased the following day. This action of the drug is mediated through which of the following receptors?
1. Interleukin – 10
2. Histone deacetylase
3. Adenosine A1
4. Beta 2 adrenergic receptors

Correct Answer: Adenosine A1

Topic: Endocrine Pharmacology

Q.13. Which of the following drugs is not likely to cause Pulmonary fibrosis?
1. Metformin
2. Methotrexate
3. Bleomycin
4. Nitrofurantoin

Correct Answer: Metformin

Orthopaedics

Topic: Metabolic Disorders

Q.14. An intrauterine scan at the 13th week of pregnancy showed a fetus with multiple long bone fractures. What is commonly associated with this finding?
1. Achondroplasia
2. Osteogenesis imperfecta
3. Cretinism
4. Marfan syndrome

Correct Answer: Osteogenesis imperfecta

Topic: Spine + Pelvis + Lower Limb Traumatology

Q.15. A male patient presented with a bone fracture following a road traffic accident. After 2 days he developed dyspnea, petechiae involving the whole body, and a fall in oxygen saturation. What is the likely diagnosis?
1. Fat embolism
2. Air embolism
3. Venous thromboembolism
4. Pulmonary hypertension

Correct Answer: Fat embolism

Psychiatry

Topic: Eating Disorders

Q.16. A 16-year-old girl has intense cravings for food. She eats large amounts of food, which is followed by self-induced vomiting. What is the probable diagnosis?

  1. Anorexia nervosa
  2. Bulimia nervosa
  3. Atypical depression
  4. Binge eating disorder

Correct Answer: Bulimia nervosa

Topic: Mood Disorders

Q.17. A woman, who is 4 days postpartum, presented with tearfulness, mood swings, and occasional insomnia. What is the likely diagnosis?

  1. Postpartum depression
  2. Postpartum blues
  3. Postpartum psychosis
  4. Postpartum anxiety

Correct Answer: Postpartum blues

Physiology

Topic: Nerve Muscle Physiology

Q.18. A person after sleeping overnight with the arm under his head now experiences paresis but no numbness in the morning. Which of the following is the best explanation for it?

  1. C fibres are more sensitive to pressure than A fibres
  2. A fibres are more sensitive to hypoxia than B fibres
  3. A fibres are more susceptible to pressure changes than C fibres
  4. A fibres are more susceptible to hypoxia than C fibres

Correct Answer: A fibres are more susceptible to pressure changes than C fibres

Topic: The Nervous System

Q.19. A 65-year-old suffered from a stroke 2 days ago. He now presents with involuntary, violent, and flinging movements of the limbs on one side. What is the likely site of lesion in this patient?

  1. Subthalamic nuclei
  2. Globus pallidus
  3. Putamen
  4. Caudate nucleus

Correct Answer: Subthalamic nuclei

Medicine 

Topic: Neurology

Q.20. A female patient presents to you with a unilateral headache. It is associated with nausea, photophobia, and phonophobia. What is the drug of choice for acute management?

  1. Flunarizine
  2. Sumatriptan
  3. Propranolol
  4. Topiramate

Correct Answer: Sumatriptan

Topic: Rheumatology / Connective Tissue Disorder

Q.21. A patient presents to you with fever, night sweats, ptosis, and bilateral facial nerve palsy. Investigations showed leukocytosis and bilateral hilar lymphadenopathy. Which of the following is the most likely diagnosis?

  1. Sarcoidosis
  2. Tuberculosis
  3. Lymphoma
  4. Hypersensitive pneumonitis

Correct Answer: Sarcoidosis

Pediatrics

Topic: Neonatology

Q.22. A 1 day- old neonate has not passed urine since birth. What is the next step in management?

  1. Continue breast feeding not observed
  2. Admit to NICU’
  3. Start artificial feeding
  4. Start intravenous fluids

Correct Answer: Continue breast feeding not observed

Topic: Fluid and Electrolyte Disturbances

Q.23. A 7 – year old boy presented with abdominal pain, vomiting, oliguria, and periorbital puffiness following chemotherapy. Investigations reveal hyperuricemia, raised creatinine levels, and hyperkalemia. What is the next best step in the management of this condition ?

  1. Hydration
  2. Probenecid
  3. Allopurinol
  4. Rasburicase

Correct Answer: Hydration

PSM

Topic: Concept of Health and Disease

Q.24. The average life expectancy for a woman in Japan is 87 years. Due to recent advances in testing for cervical cancer, there is an increase in life expectancy by 15 years. The healthcare utility value is 0.8. Which of the following can be calculated from the parameters given?

  1. HALE
  2. DALY
  3. DFLE
  4. QALY

Correct Answer: QALY

Topic: Communicable and Non-communicable Diseases

Q.25. Although many animals are implicated in the spread of rabies, dogs are the most common ones. Also, it usually affects children in developing countries. Knowing this, what is the most cost-effective and logical way to reduce the incidence of rabies?

  1. Testing all dogs for rabies
  2. Reduce stray dog population and vaccinate all dogs
  3. Increase the laboratory facilities
  4. Increase capacity of healthcare workers for surveillance

Correct Answer:  Reduce stray dog population and vaccinate all dogs

Gynaecology & Obstetrics

Topic: Obstetrics

Q.26. A woman at 26 weeks of gestation presents for routine evaluation. On examination, fundal height corresponds to 24 weeks. Ultrasonography revealed decreased amniotic fluid. Which of the following conditions would have led to this presentation?

  1. Renal agenesis
  2. Tracheoesophageal fistula
  3. Cardiac abnormalities
  4. Ureteral stricture

Correct answer: Renal agenesis

Topic: Obstetrics

Q.27. A type 1 diabetic mother is on magnesium sulfate infusion post – cesarean section for preeclampsia. She develops delirium and is drowsy. She has a respiratory rate of 10/min, random blood glucose level of 240 mg / dL, oliguria, and bilaterally absent knee reflex. What is the cause of her condition?

  1. Magnesium sulfate toxicity
  2. Diabetic ketoacidosis
  3. Eclampsia
  4. Diabetes insipidus

Correct answer: Magnesium sulfate toxicity

Forensic Medicine

Topic: Court of Law 

Sub-Topic: 

Q.28. A 45-year-old female patient is told about the benefits and complications of a hysterectomy, and she agrees to the procedure. What kind of consent is this?

  1. Informed consent
  2. Implied consent
  3. Opt-out
  4. Passive consent

Correct answer: Informed consent

Topic: Asphyxial Deaths 

Q.29. A dead body is brought for evaluation. On post-mortem examination, a ligature completely encircled the neck, horizontal, and below the thyroid level was seen. There was no dribbling of saliva. What is the cause of death?

  1. Throttling
  2. Ligature strangulation
  3. Gagging
  4. Hanging

Correct answer: Ligature strangulation

Download complete

NEET PG 2021 Previous Year Questions

NEET PG 2021 was a significant national-level entrance examination conducted on September 11, 2021, for aspiring medical professionals seeking admission to postgraduate medical courses in India. Check out the NEET PG 2021 previous year questions below - 

Microbiology

Topic: Parasitology

Q.1. A patient from Uttar Pradesh presented with fever, pallor, and hepatosplenomegaly. Peripheral smear examination showed pancytopenia. Buffy coat examination showed macrophages laden with organisms with a kinetoplast. What is the vector for the likely disease?

  1. Sandfly
  2. Tse-Tse fly
  3. Triatomine bug
  4. Female anopheles mosquito

Correct Answer: Sandfly

Topic: Virology

Q.2. An unimmunized 2-year old child presented with coryza, conjunctivitis, and bluish-white spots in his buccal mucosa near the lower molar teeth. A day later, he developed a maculopapular rash on the face and neck. What is the nature of the causative virus?

ss- Single stranded

ds – Double – stranded

  1. Enveloped ss RNA
  2. Naked ss RNA
  3. Naked ds RNA
  4. Enveloped ds RNA

Correct Answer: Enveloped ss RNA

Radiology

Topic: Basics of Radiology 

Q.3. A patient following a skid resulting in a motor traffic accident was brought to the emergency room 2 hours later. On examination, he was stable with GCS 15/15. The pupil was reactive to light. Tenderness and bruising over the left lower chest wall with petechiae was seen. Severe tenderness was elicited in the left hypochondriac region and BP- 90/50 mm of Hg. What is the best investigation used in the ER?
GCS: Glasgow coma scale
FAST: Focused Assessment with Sonography for Trauma

  1. FAST
  2. X-ray
  3. Diagnostic peritoneal lavage
  4. CT scan

Correct Answer: FAST

Topic: Radiotherapy

Q.4. A child with acute lymphoblastic leukaemia undergoes prophylactic irradiation prior autologous hematopoietic stem cell transplantation. Which of the following will be the least affected?

  1. Spermatogonia
  2. Intestinal epithelial cells
  3. Neurons
  4. Bone marrow/ erythroid precursor cells

Correct Answer: Neurons 

Anatomy

Topic: Neuro Anatomy

Q.5. A patient presented with vision loss. On radiological investigation, an aneurysm causing damage to the optic chiasma was noted. Which of the following arteries is most likely to be the artery that is causing the damage?

  1. Anterior communicating artery
  2. Anterior choroidal artery
  3. Middle cerebral artery
  4. Anterior cerebral artery

Correct Answer: Anterior communicating artery

Topic: Upper Limb

Q.6. A 7-year-old boy was brought to the hospital with multiple fractures of the humerus secondary to a fall from height. On examination, there is difficulty in flexion of the elbow and supination of the forearm and associated loss of sensation over the lateral aspect of the forearm. Which is the nerve most likely to be injured?

  1. Median nerve
  2. Radial nerve
  3. Musculocutaneous nerve
  4. Ulnar Nerve

Correct Answer: Musculocutaneous nerve

Biochemistry

Topic: Lipid Chemistry

Q.7. A 5 year old boy presented with easy fatigability, irritability and inability to concentrate. Labs revealed the following findings a and b (light and oil immersion respectively) on the bone marrow aspiration. Which of the following is the most likely enzyme deficient in this condition?

  1. Hexosaminidase
  2. Glucocerebrosidase
  3. Sphingomyelinase
  4. N-acetylglucosaminidase

Correct Answer: Glucocerebrosidase

Topic: Oxidative Phosphorylation 

Q.8. You are conducting an experiment on mitochondrial respiration. You add malate/pyruvate and respiration is normal. You add succinate and respiration is normal. When you add another substance in the presence of pyruvate/succinate and malate, respiration is blocked. Which of the following substances is most likely added?

  1. Rotenone
  2. Antimycin A
  3. Oligomycin
  4. 2,4-dinitrophenol

Correct Answer: Oligomycin

Ophthalmology

Topic: Neuro Ophthalmology

Q.9. A 33-year-old woman presents with complaints of progressive loss of vision in the right halves of both eyes. Where is the lesion located in the optic pathway?

  1. Left optic tract
  2. Right visual cortex
  3. Optic chiasma
  4. Right optic nerve

Correct Answer: Left optic tract

Topic: Optics

Q.10. A 15-year-old girl is not compliant with spectacles for her myopic astigmatism. What would be the appropriate management in her case?

  1. LASIK
  2. Spherical equivalent spectacles
  3. Femto LASIK
  4. Implantable collamer lens

Correct Answer: Spherical equivalent spectacles

Surgery

Topic: Urology

Q.11. A 75-year-old man with prostate carcinoma presents to you with a PSA of 9 ng/mL with a small tumor focus. His Gleason score is 6. What will be your most likely management?
PSA- Prostate-specific antigen

  1. Radical prostatectomy
  2. External beam radiation
  3. Brachytherapy
  4. Active surveillance

Correct Answer: Active surveillance

Topic: Others

Q.12. A patient after a road traffic accident presented with pain in the abdomen. The resident examined the child and found that vitals were stable and tenderness was present in the left lumbar region. Which is the best investigation of choice?

  1. Contrast enhanced CT scan
  2. Retrograde urethrogram
  3. Wait and watch
  4. Emergency laparotomy

Correct Answer: Contrast enhanced CT scan

Dermatology

Topic: Immunobullous Disorders

Q.13. A 30-year-old presented with flaccid bullae on her skin which are easy to rupture. The biopsy revealed a suprabasal split. What is the most likely diagnosis?

  1. Pemphigus vulgaris 
  2. Pemphigus foliaceus
  3. Pemphigus vegetans
  4. Erythema multiforme

Correct Answer: Pemphigus vulgaris 

Topic: Bacterial Infections

Q.14. A patient taking multi-drug therapy (MDT) presents with worsening of existing lesions and nerve involvement. What will be your next best step of action?

  1. Stop MDT, start systemic corticosteroids
  2. Continue MDT, start systemic steroids
  3. Stop MDT thalidomide
  4. Continue MDT, start thalidomide

Correct Answer: Continue MDT, start systemic steroids

Anaesthesia

Topic: Monitoring in Anesthesia

Q.15. A 30-Year-old male patient was intubated for surgery by the final-year resident. Which of the following is the best method to confirm the position of the endotracheal tube?

  1. X-ray chest
  2. Auscultation
  3. End-tidal CO2 concentration
  4. Chest rise

Correct Answer: End-tidal CO2 concentration

Topic: Monitoring in Anesthesia

Q.16. A patient undergoing surgery is maintained on anesthesia with halothane. During the procedure, the patient suddenly developed hyperthermia and muscle rigidity. Which of the following drugs is most likely implicated in this condition?

  1. D-curare
  2. Suxamethonium
  3. Cis-atracurium
  4. Rocuronium

Correct Answer: Suxamethonium

ENT

Topic: Larynx

Q.17. Following total thyroidectomy, a patient started having difficulty in breathing, and repeated attempts to extubate were unsuccessful. The most probable cause is _________

  1. Superior laryngeal nerve injury
  2. Unilateral recurrent laryngeal nerve injury
  3. Bilateral recurrent laryngeal nerve injury
  4. Hematoma

Correct Answer: Bilateral recurrent laryngeal nerve injury

Topic: Nose and Paranasal Sinuses

Q.18. A female patient presents with nasal obstruction, nasal discharge, and loss of smell. On examination, foul-smelling discharge and yellowish – green crusts are present in the nasal cavity. She is found to have merciful anosmia. Which of the following findings can also be seen during the examination of her nose?

  1. Roomy nasal cavity
  2. Nasal polyps
  3. Inferior turbinate hypertrophy
  4. Foreign body

Correct Answer: Roomy nasal cavity

Pharmacology

Topic: CVS Pharmacology

Q.19. A patient with renal insufficiency presented with very low urine output, pedal edema, headache and BP 160/90. Which anti-hypertensive can be administered?

  1. Aliskiren
  2. Chlorthalidone
  3. Amlodipine
  4. Prazosin

Correct Answer: Amlodipine

Topic: Anticancer Drugs 

Q.20. Which of the following interleukin (IL) is inhibited by tocilizumab?

  1. IL-6
  2. IL-2
  3. IL-4
  4. IL-5

Correct Answer: IL-6

Orthopaedics

Topic: Spine + Pelvis + Lower Limb Traumatology

Q.21. A 20-year-old male patient presented with a history of lower backache and early morning stiffness for two years. He also gave a history of bilateral heel pain for 6 months. Which of the following is the most likely diagnosis?

  1. Ankylosing spondylitis
  2. Tuberculosis of the spine
  3. Disc prolapse
  4. Mechanical pain

Correct Answer: Ankylosing spondylitis

Topic: Spine + Pelvis + Lower Limb Traumatology

Q.22. A patient was brought to the hospital with complaints of pain around the left hip joint following a road traffic accident. On examination, the affected limb was flexed, adducted, and medially rotated with obvious shortening. What is the most likely diagnosis?

  1. Anterior hip dislocation
  2. Posterior hip dislocation
  3. Transcervical fracture
  4. Intertrochanteric fracture

Correct Answer: Posterior hip dislocation

Psychiatry

Topic: Psychology 

Q.23. During psychotherapy, the therapist had mixed conscious and unconscious feelings toward his patient. This is known as ___________.

  1. Countertransference
  2. Transference
  3. Dissociation
  4. Preoccupation

Correct Answer: Countertransference

Topic: Substance-Related and Addictive Disorders

Q.24. A patient stopped alcohol consumption for 3 days and presented with irritability, disorientation, paranoid delusions, agitation, visual hallucinations, and altered sensorium. What is the likely diagnosis in this case?

  1. Delirium tremens
  2. Wernicke’s encephalopathy
  3. Korsakoff psychosis
  4. Alcohol-induced psychosis

Correct Answer: Delirium tremens

Physiology

Topic: General Physiology

Q.25. A 30-year-old man weighing 70 kg had a sodium level of 120 mEq/L. Calculate the sodium deficit.

  1. 280 mEq
  2. 480 mEq
  3. 840 mEq
  4. 1400 mEq

Correct Answer: 840 mEq

Topic: General Physiology

Q.26. A body fluid sample is being studied with Na: 10 mEq/L K: 140 mEq/L, Cl: 4 mEq/L. Identify the compartment from which the piece has been obtained.

  1. Interstitial
  2. Intracellular fluid
  3. Extracellular fluid
  4. Plasma

Correct Answer: Intracellular fluid

Medicine 

Topic: Neurology

Q.27. A 37-year-old woman presents with headaches for 6 months. She has been taking analgesics regularly. The headache recently increased in severity for 3 days but was reduced in stopping the analgesic. What is the likely diagnosis?

  1. Medication overuse headache
  2. Tension headache
  3. Chronic migraine
  4. Cluster headache

Correct Answer: Medication overuse headache

Topic: Cardiology

Q.28. A 20-year-old woman presents with breathlessness and chest pain. She is a known case of mitral stenosis. Her pulse is irregularly irregular. No thrombus is seen on echocardiography. What is the best agent to prevent future thrombotic events?

  1. Dabigatran
  2. Aspirin - 150 mg
  3. Oral warfarin
  4. Aspirin + Clopidogrel

Correct Answer: Oral warfarin

Pediatrics

Topic: Paediatric Cardiology

Q.29. In foetal circulation, which vessel carries deoxygenated blood back to the placenta?

  1. Umbilical Vein
  2. Umbilical Artery
  3. Descending aorta
  4. Pulmonary Artery

Correct Answer: Umbilical artery

Topic: Paediatric Neurology

Q.30. A 10-year-old boy presented with seizures. His past history is significant for an episode of fever with rash at 1 year of age which resolved spontaneously. What is the most helpful investigation to diagnose his condition?

  1. lgG measles in CSF
  2. MRI mesial temporal lobe sclerosis
  3. lgM measles in CSF
  4. C1Q4 antibodies in the CSF

Correct Answer: lgG measles in CSF

PSM 

Topic: Environment and Health

Q.31. Which of the following thermometers is used to measure the low air velocity rather than the cooling power of the air?

  1. Kata thermometer
  2. Globe thermometer
  3. Wet thermometer
  4. Dial thermometer

Correct Answer: Kata thermometer

Topic: Environment and Health

Q.32. There is an outbreak of buboes in the community. What is the vector responsible for this condition?

  1. Xenopsylla cheopis
  2. Phlebotomus argentipes
  3. Ixodes tick
  4. Female Anopheles mosquito

Correct Answer: Xenopsylla cheopis

Gynaecology & Obstetrics

Topic: Gynecology

Q.33. A 27 year old female patient was found to have uterus didelphys. Which of the following is not likely to be a complication to this uterine anomaly?

  1. Abortion
  2. Endometriosis
  3. Preterm labour
  4. Transverse lie 

Correct Answer: Transverse lie 

Topic: Obstetrics

Q.34. A pregnant woman at 36 weeks of gestation with a prosthetic valve replacement for mitral stenosis is on warfarin therapy. Her INR is 3. What is the appropriate next step in management ?

LMWH- Low Molecular Weight Heparin

  1. Stop warfarin and start LMWH
  2. Stop warfarin and start heparin
  3. Continue warfarin
  4. Stop warfarin, start LMWH and aspirin

Correct Answer: Stop warfarin and start LMWH

Forensic Medicine

Topic: Forensic Toxicology

Q.35. A guy was playing in a garden. Suddenly, he collapsed and was rushed to the hospital. His friend who played with him in the garden informed the doctors that he ate one of the fruits in the garden post which he seemed to have developed these symptoms. He also had irritability, restlessness, dry hot skin, and was unable to pass urine and stools. Identify the poison and its appropriate antidote.

  1. Datura, Pralidoxime
  2. Datura, Physostigmine
  3. Yellow oleander, Digoxin
  4. Yellow oleander, Physostigmine

Correct Answer: Datura, Physostigmine

Topic: Forensic Thanatology 

Q.36. A man was working in a field and he suddenly collapsed on a hot summer afternoon. On examination, there were no physical signs of dehydration. His serum electrolyte level was normal. The doctor found that the body temperature was 106 degrees Fahrenheit. Which of the following symptoms is least likely to be seen in this patient?

  1. Hot skin
  2. Hypotension
  3. Sweating 
  4. Disorientation

Correct Answer: Sweating

Download complete

NEET PG 2020 Previous Year Questions

NEET PG 2020 was held on January 5, 2020, for medical graduates seeking admission to postgraduate medical courses in India. Check out the NEET PG 2020 previous year questions below- 

Microbiology

Topic: Mycology

Q.1. A 35-year-old HIV-positive patient presents with mucosal lesions in the mouth as shown in the image below. On microscopy , bugging yeasts and pseudohyphae are seen. What is the likely diagnosis?

  1. Oral candidiasis
  2. Hairy leukoplakia 
  3. Lichen planus
  4. Diphtheria

Correct Answer: Oral candidiasis

Topic: Virology

Q.2. Zika virus is transmitted by:

  1. Aedes aegypti
  2. Culex
  3. Anopheles
  4. Phlebotomus papatasii

Correct Answer: Aedes aegypti

Radiology

Topic: Head & Neck Imaging 

Q.3. A cystic lesion in the suprasellar region with calcification is seen on the MRI. What is the most likely diagnosis?

  1. Craniopharyngioma
  2. Pituitary adenoma
  3. Meningioma
  4. Oligodendroglioma

Correct Answer: Craniopharyngioma

Topic: OBGyn Imaging

Q.4. Personal monitoring of radiation is done by:

  1. TLD badge
  2. Collimators
  3. Linear accelerator
  4. Grid

Correct Answer: TLD badge

Anatomy

Topic: Back Region

Q.5. The muscle marked by the arrow in the image below is innervated by the :

  1. Dorsal scapular nerve
  2. Suprascapular nerve
  3. From the dorsal rami of C1
  4. Subscapular nerve

Correct Answer: Dorsal scapular nerve

Topic: Upper Limb

Q.6. Which of the following forms the lateral boundary of the anatomical snuff box?

  1. Extensor pollicis brevis and abductor pollicis longus
  2. Extensor pollicis longus and abductor pollicis brevis
  3. Extensor pollicis longus and extensor pollicis brevis
  4. Abductor pollicis longus and abductor pollicis brevis

Correct Answer: Extensor pollicis brevis and abductor pollicis longus

Biochemistry

Topic: Introduction

Q.7. What is the function of the proteasome?

  1. Protein folding
  2. Pot-translation modification
  3. Protein degradation
  4. Protein sorting

Correct Answer: Protein degradation

Topic: Amino Acids and Proteins Chemistry

Q.8. Replacing alanine by which amino acid, will increase the UV absorbance of protein at 280 nm wavelength?

  1. Leucine
  2. Proline
  3. Arginine
  4. Tryptophan

Correct Answer: Tryptophan

Ophthalmology

Topic: Cornea

Q.9. Which of the following layers is responsible for maintaining the hydration and transparency of the cornea?

  1. Descemet’s membrane
  2. Endothelial cells
  3. Stroma
  4. Corneal epithelium

Correct Answer: Endothelial cells

Topic: Cornea

Q.10. What is the characteristic feature of a fungal ulcer?

  1. Reverse hypopyon
  2. Dendritic ulcer on a fluorescein dye
  3. Ring abscess
  4. Satellite lesion

Correct Answer: Satellite lesion

Surgery

Topic: Endocrine Surgery

Q.11. What is the most common site of gastrinoma in MEN 1 syndrome?

  1. Jejunum
  2. Ileum
  3. Duodenum
  4. Stomach

Correct Answer: Duodenum

Topic: Cardiothoracic Vascular Surgery

Q.12. A 50-year-old male who is a chronic smoker presented to the hospital with intermittent claudication pain of both thigh and buttock region on walking about 500 m. Which of the following is a likely diagnosis?

  1. Arterial disease involving the superficial femoral artery
  2. Arterial disease with aortoiliac involvement
  3. Femoral venous insufficiency
  4. Arterial disease involving the profunda femoris artery

Correct Answer: Arterial disease with aortoiliac involvement

Dermatology

Topic: Miscellaneous Disorders

Q.13. Which of the following drugs used as nail lacquer belongs to morpholines?

  1. Amorolfine
  2. Oxiconazole
  3. Ciclopirox olamine
  4. Tioconazole

Correct Answer: Amorolfine

Topic: Skin Appendages and their Disorders

Q.14. A child was born with membranes around the body and had ectropion and eclabium. He is brought to the OPD with lesions covering his face, trunk, and extremities. Which of the following is an unlikely diagnosis?

  1. Icthyosis vulgaris
  2. Lamellar ichthyosis
  3. Bathing suit ichthyosis
  4. Harlequin ichthyosis

Correct Answer: Icthyosis vulgaris

Anaesthesia

Topic: Pre-Anaesthetic Evaluation

Q.15. Which of the following investigations provides the most accurate prognostic information with respect to predicting risk of perioperative cardiac complications?

  1. Exercise ECG testing
  2. Dobutamine stress echocardiography
  3. Myocardial perfusion scintigraphy
  4. Coronary angiography

Correct Answer: Dobutamine stress echocardiography

Topic: Neuromuscular Blockade

Q.16. A patient is given a nicotinic receptor antagonist as a muscle relaxant. Which drug is given postoperatively to recover from muscle weakness?

  1. Physostigmine
  2. Neostigmine
  3. Carbachol
  4. Succinylcholine

Correct Answer: Neostigmine

ENT

Topic: Ear

Q.17. From which of the following structures does the saccule develop?

  1. Saculus anterior
  2. Saculus posterior
  3. Pars superior
  4. Pars inferior

Correct Answer: Pars inferior

Topic: Ear

Q.18. What is the surgery done to widen the cartilaginous part of the external auditory canal called?

  1. Meatoplasty
  2. Tympanoplasty
  3. Myringoplasty
  4. Otoplasty

Correct Answer: Meatoplasty

Pharmacology

Topic: Anticancer Drugs 

Q.19. Which of the following is an inhibitor of DNA synthesis?

  1. 6-mercaptopurine
  2. Mitomycin
  3. Actinomycin
  4. Asparaginase

Correct Answer: 6-mercaptopurine

Topic: Renal Pharmacology

Q.20. A patient on lithium developed hypertension. He was started on thiazides for hypertension. After a few days, he developed coarse tremors and other symptoms suggestive of lithium toxicity. Explain the likely mechanism of this interaction.

  1. Thiazide inhibits the metabolism of lithium
  2. Thiazides act as an add on drug to lithium
  3. Thiazides increase the tubular resorption of lithium
  4. Thiazides cause water loss thereby increase lithium levels

Correct Answer: Thiazides increase the tubular resorption of lithium

Orthopaedics

Topic: Arthritis

Q.21. A patient presented with pain in the hand. The joints involved were proximal interphalangeal joint, distal interphalangeal joint and first carpometacarpal joint. The wrist and metacarpophalangeal joints were spared. What is the likely diagnosis?

  1. Osteoarthritis
  2. Rheumatoid arthritis
  3. Psoriatic arthritis
  4. Pseudogout

Correct Answer: Osteoarthritis

Topic: Sports Injuries

Q.22. A maid is playing with a child by spinning him while holding his hands. A few hours later, the child starts crying, does not use his arm, and does not let anybody touch him. What is the possible diagnosis?

  1. Pulled elbow
  2. Olecranon fracture
  3. Fracture head of radius
  4. Elbow dislocation
    Correct Answer: Pulled elbow

Psychiatry

Topic: Substance-Related and Addictive Disorders

Q.23. A 25-year-old male presented to the clinic with a complaint of the feeling of insects crawling under his skin. Which of the following drug abuse can cause the symptoms of this patient?

  1. Cannabis
  2. Cocaine
  3. Amphetamine
  4. Alcohol

Correct Answer: Cocaine

Topic: Mood Disorders

Q.24. A 60-year-old male who lost his wife 3 months back complains that his intestines have become rotten. He feels responsible for his wife’s death and should be sent to prison. He complains of feeling low all the time and has lost interest in daily activities since his wife’s death. What is the most likely diagnosis?

  1. Normal grief reaction
  2. Psychotic depression
  3. Delusional disorder
  4. Schizophrenia

Correct Answer: Psychotic depression

Physiology

Topic: Respiratory System

Q.25. A person develops headache and breathlessness on a trekking expedition following a rapid ascent to over 3000 meters above the sea level. Which of the following is not used in the treatment of this condition?

  1. IV digoxin
  2. Immediate descent
  3. Administration of oxygen
  4. Tablet acetazolamide

Correct Answer: IV digoxin

Topic: Endocrine and Reproductive System

Sub-Topic: 

Q.26. Prolactin level is highest during?

  1. 24 hours after ovulation
  2. 24 hours after delivery
  3. REM sleep
  4. After running four 1 hours

Correct Answer: 24 hours after delivery

Medicine 

Topic: Nephrology / Kidney Disease

Q.27. Which of the following is associated with pauci-immune glomerulonephritis?

  1. SLE nephritis
  2. Anti-GBM glomerulonephritis
  3. IgA nephropathy
  4. Granulomatosis with polyangiitis (GPA)

Correct Answer: Granulomatosis with polyangiitis (GPA)

Topic: Emergency Medicine

Q.28. At a high altitude of 3000m, a person complains of breathlessness. All of the following can be used for management of this person except

  1. Intravenous digoxin
  2. Oxygen supplementation
  3. Immediate descent
  4. Acetazolamide

Correct Answer: Intravenous digoxin

Pediatrics

Topic: Musculoskeletal Disorders in Children

Q.29. A 4-yr old male child presents with muscle weakness. His mother says that her child has difficulty climbing stairs and getting up from the floor. On muscle biopsy, small muscle fibrils and absence of dystrophin was found. What is the diagnosis out of given options?

  1. Becker's muscle dystrophy
  2. Duchenne muscular dystrophy
  3. Myotonic dystrophy
  4. Limb-girdle muscular dystrophy

Correct Answer: Duchenne muscular dystrophy

Topic: Paediatric Neurology

Q.30. Two girls in the same class are diagnosed with meningococcal meningitis. Their 12-year old friend from the same school is afraid of contracting the disease. What advice should be given to the exposed students?

  1. Two doses of polysaccharide vaccine
  2. Antibiotic prophylaxis
  3. Two doses of conjugate vaccine
  4. Single dose of meningococcal vaccine

Correct Answer: Antibiotic prophylaxis

PSM 

Topic: Health Planning and Management

Q.31. The monetary benefit is measured in which of the following analyses?

  1. Cost-effective analysis
  2. Cost-benefit analysis
  3. Network analysis
  4. Input-Output analysis

Correct Answer: Cost-benefit analysis

Topic: Allied Health Disciplines

Q.32. An employee diagnosed with TB gets extended sickness benefit for?

  1. 1 year
  2. 2 years
  3. 3 years
  4. 4 years

Correct Answer: 2 years

Gynaecology & Obstetrics

Topic: Obstetrics

Q.33. 24 yr old women who had home delivery 2 weeks back, now present with complete perineal tear. What is the next line of management?

  1. Repair immediately
  2. Repair after 3 weeks
  3. Repair after 6 months
  4. Repair after 3 months

Correct Answer: Repair after 3 months

Topic: Gynecology

Q.34. The remnants of Wolffian ducts in females are located in

  1. Pouch of Douglas
  2. Leaves of broad ligament
  3. Uterovesical pouch
  4. Iliac fossa

Correct Answer: Leaves of broad ligament

Forensic Medicine

Topic: Forensic Identification

Q.35. A deceased male was brought to the morgue for a post-mortem examination. His relatives reported that he had a tattoo over his skin, which was not found on examination. Which of the following structures would you examine?

  1. Lymph nodes
  2. Liver
  3. Arteries
  4. Skin

Correct Answer: Lymph nodes

Topic: Forensic Toxicology

Q.36. The most common drug which causes physical dependence is?

  1. Ketamine
  2. Heroin
  3. LSD
  4. Phencyclidine

Correct Answer: Heroin

Download complete

NEET PG 2019 Previous Year Questions

NEET PG 2019 was conducted on 6th January, 2019 with huge participation. Check out the previous year questions below - 

Microbiology

Topic: Systemic Bacteriology

Q.1. BCYE medium is used to culture:

  1. Leptospira
  2. Bacillus cereus
  3. Campylobacter
  4. Legionella

Correct Answer: Legionella

Topic: Systemic Bacteriology

Q.2. A 16-year-old boy is admitted with fever, icterus, conjunctival suffusion, and hematuria for 15 days. Which serological test should be done for diagnosis?

  1. Widal test
  2. Microscopic agglutination test
  3. Weil felix reaction
  4. Paul Bunnell test

Correct Answer: Microscopic agglutination test

Radiology

Topic: Basics of Radiology 

Q.3. What is the best imaging study for the earliest diagnosis of cerebral infarct?

  1. Flair MRI
  2. Non-contrast CT
  3. CECT
  4. Diffusion weighted MRI

Correct Answer: Diffusion weighted MRI

Topic: Basics of Radiology 

Q.4. Which of the following is a water soluble contrast?

  1. Iohexol
  2. Barium 
  3. Borium 
  4. Calcium

Correct Answer: Iohexol

Anatomy

Topic: Back Region

Q.5. The movements at the _______ joint permit a person to look to the right and left.

  1. Atlanto-occipital
  2. Atlanto -axial
  3. C2-C3
  4. C3-C4

Correct Answer: Atlanto -axial

Topic: Abdomen

Q.6. What is the basis for the formation of omphaloceles?

  1. Excessive length of the intestine
  2. Physiological hernia failing to go back
  3. Herniation of the liver
  4. Herniation of umbilicus

Correct Answer: Physiological hernia failing to go back

Biochemistry

Topic: Lipid Metabolism

Q.7. An infant present with hypotonia and seizures. It was confirmed to be cerebrohepatorenal syndrome. Which of the following is accumulated in the brain in cerebrohepatorenal syndrome?

  1. Glucose
  2. Long chain fatty acid
  3. Lactic Acid
  4. Triglycerides

Correct Answer: Long Chain Fatty Acid

Topic: Genetics

Q.8. Which of the following is autosomal dominant?

  1. Cystic fibrosis
  2. Duchene muscular dystrophy
  3. Sickle cell anaemia
  4. Achondroplasia

Correct Answer: Achondroplasia

Ophthalmology

Topic: Optics

Q.9. Which of the following would be prescribed for simple myopic astigmatism?

  1. +1.00 DS
  2. -1.00 DC × 180 Degree
  3. -1.00 DS
  4. -1.00DS – 1.00 DC × 180 Degree

Correct Answer: -1.00 DC × 180 Degree

Topic: Retina

Q.10. Not true about retinitis pigmentosa is:

  1. Retinal pigmentation
  2. Pale waxy disc
  3. Narrowing of vessels
  4. ERG - Normal

Correct Answer: ERG- Normal

Surgery

Topic: Urology

Q.11. Which of the following drugs cause carcinoma bladder?

  1. Cyclophosphamide
  2. Cisplatin
  3. Taxane
  4. Tamoxifen

Correct Answer: Cyclophosphamide

Topic: Gastrointestinal Surgery

Q.12. Dohlman's procedure is for:

  1. Meckel's diverticulum
  2. Zenker’s diverticulum
  3. Bochadlek hernia
  4. Menetrier's disease

Correct Answer: Zenker’s diverticulum

Dermatology

Topic: Eczema

Q.13. Bindi Leukoderma is caused by which chemical?

  1. A Mono-benzyl ether of Hydroquinone (MBH)
  2. Crocein Scarlet MOO and Solvent Yellow 3
  3. p-phenylenediamine (PPD)
  4. Para Tertiary butylphenol (PTBP)

Correct Answer: Para Tertiary butylphenol (PTBP)

Topic: Sexually Transmitted Infections

Q.14. A young female presented with vaginal itching and green frothy genital discharge. Strawberry vagina is seen on examination. What will be the drug of choice?

  1. Doxycycline
  2. Oral fluconazole
  3. Metronidazole
  4. Azithromycin

Correct Answer: Metronidazole

Anaesthesia

Topic: Neuromuscular Blockade

Q.15. What is the mechanism of action of the curare group of muscle relaxants?

  1. Persistently depolarizing at neuromuscular junction
  2. Competitively blocking the binding of ACh to its receptors
  3. Repetitive stimulation of ACh receptors on muscle endplate
  4. Inhibiting the calcium channels on presynaptic membrane

Correct Answer: Competitively blocking the binding of ACh to its receptors

Topic: Inhalational Anaesthetic Agents

Q.16. Which of the following is not used for induction of anaesthesia in paediatric patients?

  1. Halothane
  2. Sevoflurane
  3. Desflurane
  4. Nitrous oxide

Correct Answer: Desflurane

ENT

Topic: Ear

Q.17. Which of the following is not a feature of tubercular otitis media?

  1. Ear ache
  2. Multiple perforations
  3. Pale granulation
  4. Foul smelling ear discharge

Correct Answer: Ear ache

Topic: Nose and Paranasal Sinuses

Q.18. Pott’s puffy tumor is

  1. Subperiosteal abscess of frontal bone
  2. Subperiosteal abscess of ethmoid bone
  3. Mucocele of frontal bone
  4. Mucocele of ethmoid bone

Correct Answer: Subperiosteal abscess of frontal bone

Pharmacology

Topic: Drugs Affecting Blood and Blood formation

Q.19. Which of the following is an oral factor Xa inhibitor?

  1. Bivalirudin
  2. Dabigatran
  3. Rivaroxaban
  4. Enoxaparin

Correct Answer: Rivaroxaban

Topic: Antimicrobials : Antibacterial Drugs

Q.20. What is the mechanism of action of oseltamivir and zanamivir?

  1. DNA polymerase inhibition
  2. Protein synthesis inhibition
  3. Nucleotide analogue
  4. Neuraminidase inhibition

Correct Answer: Neuraminidase inhibition

Orthopaedics

Topic: General + Upper Limb Traumatology

Q.21. Which of the following fractures is most prone to non-union?

  1. Proximal scaphoid
  2. Intertrochanteric
  3. Distal radius
  4. Talar neck

Correct Answer: Proximal scaphoid

Topic: Miscellaneous

Q.22. Degloving injury refers to_____?

  1. Skin and subcutaneous fat are stripped from the underlying fascia
  2. Skin, subcutaneous fat and fascia are stripped from tendons
  3. Skin, subcutaneous fat, fascia and tendons are stripped from bone
  4. Only skin is stripped off

Correct Answer: Skin and subcutaneous fat are stripped from the underlying fascia

Psychiatry

Topic: Substance-Related and Addictive Disorders

Q.23. Which of the following drugs is used as an antismoking agent?

  1. Busulfan
  2. Acamprosate
  3. Varenicline
  4. Gabapentin

Correct Answer: Varenicline

Topic: Schizophrenia Spectrum and Other Psychotic Disorders

Q.24. Which of the following is not a risk factor for delusional disorder?

  1. Recent immigration
  2. Social isolation 
  3. Family history
  4. Young age

Correct Answer: Young age

Physiology

Topic: Excretory System

Q.25. According to myogenic hypothesis of renal autoregulation the afferent arterioles contract in response to stretch – induced by ____________

  1. NO
  2. Noradrenaline
  3. Opening of calcium channels
  4. Adenosine

Correct Answer: Opening of calcium channels

Topic: The Nervous System

Q.26. Which of the following is true for decorticate rigidity?

  1. It is produced by the removal of cerebral cortex and basal ganglia
  2. Flexion of lower limbs and extension of upper limbs occurs
  3. Rigidity is pronounced
  4. It is characterised by flexion of upper limbs and extension of lower limbs

Correct Answer: It is characterised by flexion of upper limbs and extension of lower limbs

Medicine 

Topic: Pulmonology

Q.27. Nasal polyps are commonly associated with: -

  1. Intrinsic Asthma
  2. Brittle Asthma
  3. Extrinsic Asthma
  4. Exercise induced asthma

Correct Answer: Intrinsic Asthma

Topic: Haematology

Q.28. A 47 year old man with a diagnosis of acute myeloid leukaemia with a blood type O negative blood group presents to the transplant clinic to discuss proceeding with an allogeneic stem cell transplant. Which of the following would be an optimal donor?

  1. His identical twin brother
  2. Umbilical cord transplant
  3. His HHLA identical 50-year-old brother who is otherwise healthy and is blood type O+
  4. An HLA identical matched unrelated donor who is blood type

Correct Answer: His HHLA identical 50-year-old brother who is otherwise healthy and is blood type O+

Pediatrics

Topic: Paediatric Gastroenterology

Q.29. Unconjugated hyperbilirubinemia, which did not subside even after 3 weeks of birth, was observed in a neonate. On investigating, liver enzymes, PT/INR and albumin levels were normal. No hemolysis was seen on a peripheral blood smear. A drop in bilirubin level was observed within a week after treatment with phenobarbital. What is the most likely diagnosis?

  1. Rotor syndrome
  2. Crigler Najjar type 2
  3. Dubin Johnson syndrome
  4. Crigler Najjar type 1

Correct Answer: Crigler Najjar type 2

Topic: Pediatric Respiratory Disorders

Q.30. Which of the following is not seen in a child with cystic fibrosis?

  1. Sweat chloride test chloride conc of 70mEq/L
  2. Increase immunoreactive trypsinogen level
  3. Hyperkalemia
  4. Contraction alkalosis

Correct Answer: Hyperkalemia

PSM 

Topic: Concept of Health and Disease

Q.31. Which of the following parameters would you use to check the efficiency of the surveillance system for malaria under the National Vector Borne Disease Control Programme?

  1. Annual Parasite Index
  2. Annual Blood Examination Rate
  3. Slide positivity rate
  4. Slide falciparum rate

Correct Answer: Annual Blood Examination Rate

Topic: Screening of Diseases

Q.32. Which of the following is the best level of prevention of breast cancer?

  1. Specific protection
  2. Early diagnosis and treatment
  3. Disability limitation
  4. Rehabilitation

Correct Answer: Early diagnosis and treatment

Gynaecology & Obstetrics

Topic: Obstetrics

Sub-Topic: 

Q.33. In which trimester does acute fatty liver manifest most commonly during pregnancy?

  1. First trimester
  2. Second trimester
  3. Third trimester
  4. Both A & B

Correct Answer: Third trimester

Topic: Obstetrics

Q.34. You discover that there is evidence of cardiac malformation in a foetus during a routine USG scan at 16 weeks. What is the minimum level of fasting blood sugar at which you can suspect overt diabetes?

  1. 106 mg/dL
  2. 126 mg/dL
  3. 116 mg/dL
  4. 130 mg/dL

Correct Answer: 126 mg/dL

Forensic Medicine

Topic: IPC Sections 

Q.35. Which Section of the IPC deals with the punishment of perjury?

  1. 191
  2. 192
  3. 193
  4. 197

Correct Answer: 193

Topic: Sexual Jurisprudence

Q.36. Frotteurism is _____________

  1. Obtaining sexual pleasure by wearing clothes of opposite sex
  2. Desire to seek surgery to become member of opposite sex
  3. Sexual gratification by rubbing his private parts against another person
  4. Exposure of one’s genitals to an unsuspecting stranger

Correct Answer: Sexual gratification by rubbing his private parts against another persont

Download complete
Rapid Revision 5.0

Pharmacology

Topic: Respiratory Pharmacology

Q.12. A male patient with chronic obstructive pulmonary disease (COPD) was prescribed theophylline. He noticed that his urine output had increased the following day. This action of the drug is mediated through which of the following receptors?
1. Interleukin – 10
2. Histone deacetylase
3. Adenosine A1
4. Beta 2 adrenergic receptors

Correct Answer: Adenosine A1

Topic: Endocrine Pharmacology

Q.13. Which of the following drugs is not likely to cause Pulmonary fibrosis?
1. Metformin
2. Methotrexate
3. Bleomycin
4. Nitrofurantoin

Correct Answer: Metformin

Orthopaedics

Topic: Metabolic Disorders

Q.14. An intrauterine scan at the 13th week of pregnancy showed a fetus with multiple long bone fractures. What is commonly associated with this finding?
1. Achondroplasia
2. Osteogenesis imperfecta
3. Cretinism
4. Marfan syndrome

Correct Answer: Osteogenesis imperfecta

Topic: Spine + Pelvis + Lower Limb Traumatology

Q.15. A male patient presented with a bone fracture following a road traffic accident. After 2 days he developed dyspnea, petechiae involving the whole body, and a fall in oxygen saturation. What is the likely diagnosis?
1. Fat embolism
2. Air embolism
3. Venous thromboembolism
4. Pulmonary hypertension

Correct Answer: Fat embolism

Psychiatry

Topic: Eating Disorders

Q.16. A 16-year-old girl has intense cravings for food. She eats large amounts of food, which is followed by self-induced vomiting. What is the probable diagnosis?

  1. Anorexia nervosa
  2. Bulimia nervosa
  3. Atypical depression
  4. Binge eating disorder

Correct Answer: Bulimia nervosa

Topic: Mood Disorders

Q.17. A woman, who is 4 days postpartum, presented with tearfulness, mood swings, and occasional insomnia. What is the likely diagnosis?

  1. Postpartum depression
  2. Postpartum blues
  3. Postpartum psychosis
  4. Postpartum anxiety

Correct Answer: Postpartum blues

Physiology

Topic: Nerve Muscle Physiology

Q.18. A person after sleeping overnight with the arm under his head now experiences paresis but no numbness in the morning. Which of the following is the best explanation for it?

  1. C fibres are more sensitive to pressure than A fibres
  2. A fibres are more sensitive to hypoxia than B fibres
  3. A fibres are more susceptible to pressure changes than C fibres
  4. A fibres are more susceptible to hypoxia than C fibres

Correct Answer: A fibres are more susceptible to pressure changes than C fibres

Topic: The Nervous System

Q.19. A 65-year-old suffered from a stroke 2 days ago. He now presents with involuntary, violent, and flinging movements of the limbs on one side. What is the likely site of lesion in this patient?

  1. Subthalamic nuclei
  2. Globus pallidus
  3. Putamen
  4. Caudate nucleus

Correct Answer: Subthalamic nuclei

Medicine 

Topic: Neurology

Q.20. A female patient presents to you with a unilateral headache. It is associated with nausea, photophobia, and phonophobia. What is the drug of choice for acute management?

  1. Flunarizine
  2. Sumatriptan
  3. Propranolol
  4. Topiramate

Correct Answer: Sumatriptan

Topic: Rheumatology / Connective Tissue Disorder

Q.21. A patient presents to you with fever, night sweats, ptosis, and bilateral facial nerve palsy. Investigations showed leukocytosis and bilateral hilar lymphadenopathy. Which of the following is the most likely diagnosis?

  1. Sarcoidosis
  2. Tuberculosis
  3. Lymphoma
  4. Hypersensitive pneumonitis

Correct Answer: Sarcoidosis

Pediatrics

Topic: Neonatology

Q.22. A 1 day- old neonate has not passed urine since birth. What is the next step in management?

  1. Continue breast feeding not observed
  2. Admit to NICU’
  3. Start artificial feeding
  4. Start intravenous fluids

Correct Answer: Continue breast feeding not observed

Topic: Fluid and Electrolyte Disturbances

Q.23. A 7 – year old boy presented with abdominal pain, vomiting, oliguria, and periorbital puffiness following chemotherapy. Investigations reveal hyperuricemia, raised creatinine levels, and hyperkalemia. What is the next best step in the management of this condition ?

  1. Hydration
  2. Probenecid
  3. Allopurinol
  4. Rasburicase

Correct Answer: Hydration

PSM

Topic: Concept of Health and Disease

Q.24. The average life expectancy for a woman in Japan is 87 years. Due to recent advances in testing for cervical cancer, there is an increase in life expectancy by 15 years. The healthcare utility value is 0.8. Which of the following can be calculated from the parameters given?

  1. HALE
  2. DALY
  3. DFLE
  4. QALY

Correct Answer: QALY

Topic: Communicable and Non-communicable Diseases

Q.25. Although many animals are implicated in the spread of rabies, dogs are the most common ones. Also, it usually affects children in developing countries. Knowing this, what is the most cost-effective and logical way to reduce the incidence of rabies?

  1. Testing all dogs for rabies
  2. Reduce stray dog population and vaccinate all dogs
  3. Increase the laboratory facilities
  4. Increase capacity of healthcare workers for surveillance

Correct Answer:  Reduce stray dog population and vaccinate all dogs

Gynaecology & Obstetrics

Topic: Obstetrics

Q.26. A woman at 26 weeks of gestation presents for routine evaluation. On examination, fundal height corresponds to 24 weeks. Ultrasonography revealed decreased amniotic fluid. Which of the following conditions would have led to this presentation?

  1. Renal agenesis
  2. Tracheoesophageal fistula
  3. Cardiac abnormalities
  4. Ureteral stricture

Correct answer: Renal agenesis

Topic: Obstetrics

Q.27. A type 1 diabetic mother is on magnesium sulfate infusion post – cesarean section for preeclampsia. She develops delirium and is drowsy. She has a respiratory rate of 10/min, random blood glucose level of 240 mg / dL, oliguria, and bilaterally absent knee reflex. What is the cause of her condition?

  1. Magnesium sulfate toxicity
  2. Diabetic ketoacidosis
  3. Eclampsia
  4. Diabetes insipidus

Correct answer: Magnesium sulfate toxicity

Forensic Medicine

Topic: Court of Law 

Sub-Topic: 

Q.28. A 45-year-old female patient is told about the benefits and complications of a hysterectomy, and she agrees to the procedure. What kind of consent is this?

  1. Informed consent
  2. Implied consent
  3. Opt-out
  4. Passive consent

Correct answer: Informed consent

Topic: Asphyxial Deaths 

Q.29. A dead body is brought for evaluation. On post-mortem examination, a ligature completely encircled the neck, horizontal, and below the thyroid level was seen. There was no dribbling of saliva. What is the cause of death?

  1. Throttling
  2. Ligature strangulation
  3. Gagging
  4. Hanging

Correct answer: Ligature strangulation

Download complete

NEET PG 2021 Previous Year Questions

NEET PG 2021 was a significant national-level entrance examination conducted on September 11, 2021, for aspiring medical professionals seeking admission to postgraduate medical courses in India. Check out the NEET PG 2021 previous year questions below - 

Microbiology

Topic: Parasitology

Q.1. A patient from Uttar Pradesh presented with fever, pallor, and hepatosplenomegaly. Peripheral smear examination showed pancytopenia. Buffy coat examination showed macrophages laden with organisms with a kinetoplast. What is the vector for the likely disease?

  1. Sandfly
  2. Tse-Tse fly
  3. Triatomine bug
  4. Female anopheles mosquito

Correct Answer: Sandfly

Topic: Virology

Q.2. An unimmunized 2-year old child presented with coryza, conjunctivitis, and bluish-white spots in his buccal mucosa near the lower molar teeth. A day later, he developed a maculopapular rash on the face and neck. What is the nature of the causative virus?

ss- Single stranded

ds – Double – stranded

  1. Enveloped ss RNA
  2. Naked ss RNA
  3. Naked ds RNA
  4. Enveloped ds RNA

Correct Answer: Enveloped ss RNA

Radiology

Topic: Basics of Radiology 

Q.3. A patient following a skid resulting in a motor traffic accident was brought to the emergency room 2 hours later. On examination, he was stable with GCS 15/15. The pupil was reactive to light. Tenderness and bruising over the left lower chest wall with petechiae was seen. Severe tenderness was elicited in the left hypochondriac region and BP- 90/50 mm of Hg. What is the best investigation used in the ER?
GCS: Glasgow coma scale
FAST: Focused Assessment with Sonography for Trauma

  1. FAST
  2. X-ray
  3. Diagnostic peritoneal lavage
  4. CT scan

Correct Answer: FAST

Topic: Radiotherapy

Q.4. A child with acute lymphoblastic leukaemia undergoes prophylactic irradiation prior autologous hematopoietic stem cell transplantation. Which of the following will be the least affected?

  1. Spermatogonia
  2. Intestinal epithelial cells
  3. Neurons
  4. Bone marrow/ erythroid precursor cells

Correct Answer: Neurons 

Anatomy

Topic: Neuro Anatomy

Q.5. A patient presented with vision loss. On radiological investigation, an aneurysm causing damage to the optic chiasma was noted. Which of the following arteries is most likely to be the artery that is causing the damage?

  1. Anterior communicating artery
  2. Anterior choroidal artery
  3. Middle cerebral artery
  4. Anterior cerebral artery

Correct Answer: Anterior communicating artery

Topic: Upper Limb

Q.6. A 7-year-old boy was brought to the hospital with multiple fractures of the humerus secondary to a fall from height. On examination, there is difficulty in flexion of the elbow and supination of the forearm and associated loss of sensation over the lateral aspect of the forearm. Which is the nerve most likely to be injured?

  1. Median nerve
  2. Radial nerve
  3. Musculocutaneous nerve
  4. Ulnar Nerve

Correct Answer: Musculocutaneous nerve

Biochemistry

Topic: Lipid Chemistry

Q.7. A 5 year old boy presented with easy fatigability, irritability and inability to concentrate. Labs revealed the following findings a and b (light and oil immersion respectively) on the bone marrow aspiration. Which of the following is the most likely enzyme deficient in this condition?

  1. Hexosaminidase
  2. Glucocerebrosidase
  3. Sphingomyelinase
  4. N-acetylglucosaminidase

Correct Answer: Glucocerebrosidase

Topic: Oxidative Phosphorylation 

Q.8. You are conducting an experiment on mitochondrial respiration. You add malate/pyruvate and respiration is normal. You add succinate and respiration is normal. When you add another substance in the presence of pyruvate/succinate and malate, respiration is blocked. Which of the following substances is most likely added?

  1. Rotenone
  2. Antimycin A
  3. Oligomycin
  4. 2,4-dinitrophenol

Correct Answer: Oligomycin

Ophthalmology

Topic: Neuro Ophthalmology

Q.9. A 33-year-old woman presents with complaints of progressive loss of vision in the right halves of both eyes. Where is the lesion located in the optic pathway?

  1. Left optic tract
  2. Right visual cortex
  3. Optic chiasma
  4. Right optic nerve

Correct Answer: Left optic tract

Topic: Optics

Q.10. A 15-year-old girl is not compliant with spectacles for her myopic astigmatism. What would be the appropriate management in her case?

  1. LASIK
  2. Spherical equivalent spectacles
  3. Femto LASIK
  4. Implantable collamer lens

Correct Answer: Spherical equivalent spectacles

Surgery

Topic: Urology

Q.11. A 75-year-old man with prostate carcinoma presents to you with a PSA of 9 ng/mL with a small tumor focus. His Gleason score is 6. What will be your most likely management?
PSA- Prostate-specific antigen

  1. Radical prostatectomy
  2. External beam radiation
  3. Brachytherapy
  4. Active surveillance

Correct Answer: Active surveillance

Topic: Others

Q.12. A patient after a road traffic accident presented with pain in the abdomen. The resident examined the child and found that vitals were stable and tenderness was present in the left lumbar region. Which is the best investigation of choice?

  1. Contrast enhanced CT scan
  2. Retrograde urethrogram
  3. Wait and watch
  4. Emergency laparotomy

Correct Answer: Contrast enhanced CT scan

Dermatology

Topic: Immunobullous Disorders

Q.13. A 30-year-old presented with flaccid bullae on her skin which are easy to rupture. The biopsy revealed a suprabasal split. What is the most likely diagnosis?

  1. Pemphigus vulgaris 
  2. Pemphigus foliaceus
  3. Pemphigus vegetans
  4. Erythema multiforme

Correct Answer: Pemphigus vulgaris 

Topic: Bacterial Infections

Q.14. A patient taking multi-drug therapy (MDT) presents with worsening of existing lesions and nerve involvement. What will be your next best step of action?

  1. Stop MDT, start systemic corticosteroids
  2. Continue MDT, start systemic steroids
  3. Stop MDT thalidomide
  4. Continue MDT, start thalidomide

Correct Answer: Continue MDT, start systemic steroids

Anaesthesia

Topic: Monitoring in Anesthesia

Q.15. A 30-Year-old male patient was intubated for surgery by the final-year resident. Which of the following is the best method to confirm the position of the endotracheal tube?

  1. X-ray chest
  2. Auscultation
  3. End-tidal CO2 concentration
  4. Chest rise

Correct Answer: End-tidal CO2 concentration

Topic: Monitoring in Anesthesia

Q.16. A patient undergoing surgery is maintained on anesthesia with halothane. During the procedure, the patient suddenly developed hyperthermia and muscle rigidity. Which of the following drugs is most likely implicated in this condition?

  1. D-curare
  2. Suxamethonium
  3. Cis-atracurium
  4. Rocuronium

Correct Answer: Suxamethonium

ENT

Topic: Larynx

Q.17. Following total thyroidectomy, a patient started having difficulty in breathing, and repeated attempts to extubate were unsuccessful. The most probable cause is _________

  1. Superior laryngeal nerve injury
  2. Unilateral recurrent laryngeal nerve injury
  3. Bilateral recurrent laryngeal nerve injury
  4. Hematoma

Correct Answer: Bilateral recurrent laryngeal nerve injury

Topic: Nose and Paranasal Sinuses

Q.18. A female patient presents with nasal obstruction, nasal discharge, and loss of smell. On examination, foul-smelling discharge and yellowish – green crusts are present in the nasal cavity. She is found to have merciful anosmia. Which of the following findings can also be seen during the examination of her nose?

  1. Roomy nasal cavity
  2. Nasal polyps
  3. Inferior turbinate hypertrophy
  4. Foreign body

Correct Answer: Roomy nasal cavity

Pharmacology

Topic: CVS Pharmacology

Q.19. A patient with renal insufficiency presented with very low urine output, pedal edema, headache and BP 160/90. Which anti-hypertensive can be administered?

  1. Aliskiren
  2. Chlorthalidone
  3. Amlodipine
  4. Prazosin

Correct Answer: Amlodipine

Topic: Anticancer Drugs 

Q.20. Which of the following interleukin (IL) is inhibited by tocilizumab?

  1. IL-6
  2. IL-2
  3. IL-4
  4. IL-5

Correct Answer: IL-6

Orthopaedics

Topic: Spine + Pelvis + Lower Limb Traumatology

Q.21. A 20-year-old male patient presented with a history of lower backache and early morning stiffness for two years. He also gave a history of bilateral heel pain for 6 months. Which of the following is the most likely diagnosis?

  1. Ankylosing spondylitis
  2. Tuberculosis of the spine
  3. Disc prolapse
  4. Mechanical pain

Correct Answer: Ankylosing spondylitis

Topic: Spine + Pelvis + Lower Limb Traumatology

Q.22. A patient was brought to the hospital with complaints of pain around the left hip joint following a road traffic accident. On examination, the affected limb was flexed, adducted, and medially rotated with obvious shortening. What is the most likely diagnosis?

  1. Anterior hip dislocation
  2. Posterior hip dislocation
  3. Transcervical fracture
  4. Intertrochanteric fracture

Correct Answer: Posterior hip dislocation

Psychiatry

Topic: Psychology 

Q.23. During psychotherapy, the therapist had mixed conscious and unconscious feelings toward his patient. This is known as ___________.

  1. Countertransference
  2. Transference
  3. Dissociation
  4. Preoccupation

Correct Answer: Countertransference

Topic: Substance-Related and Addictive Disorders

Q.24. A patient stopped alcohol consumption for 3 days and presented with irritability, disorientation, paranoid delusions, agitation, visual hallucinations, and altered sensorium. What is the likely diagnosis in this case?

  1. Delirium tremens
  2. Wernicke’s encephalopathy
  3. Korsakoff psychosis
  4. Alcohol-induced psychosis

Correct Answer: Delirium tremens

Physiology

Topic: General Physiology

Q.25. A 30-year-old man weighing 70 kg had a sodium level of 120 mEq/L. Calculate the sodium deficit.

  1. 280 mEq
  2. 480 mEq
  3. 840 mEq
  4. 1400 mEq

Correct Answer: 840 mEq

Topic: General Physiology

Q.26. A body fluid sample is being studied with Na: 10 mEq/L K: 140 mEq/L, Cl: 4 mEq/L. Identify the compartment from which the piece has been obtained.

  1. Interstitial
  2. Intracellular fluid
  3. Extracellular fluid
  4. Plasma

Correct Answer: Intracellular fluid

Medicine 

Topic: Neurology

Q.27. A 37-year-old woman presents with headaches for 6 months. She has been taking analgesics regularly. The headache recently increased in severity for 3 days but was reduced in stopping the analgesic. What is the likely diagnosis?

  1. Medication overuse headache
  2. Tension headache
  3. Chronic migraine
  4. Cluster headache

Correct Answer: Medication overuse headache

Topic: Cardiology

Q.28. A 20-year-old woman presents with breathlessness and chest pain. She is a known case of mitral stenosis. Her pulse is irregularly irregular. No thrombus is seen on echocardiography. What is the best agent to prevent future thrombotic events?

  1. Dabigatran
  2. Aspirin - 150 mg
  3. Oral warfarin
  4. Aspirin + Clopidogrel

Correct Answer: Oral warfarin

Pediatrics

Topic: Paediatric Cardiology

Q.29. In foetal circulation, which vessel carries deoxygenated blood back to the placenta?

  1. Umbilical Vein
  2. Umbilical Artery
  3. Descending aorta
  4. Pulmonary Artery

Correct Answer: Umbilical artery

Topic: Paediatric Neurology

Q.30. A 10-year-old boy presented with seizures. His past history is significant for an episode of fever with rash at 1 year of age which resolved spontaneously. What is the most helpful investigation to diagnose his condition?

  1. lgG measles in CSF
  2. MRI mesial temporal lobe sclerosis
  3. lgM measles in CSF
  4. C1Q4 antibodies in the CSF

Correct Answer: lgG measles in CSF

PSM 

Topic: Environment and Health

Q.31. Which of the following thermometers is used to measure the low air velocity rather than the cooling power of the air?

  1. Kata thermometer
  2. Globe thermometer
  3. Wet thermometer
  4. Dial thermometer

Correct Answer: Kata thermometer

Topic: Environment and Health

Q.32. There is an outbreak of buboes in the community. What is the vector responsible for this condition?

  1. Xenopsylla cheopis
  2. Phlebotomus argentipes
  3. Ixodes tick
  4. Female Anopheles mosquito

Correct Answer: Xenopsylla cheopis

Gynaecology & Obstetrics

Topic: Gynecology

Q.33. A 27 year old female patient was found to have uterus didelphys. Which of the following is not likely to be a complication to this uterine anomaly?

  1. Abortion
  2. Endometriosis
  3. Preterm labour
  4. Transverse lie 

Correct Answer: Transverse lie 

Topic: Obstetrics

Q.34. A pregnant woman at 36 weeks of gestation with a prosthetic valve replacement for mitral stenosis is on warfarin therapy. Her INR is 3. What is the appropriate next step in management ?

LMWH- Low Molecular Weight Heparin

  1. Stop warfarin and start LMWH
  2. Stop warfarin and start heparin
  3. Continue warfarin
  4. Stop warfarin, start LMWH and aspirin

Correct Answer: Stop warfarin and start LMWH

Forensic Medicine

Topic: Forensic Toxicology

Q.35. A guy was playing in a garden. Suddenly, he collapsed and was rushed to the hospital. His friend who played with him in the garden informed the doctors that he ate one of the fruits in the garden post which he seemed to have developed these symptoms. He also had irritability, restlessness, dry hot skin, and was unable to pass urine and stools. Identify the poison and its appropriate antidote.

  1. Datura, Pralidoxime
  2. Datura, Physostigmine
  3. Yellow oleander, Digoxin
  4. Yellow oleander, Physostigmine

Correct Answer: Datura, Physostigmine

Topic: Forensic Thanatology 

Q.36. A man was working in a field and he suddenly collapsed on a hot summer afternoon. On examination, there were no physical signs of dehydration. His serum electrolyte level was normal. The doctor found that the body temperature was 106 degrees Fahrenheit. Which of the following symptoms is least likely to be seen in this patient?

  1. Hot skin
  2. Hypotension
  3. Sweating 
  4. Disorientation

Correct Answer: Sweating

Download complete

NEET PG 2020 Previous Year Questions

NEET PG 2020 was held on January 5, 2020, for medical graduates seeking admission to postgraduate medical courses in India. Check out the NEET PG 2020 previous year questions below- 

Microbiology

Topic: Mycology

Q.1. A 35-year-old HIV-positive patient presents with mucosal lesions in the mouth as shown in the image below. On microscopy , bugging yeasts and pseudohyphae are seen. What is the likely diagnosis?

  1. Oral candidiasis
  2. Hairy leukoplakia 
  3. Lichen planus
  4. Diphtheria

Correct Answer: Oral candidiasis

Topic: Virology

Q.2. Zika virus is transmitted by:

  1. Aedes aegypti
  2. Culex
  3. Anopheles
  4. Phlebotomus papatasii

Correct Answer: Aedes aegypti

Radiology

Topic: Head & Neck Imaging 

Q.3. A cystic lesion in the suprasellar region with calcification is seen on the MRI. What is the most likely diagnosis?

  1. Craniopharyngioma
  2. Pituitary adenoma
  3. Meningioma
  4. Oligodendroglioma

Correct Answer: Craniopharyngioma

Topic: OBGyn Imaging

Q.4. Personal monitoring of radiation is done by:

  1. TLD badge
  2. Collimators
  3. Linear accelerator
  4. Grid

Correct Answer: TLD badge

Anatomy

Topic: Back Region

Q.5. The muscle marked by the arrow in the image below is innervated by the :

  1. Dorsal scapular nerve
  2. Suprascapular nerve
  3. From the dorsal rami of C1
  4. Subscapular nerve

Correct Answer: Dorsal scapular nerve

Topic: Upper Limb

Q.6. Which of the following forms the lateral boundary of the anatomical snuff box?

  1. Extensor pollicis brevis and abductor pollicis longus
  2. Extensor pollicis longus and abductor pollicis brevis
  3. Extensor pollicis longus and extensor pollicis brevis
  4. Abductor pollicis longus and abductor pollicis brevis

Correct Answer: Extensor pollicis brevis and abductor pollicis longus

Biochemistry

Topic: Introduction

Q.7. What is the function of the proteasome?

  1. Protein folding
  2. Pot-translation modification
  3. Protein degradation
  4. Protein sorting

Correct Answer: Protein degradation

Topic: Amino Acids and Proteins Chemistry

Q.8. Replacing alanine by which amino acid, will increase the UV absorbance of protein at 280 nm wavelength?

  1. Leucine
  2. Proline
  3. Arginine
  4. Tryptophan

Correct Answer: Tryptophan

Ophthalmology

Topic: Cornea

Q.9. Which of the following layers is responsible for maintaining the hydration and transparency of the cornea?

  1. Descemet’s membrane
  2. Endothelial cells
  3. Stroma
  4. Corneal epithelium

Correct Answer: Endothelial cells

Topic: Cornea

Q.10. What is the characteristic feature of a fungal ulcer?

  1. Reverse hypopyon
  2. Dendritic ulcer on a fluorescein dye
  3. Ring abscess
  4. Satellite lesion

Correct Answer: Satellite lesion

Surgery

Topic: Endocrine Surgery

Q.11. What is the most common site of gastrinoma in MEN 1 syndrome?

  1. Jejunum
  2. Ileum
  3. Duodenum
  4. Stomach

Correct Answer: Duodenum

Topic: Cardiothoracic Vascular Surgery

Q.12. A 50-year-old male who is a chronic smoker presented to the hospital with intermittent claudication pain of both thigh and buttock region on walking about 500 m. Which of the following is a likely diagnosis?

  1. Arterial disease involving the superficial femoral artery
  2. Arterial disease with aortoiliac involvement
  3. Femoral venous insufficiency
  4. Arterial disease involving the profunda femoris artery

Correct Answer: Arterial disease with aortoiliac involvement

Dermatology

Topic: Miscellaneous Disorders

Q.13. Which of the following drugs used as nail lacquer belongs to morpholines?

  1. Amorolfine
  2. Oxiconazole
  3. Ciclopirox olamine
  4. Tioconazole

Correct Answer: Amorolfine

Topic: Skin Appendages and their Disorders

Q.14. A child was born with membranes around the body and had ectropion and eclabium. He is brought to the OPD with lesions covering his face, trunk, and extremities. Which of the following is an unlikely diagnosis?

  1. Icthyosis vulgaris
  2. Lamellar ichthyosis
  3. Bathing suit ichthyosis
  4. Harlequin ichthyosis

Correct Answer: Icthyosis vulgaris

Anaesthesia

Topic: Pre-Anaesthetic Evaluation

Q.15. Which of the following investigations provides the most accurate prognostic information with respect to predicting risk of perioperative cardiac complications?

  1. Exercise ECG testing
  2. Dobutamine stress echocardiography
  3. Myocardial perfusion scintigraphy
  4. Coronary angiography

Correct Answer: Dobutamine stress echocardiography

Topic: Neuromuscular Blockade

Q.16. A patient is given a nicotinic receptor antagonist as a muscle relaxant. Which drug is given postoperatively to recover from muscle weakness?

  1. Physostigmine
  2. Neostigmine
  3. Carbachol
  4. Succinylcholine

Correct Answer: Neostigmine

ENT

Topic: Ear

Q.17. From which of the following structures does the saccule develop?

  1. Saculus anterior
  2. Saculus posterior
  3. Pars superior
  4. Pars inferior

Correct Answer: Pars inferior

Topic: Ear

Q.18. What is the surgery done to widen the cartilaginous part of the external auditory canal called?

  1. Meatoplasty
  2. Tympanoplasty
  3. Myringoplasty
  4. Otoplasty

Correct Answer: Meatoplasty

Pharmacology

Topic: Anticancer Drugs 

Q.19. Which of the following is an inhibitor of DNA synthesis?

  1. 6-mercaptopurine
  2. Mitomycin
  3. Actinomycin
  4. Asparaginase

Correct Answer: 6-mercaptopurine

Topic: Renal Pharmacology

Q.20. A patient on lithium developed hypertension. He was started on thiazides for hypertension. After a few days, he developed coarse tremors and other symptoms suggestive of lithium toxicity. Explain the likely mechanism of this interaction.

  1. Thiazide inhibits the metabolism of lithium
  2. Thiazides act as an add on drug to lithium
  3. Thiazides increase the tubular resorption of lithium
  4. Thiazides cause water loss thereby increase lithium levels

Correct Answer: Thiazides increase the tubular resorption of lithium

Orthopaedics

Topic: Arthritis

Q.21. A patient presented with pain in the hand. The joints involved were proximal interphalangeal joint, distal interphalangeal joint and first carpometacarpal joint. The wrist and metacarpophalangeal joints were spared. What is the likely diagnosis?

  1. Osteoarthritis
  2. Rheumatoid arthritis
  3. Psoriatic arthritis
  4. Pseudogout

Correct Answer: Osteoarthritis

Topic: Sports Injuries

Q.22. A maid is playing with a child by spinning him while holding his hands. A few hours later, the child starts crying, does not use his arm, and does not let anybody touch him. What is the possible diagnosis?

  1. Pulled elbow
  2. Olecranon fracture
  3. Fracture head of radius
  4. Elbow dislocation
    Correct Answer: Pulled elbow

Psychiatry

Topic: Substance-Related and Addictive Disorders

Q.23. A 25-year-old male presented to the clinic with a complaint of the feeling of insects crawling under his skin. Which of the following drug abuse can cause the symptoms of this patient?

  1. Cannabis
  2. Cocaine
  3. Amphetamine
  4. Alcohol

Correct Answer: Cocaine

Topic: Mood Disorders

Q.24. A 60-year-old male who lost his wife 3 months back complains that his intestines have become rotten. He feels responsible for his wife’s death and should be sent to prison. He complains of feeling low all the time and has lost interest in daily activities since his wife’s death. What is the most likely diagnosis?

  1. Normal grief reaction
  2. Psychotic depression
  3. Delusional disorder
  4. Schizophrenia

Correct Answer: Psychotic depression

Physiology

Topic: Respiratory System

Q.25. A person develops headache and breathlessness on a trekking expedition following a rapid ascent to over 3000 meters above the sea level. Which of the following is not used in the treatment of this condition?

  1. IV digoxin
  2. Immediate descent
  3. Administration of oxygen
  4. Tablet acetazolamide

Correct Answer: IV digoxin

Topic: Endocrine and Reproductive System

Sub-Topic: 

Q.26. Prolactin level is highest during?

  1. 24 hours after ovulation
  2. 24 hours after delivery
  3. REM sleep
  4. After running four 1 hours

Correct Answer: 24 hours after delivery

Medicine 

Topic: Nephrology / Kidney Disease

Q.27. Which of the following is associated with pauci-immune glomerulonephritis?

  1. SLE nephritis
  2. Anti-GBM glomerulonephritis
  3. IgA nephropathy
  4. Granulomatosis with polyangiitis (GPA)

Correct Answer: Granulomatosis with polyangiitis (GPA)

Topic: Emergency Medicine

Q.28. At a high altitude of 3000m, a person complains of breathlessness. All of the following can be used for management of this person except

  1. Intravenous digoxin
  2. Oxygen supplementation
  3. Immediate descent
  4. Acetazolamide

Correct Answer: Intravenous digoxin

Pediatrics

Topic: Musculoskeletal Disorders in Children

Q.29. A 4-yr old male child presents with muscle weakness. His mother says that her child has difficulty climbing stairs and getting up from the floor. On muscle biopsy, small muscle fibrils and absence of dystrophin was found. What is the diagnosis out of given options?

  1. Becker's muscle dystrophy
  2. Duchenne muscular dystrophy
  3. Myotonic dystrophy
  4. Limb-girdle muscular dystrophy

Correct Answer: Duchenne muscular dystrophy

Topic: Paediatric Neurology

Q.30. Two girls in the same class are diagnosed with meningococcal meningitis. Their 12-year old friend from the same school is afraid of contracting the disease. What advice should be given to the exposed students?

  1. Two doses of polysaccharide vaccine
  2. Antibiotic prophylaxis
  3. Two doses of conjugate vaccine
  4. Single dose of meningococcal vaccine

Correct Answer: Antibiotic prophylaxis

PSM 

Topic: Health Planning and Management

Q.31. The monetary benefit is measured in which of the following analyses?

  1. Cost-effective analysis
  2. Cost-benefit analysis
  3. Network analysis
  4. Input-Output analysis

Correct Answer: Cost-benefit analysis

Topic: Allied Health Disciplines

Q.32. An employee diagnosed with TB gets extended sickness benefit for?

  1. 1 year
  2. 2 years
  3. 3 years
  4. 4 years

Correct Answer: 2 years

Gynaecology & Obstetrics

Topic: Obstetrics

Q.33. 24 yr old women who had home delivery 2 weeks back, now present with complete perineal tear. What is the next line of management?

  1. Repair immediately
  2. Repair after 3 weeks
  3. Repair after 6 months
  4. Repair after 3 months

Correct Answer: Repair after 3 months

Topic: Gynecology

Q.34. The remnants of Wolffian ducts in females are located in

  1. Pouch of Douglas
  2. Leaves of broad ligament
  3. Uterovesical pouch
  4. Iliac fossa

Correct Answer: Leaves of broad ligament

Forensic Medicine

Topic: Forensic Identification

Q.35. A deceased male was brought to the morgue for a post-mortem examination. His relatives reported that he had a tattoo over his skin, which was not found on examination. Which of the following structures would you examine?

  1. Lymph nodes
  2. Liver
  3. Arteries
  4. Skin

Correct Answer: Lymph nodes

Topic: Forensic Toxicology

Q.36. The most common drug which causes physical dependence is?

  1. Ketamine
  2. Heroin
  3. LSD
  4. Phencyclidine

Correct Answer: Heroin

Download complete

NEET PG 2019 Previous Year Questions

NEET PG 2019 was conducted on 6th January, 2019 with huge participation. Check out the previous year questions below - 

Microbiology

Topic: Systemic Bacteriology

Q.1. BCYE medium is used to culture:

  1. Leptospira
  2. Bacillus cereus
  3. Campylobacter
  4. Legionella

Correct Answer: Legionella

Topic: Systemic Bacteriology

Q.2. A 16-year-old boy is admitted with fever, icterus, conjunctival suffusion, and hematuria for 15 days. Which serological test should be done for diagnosis?

  1. Widal test
  2. Microscopic agglutination test
  3. Weil felix reaction
  4. Paul Bunnell test

Correct Answer: Microscopic agglutination test

Radiology

Topic: Basics of Radiology 

Q.3. What is the best imaging study for the earliest diagnosis of cerebral infarct?

  1. Flair MRI
  2. Non-contrast CT
  3. CECT
  4. Diffusion weighted MRI

Correct Answer: Diffusion weighted MRI

Topic: Basics of Radiology 

Q.4. Which of the following is a water soluble contrast?

  1. Iohexol
  2. Barium 
  3. Borium 
  4. Calcium

Correct Answer: Iohexol

Anatomy

Topic: Back Region

Q.5. The movements at the _______ joint permit a person to look to the right and left.

  1. Atlanto-occipital
  2. Atlanto -axial
  3. C2-C3
  4. C3-C4

Correct Answer: Atlanto -axial

Topic: Abdomen

Q.6. What is the basis for the formation of omphaloceles?

  1. Excessive length of the intestine
  2. Physiological hernia failing to go back
  3. Herniation of the liver
  4. Herniation of umbilicus

Correct Answer: Physiological hernia failing to go back

Biochemistry

Topic: Lipid Metabolism

Q.7. An infant present with hypotonia and seizures. It was confirmed to be cerebrohepatorenal syndrome. Which of the following is accumulated in the brain in cerebrohepatorenal syndrome?

  1. Glucose
  2. Long chain fatty acid
  3. Lactic Acid
  4. Triglycerides

Correct Answer: Long Chain Fatty Acid

Topic: Genetics

Q.8. Which of the following is autosomal dominant?

  1. Cystic fibrosis
  2. Duchene muscular dystrophy
  3. Sickle cell anaemia
  4. Achondroplasia

Correct Answer: Achondroplasia

Ophthalmology

Topic: Optics

Q.9. Which of the following would be prescribed for simple myopic astigmatism?

  1. +1.00 DS
  2. -1.00 DC × 180 Degree
  3. -1.00 DS
  4. -1.00DS – 1.00 DC × 180 Degree

Correct Answer: -1.00 DC × 180 Degree

Topic: Retina

Q.10. Not true about retinitis pigmentosa is:

  1. Retinal pigmentation
  2. Pale waxy disc
  3. Narrowing of vessels
  4. ERG - Normal

Correct Answer: ERG- Normal

Surgery

Topic: Urology

Q.11. Which of the following drugs cause carcinoma bladder?

  1. Cyclophosphamide
  2. Cisplatin
  3. Taxane
  4. Tamoxifen

Correct Answer: Cyclophosphamide

Topic: Gastrointestinal Surgery

Q.12. Dohlman's procedure is for:

  1. Meckel's diverticulum
  2. Zenker’s diverticulum
  3. Bochadlek hernia
  4. Menetrier's disease

Correct Answer: Zenker’s diverticulum

Dermatology

Topic: Eczema

Q.13. Bindi Leukoderma is caused by which chemical?

  1. A Mono-benzyl ether of Hydroquinone (MBH)
  2. Crocein Scarlet MOO and Solvent Yellow 3
  3. p-phenylenediamine (PPD)
  4. Para Tertiary butylphenol (PTBP)

Correct Answer: Para Tertiary butylphenol (PTBP)

Topic: Sexually Transmitted Infections

Q.14. A young female presented with vaginal itching and green frothy genital discharge. Strawberry vagina is seen on examination. What will be the drug of choice?

  1. Doxycycline
  2. Oral fluconazole
  3. Metronidazole
  4. Azithromycin

Correct Answer: Metronidazole

Anaesthesia

Topic: Neuromuscular Blockade

Q.15. What is the mechanism of action of the curare group of muscle relaxants?

  1. Persistently depolarizing at neuromuscular junction
  2. Competitively blocking the binding of ACh to its receptors
  3. Repetitive stimulation of ACh receptors on muscle endplate
  4. Inhibiting the calcium channels on presynaptic membrane

Correct Answer: Competitively blocking the binding of ACh to its receptors

Topic: Inhalational Anaesthetic Agents

Q.16. Which of the following is not used for induction of anaesthesia in paediatric patients?

  1. Halothane
  2. Sevoflurane
  3. Desflurane
  4. Nitrous oxide

Correct Answer: Desflurane

ENT

Topic: Ear

Q.17. Which of the following is not a feature of tubercular otitis media?

  1. Ear ache
  2. Multiple perforations
  3. Pale granulation
  4. Foul smelling ear discharge

Correct Answer: Ear ache

Topic: Nose and Paranasal Sinuses

Q.18. Pott’s puffy tumor is

  1. Subperiosteal abscess of frontal bone
  2. Subperiosteal abscess of ethmoid bone
  3. Mucocele of frontal bone
  4. Mucocele of ethmoid bone

Correct Answer: Subperiosteal abscess of frontal bone

Pharmacology

Topic: Drugs Affecting Blood and Blood formation

Q.19. Which of the following is an oral factor Xa inhibitor?

  1. Bivalirudin
  2. Dabigatran
  3. Rivaroxaban
  4. Enoxaparin

Correct Answer: Rivaroxaban

Topic: Antimicrobials : Antibacterial Drugs

Q.20. What is the mechanism of action of oseltamivir and zanamivir?

  1. DNA polymerase inhibition
  2. Protein synthesis inhibition
  3. Nucleotide analogue
  4. Neuraminidase inhibition

Correct Answer: Neuraminidase inhibition

Orthopaedics

Topic: General + Upper Limb Traumatology

Q.21. Which of the following fractures is most prone to non-union?

  1. Proximal scaphoid
  2. Intertrochanteric
  3. Distal radius
  4. Talar neck

Correct Answer: Proximal scaphoid

Topic: Miscellaneous

Q.22. Degloving injury refers to_____?

  1. Skin and subcutaneous fat are stripped from the underlying fascia
  2. Skin, subcutaneous fat and fascia are stripped from tendons
  3. Skin, subcutaneous fat, fascia and tendons are stripped from bone
  4. Only skin is stripped off

Correct Answer: Skin and subcutaneous fat are stripped from the underlying fascia

Psychiatry

Topic: Substance-Related and Addictive Disorders

Q.23. Which of the following drugs is used as an antismoking agent?

  1. Busulfan
  2. Acamprosate
  3. Varenicline
  4. Gabapentin

Correct Answer: Varenicline

Topic: Schizophrenia Spectrum and Other Psychotic Disorders

Q.24. Which of the following is not a risk factor for delusional disorder?

  1. Recent immigration
  2. Social isolation 
  3. Family history
  4. Young age

Correct Answer: Young age

Physiology

Topic: Excretory System

Q.25. According to myogenic hypothesis of renal autoregulation the afferent arterioles contract in response to stretch – induced by ____________

  1. NO
  2. Noradrenaline
  3. Opening of calcium channels
  4. Adenosine

Correct Answer: Opening of calcium channels

Topic: The Nervous System

Q.26. Which of the following is true for decorticate rigidity?

  1. It is produced by the removal of cerebral cortex and basal ganglia
  2. Flexion of lower limbs and extension of upper limbs occurs
  3. Rigidity is pronounced
  4. It is characterised by flexion of upper limbs and extension of lower limbs

Correct Answer: It is characterised by flexion of upper limbs and extension of lower limbs

Medicine 

Topic: Pulmonology

Q.27. Nasal polyps are commonly associated with: -

  1. Intrinsic Asthma
  2. Brittle Asthma
  3. Extrinsic Asthma
  4. Exercise induced asthma

Correct Answer: Intrinsic Asthma

Topic: Haematology

Q.28. A 47 year old man with a diagnosis of acute myeloid leukaemia with a blood type O negative blood group presents to the transplant clinic to discuss proceeding with an allogeneic stem cell transplant. Which of the following would be an optimal donor?

  1. His identical twin brother
  2. Umbilical cord transplant
  3. His HHLA identical 50-year-old brother who is otherwise healthy and is blood type O+
  4. An HLA identical matched unrelated donor who is blood type

Correct Answer: His HHLA identical 50-year-old brother who is otherwise healthy and is blood type O+

Pediatrics

Topic: Paediatric Gastroenterology

Q.29. Unconjugated hyperbilirubinemia, which did not subside even after 3 weeks of birth, was observed in a neonate. On investigating, liver enzymes, PT/INR and albumin levels were normal. No hemolysis was seen on a peripheral blood smear. A drop in bilirubin level was observed within a week after treatment with phenobarbital. What is the most likely diagnosis?

  1. Rotor syndrome
  2. Crigler Najjar type 2
  3. Dubin Johnson syndrome
  4. Crigler Najjar type 1

Correct Answer: Crigler Najjar type 2

Topic: Pediatric Respiratory Disorders

Q.30. Which of the following is not seen in a child with cystic fibrosis?

  1. Sweat chloride test chloride conc of 70mEq/L
  2. Increase immunoreactive trypsinogen level
  3. Hyperkalemia
  4. Contraction alkalosis

Correct Answer: Hyperkalemia

PSM 

Topic: Concept of Health and Disease

Q.31. Which of the following parameters would you use to check the efficiency of the surveillance system for malaria under the National Vector Borne Disease Control Programme?

  1. Annual Parasite Index
  2. Annual Blood Examination Rate
  3. Slide positivity rate
  4. Slide falciparum rate

Correct Answer: Annual Blood Examination Rate

Topic: Screening of Diseases

Q.32. Which of the following is the best level of prevention of breast cancer?

  1. Specific protection
  2. Early diagnosis and treatment
  3. Disability limitation
  4. Rehabilitation

Correct Answer: Early diagnosis and treatment

Gynaecology & Obstetrics

Topic: Obstetrics

Sub-Topic: 

Q.33. In which trimester does acute fatty liver manifest most commonly during pregnancy?

  1. First trimester
  2. Second trimester
  3. Third trimester
  4. Both A & B

Correct Answer: Third trimester

Topic: Obstetrics

Q.34. You discover that there is evidence of cardiac malformation in a foetus during a routine USG scan at 16 weeks. What is the minimum level of fasting blood sugar at which you can suspect overt diabetes?

  1. 106 mg/dL
  2. 126 mg/dL
  3. 116 mg/dL
  4. 130 mg/dL

Correct Answer: 126 mg/dL

Forensic Medicine

Topic: IPC Sections 

Q.35. Which Section of the IPC deals with the punishment of perjury?

  1. 191
  2. 192
  3. 193
  4. 197

Correct Answer: 193

Topic: Sexual Jurisprudence

Q.36. Frotteurism is _____________

  1. Obtaining sexual pleasure by wearing clothes of opposite sex
  2. Desire to seek surgery to become member of opposite sex
  3. Sexual gratification by rubbing his private parts against another person
  4. Exposure of one’s genitals to an unsuspecting stranger

Correct Answer: Sexual gratification by rubbing his private parts against another persont

Download complete
Rapid Revision 5.0

Auther Details

Kashika Walia

Kashika Walia is a Senior Content Writer at PrepLadder. She is devoted to give her readers easily digestible and high-value content that makes their journey towards their dream career incredibly easy.

Top searching words

The most popular search terms used by aspirants

  • important topics NEET PG
  • important topics NEET PG